Re: [obm-l] Prova da IMC - 1o. dia (correcao)

2004-07-26 Por tôpico Cláudio \(Prática\)
Outra forma de resolver o problema da IMC é provar que se um subconjunto de
R tem um número finito de pontos de acumulação, então ele é enumerável. Daí,
basta tomar um ponto e acumulação a, diferente de zero, do conjunto S, um
inteiro positivo n tal que n  2/|a| e n elementos distintos do intervalo
(a-|a|/2,a+|a|/2), cuja soma será  1.

[]s,
Claudio.

- Original Message -
From: Domingos Jr. [EMAIL PROTECTED]
To: [EMAIL PROTECTED]
Sent: Monday, July 26, 2004 12:43 PM
Subject: Re: [obm-l] Prova da IMC - 1o. dia (correcao)


 (um detalhe a + pra esclarecer)















 talvez a parte em que eu afirmo que podemos tomar x  0 não esteja bem
 clara, vou explicar isso melhor (se é que alguém se interessa, hehehe)
 fato: o conjunto dos racionais é enumerável.
 suponha que X = {x : x em S, x  0} seja não-enumerável (se isso não for
 verdade, podemos pegar o conjunto X = {x : x em S, x  0} como
 não enumerável).

 então uma das duas vale
 i) existem dois racionais q, r  0 com q  r e [q, r] inter X é não
 enumerável (que é o assumido na minha dem.)
 ii) os único intervalos [q, r] tq [q, r] inter X é não enumerável são
 aqueles em que q = 0.

 se (ii) é o caso, então podemos tomar um r tq [0, r] é não enumerável
 (sem perda de generalidade, assuma r = 1).
 Agora tome intervalos do tipo [0, q] com q = 2^(-i), i  0 inteiro.
 se [q, r] é não-enumerável, (i) vale, caso contrário,
 cada [2^(-i-1), 2^(-i)] inter X é enumerável e, portanto,
 [0, 1] inter X = União_{i = 0}^oo  {[2^(-i-1), 2^(-i)] inter X} é uma
 união enumerável de conjuntos enumeráveis e, portanto é enumerável.

 acho que agora está ok.

 [ ]'s

  Se S é não-enumerável, há um intervalo [x, y) onde [x, y) inter S é
  infinito, caso contrário, os conjuntos [i, i+1) inter S, com i
  inteiro, são finitos e, portanto, enumeráveis, como uma união
  enumerável (já que os intervalos [i, i+1) são enumeráveis) de conj.
  enumeráveis é enumerável, S seria enumerável.
 
  podemos assumir que x, y  0 sem perda de generalidade, pois podemos
  inverter os sinais de todos os elementos de S e escolher sempre um
  intervalo com x, y  0.
 
  seja k = teto{1/x}
  tome s_1, ..., s_k em [x, y) (k elementos distintos de [x, y) formam
  um conjunto finito), é claro que
  s_1 + ... + s_k  k*x = (1/x)*x = 1.
 
  pra completar, note que o conjunto S = {2^-i | i  0 inteiro}
  é tal que qualquer soma de quaisquer k elementos é menor que 1, basta
  ver que se X = 1/2 + 1/4 + 1/8 + ...
  2X = 1 + 1/2 + 1/4 + ...
  2X - X = 1 = X = 1.
 
  [ ]'s
 
 
=
  Instruções para entrar na lista, sair da lista e usar a lista em
  http://www.mat.puc-rio.br/~nicolau/olimp/obm-l.html
 
=
 

 =
 Instruções para entrar na lista, sair da lista e usar a lista em
 http://www.mat.puc-rio.br/~nicolau/olimp/obm-l.html
 =

=
Instruções para entrar na lista, sair da lista e usar a lista em
http://www.mat.puc-rio.br/~nicolau/olimp/obm-l.html
=


Re: [obm-l] Prova da IMC - 2o. dia

2004-07-26 Por tôpico Cláudio \(Prática\)
O problema 9 parece ser fácil:

Seja w = (z1 + z2 + ... + zn)/n.
Seja z tal que |z| = 1 e 0 = 0+0i esteja entre z e w.
Então |z - w| = |z - 0| = 1 ==
n = n*|z - w| = |nz - nw| = |z - z1| + |z - z2| + ... + |z - zn|.

[]s,
Claudio.

Basta tomar um ponto z pertencente a
- Original Message -
From: Carlos Yuzo Shine [EMAIL PROTECTED]
To: [EMAIL PROTECTED]
Sent: Monday, July 26, 2004 2:03 PM
Subject: [obm-l] Prova da IMC - 2o. dia


 Oi gente,

 A prova inteira (os dois dias) da IMC já está na
 Internet:
http://www.mathlinks.ro/viewtopic.php?t=14534

 []'s
 Shine




 __
 Do you Yahoo!?
 New and Improved Yahoo! Mail - 100MB free storage!
 http://promotions.yahoo.com/new_mail
 =
 Instruções para entrar na lista, sair da lista e usar a lista em
 http://www.mat.puc-rio.br/~nicolau/olimp/obm-l.html
 =

=
Instruções para entrar na lista, sair da lista e usar a lista em
http://www.mat.puc-rio.br/~nicolau/olimp/obm-l.html
=


[obm-l] Re: [obm-l] RE: [obm-l] Cosseno não é Polinômio

2004-05-31 Por tôpico Cláudio \(Prática\)


 Não sei se pode ser simples assim mas...

 supondo Que um polinômio tenha grau N, o número máximo de raízes será N. E
 como as funções do item 1 e item 2 tem infinitas raízes, não pode ser um
 polinômio.

Perfeito. Você também poderia ter argumentado que funções polinomiais são
ilimitadas, o que não é o caso do cosseno. Repare, no entanto, que esse
argumento não vale para x*sen(x).

E se restringíssemos os domínios de f e g a um intervalo compacto?

 Já para o item 3, estou assumindo que um polinômio de grau N é sempre
 derivável, resultando um polinômio de grau N-1, que por sua vez também é
 derivável. Ora, a primeira derivada d e H(x) resulta numa função não
 contínua, o que anula a hipótese.

Na verdade, basta reparar que uma função polinomial é derivável em toda a
reta, mas h'(x) não existe se x é inteiro.


  -Original Message-
 From: Cláudio (Prática) [mailto:[EMAIL PROTECTED]
 Sent: Monday, May 31, 2004 2:17 PM
 To: [EMAIL PROTECTED]
 Subject: [obm-l] Cosseno não é Polinômio



 Achei estas questões interessantes:

 Prove que as seguintes funções de R em R não são funções polinomiais:

 1) f(x) = cos(x).

 2) g(x) = x*sen(x)

 3) h(x) = [x] - x + 2, onde [x] = maior inteiro menor ou igual a x.

 []s,
 Claudio.



=
Instruções para entrar na lista, sair da lista e usar a lista em
http://www.mat.puc-rio.br/~nicolau/olimp/obm-l.html
=


[obm-l] Re: [obm-l] [u] Álgebra

2004-05-20 Por tôpico Cláudio \(Prática\)

- Original Message -
From: Domingos Jr. [EMAIL PROTECTED]
To: [EMAIL PROTECTED]
Sent: Tuesday, May 18, 2004 4:43 PM
Subject: [obm-l] [u] Álgebra


 Esse é bonitinho:
 Seja F um corpo de característica p, mostre que se X^p - X - a é redutível
 em F[X], então ele se decompõe (em fatores lineares) em F[X].

 [ ]'s


Seja E uma extensão de F no qual f(x) tem uma raiz b.
É fácil ver que, nesse caso, para todo r em Z_p, b + r também é raiz, o
que dá um total de p raízes.
Ou seja, em E, f(x) = (x-b)(x-b-1)(x-b-2)...(x-b-p+1).
Se b pertence a F, então acabou.

Suponhamos, portanto, que f(x) não tenha raízes em F mas que seja redutível
sobre F.
Então, f(x) = g(x)h(x), onde g e h pertencem a F[x], grau(g) = n = 1 e
grau(h) = 1.
Podemos supor s.p.d.g. que g(x) é mônico (já que f(x) também é).
Então, sobre E, g(x) = (x - b - r_1)(x - b - r_2)...(x - b - r_n), onde os
r_i são elementos distintos de Z_p (ou mais precisamente, do subcorpo primo
de F, o qual é isomorfo a Z_p).
Como g(x) pertence a F[x], temos que:
coeficiente de x^(n-1) de g(x) =
-(b+r_1)-(b+r_2)-...-(b+r_n) =
-nb - r_1 - ... - r_n = k pertence a F ==
b = -(k + r_1 + ... + r_n)/n pertence a F ==
contradição, pois b é raiz de f(x), o qual não tem raízes em F ==
f(x) é irredutível sobre F.

***

O que podemos dizer sobre a reducibilidade de x^p - x - a sobre Q, onde p é
primo e a é inteiro e primo com p?

[]s,
Claudio.

=
Instruções para entrar na lista, sair da lista e usar a lista em
http://www.mat.puc-rio.br/~nicolau/olimp/obm-l.html
=


[obm-l] Re: [obm-l] Re: [obm-l] Re: [obm-l] RECREAÇÃO!

2004-05-20 Por tôpico Cláudio \(Prática\)
Eu estava pensando no teorema do ponto fixo para contrações, mas sua
sugestão não deixa de ser interessante.

[]s,
Claudio.

- Original Message -
From: Artur Costa Steiner [EMAIL PROTECTED]
To: [EMAIL PROTECTED]
Sent: Thursday, May 20, 2004 5:52 PM
Subject: [obm-l] Re: [obm-l] Re: [obm-l] RECREAÇÃO!


 Isto eh uma aplicacao do principio da casa dos pombos, certo?
 Artur

 - Mensagem Original 
 De: [EMAIL PROTECTED]
 Para: [EMAIL PROTECTED] [EMAIL PROTECTED]
 Assunto: [obm-l] Re: [obm-l] RECREAÇÃO!
 Data: 20/05/04 16:17

 E qual seria uma solução aceitável pra esse aqui?

 Uma mesa é coberta por uma toalha de papel de mesma forma e área.
 Naturalmente, podemos fazer cada ponto da toalha corresponder ao ponto da
 mesa que ele cobre e essa correspondencia é uma bijeção.
 A toalha é então retirada, amassada, e colocada de volta sobre a mesa (sem
 nenhum pedacinho pra fora - ou seja, a toalha amassada está totalmente
 contida no interior da mesa).
 Novamente podemos fazer cada ponto da toalha corresponder ao ponto da mesa
 que ele cobre, só que a correspondencia não é mais uma bijeção.
 Prove que, apesar disso, existe um ponto da toalha que continua a
 corresponder ao mesmo ponto de mesa que correspondia antes da toalha ser
 amassada.

 []s,
 Claudio.

 - Original Message -
 From: Ricardo Bittencourt [EMAIL PROTECTED]
 To: [EMAIL PROTECTED]
 Sent: Wednesday, May 19, 2004 7:34 PM
 Subject: Re: [obm-l] Re: [obm-l] Re: [obm-l] RECREAÇÃO!


  Ricardo Bittencourt wrote:
   Perdoe-me a insistência, mas quando você fez f(t)
   tal que f(0)=0 e f(24)=L, e também g(0)=L e g(24)=0, você
   não está só modelando em matematiquês a mesma resposta
   que ele deu? O raciocínio usado me parece exatamente o mesmo,
   só muda o nome façanha pra teorema do valor intermediário.
   Ele pode não ter sido totalmente formal ao descrever a solução,
   mas eu ainda não consigo ver onde a solução dele é logicamente
   inconsistente.
 
  Aliás deixe eu colocar a dúvida de outra maneira:
  se fosse essa uma questão de olimpíada, a resposta do Will
  seria aceita ou não?
 
  
  Ricardo Bittencourt http://www.mundobizarro.tk
  [EMAIL PROTECTED] tenki ga ii kara sanpo shimashou
  -- União contra o forward - crie suas proprias piadas --
 
=
  Instruções para entrar na lista, sair da lista e usar a lista em
  http://www.mat.puc-rio.br/~nicolau/olimp/obm-l.html
 
=

 =
 Instruções para entrar na lista, sair da lista e usar a lista em
 http://www.mat.puc-rio.br/~nicolau/olimp/obm-l.html
 =

 
 OPEN Internet
 @ Primeiro provedor do DF com anti-vírus no servidor de e-mails @


 =
 Instruções para entrar na lista, sair da lista e usar a lista em
 http://www.mat.puc-rio.br/~nicolau/olimp/obm-l.html
 =

=
Instruções para entrar na lista, sair da lista e usar a lista em
http://www.mat.puc-rio.br/~nicolau/olimp/obm-l.html
=


[obm-l] Re: [obm-l] Fatoração II

2004-05-10 Por tôpico Cláudio \(Prática\)
p(x) = x^6 + x^3 + 1 = (x^9 - 1)/(x^3 - 1)
Ou seja, as raízes de p(x) são as raízes nonas da unidade com exceção de 1,
exp(i*2pi/3) e exp(i*4pi/3).

Seja w = exp(i*2pi/9).
Então as raízes de x^6 + x^3 + 1 são:
w, w^2, w^4, w^(-1), w^(-2) e w^(-4).

w + w^(-1) = 2*cos(2pi/9) = A
w^2 + w^(-2) = 2*cos(4pi/9) = B
w^4 + w^(-4) = 2*cos(8pi/9) = C

Logo:
p(x) = (x - w)*(x - w^(-1))*(x - w^2)*(x - w^(-2))*(x - w^4)*(x - w^(-4))
==
p(x) = (x^2 - Ax + 1)*(x^2 - Bx + 1)*(x^2 - Cx + 1)

Obs 1: A, B e C são irracionais;
Obs 2: p(x) é irredutível sobre Q.

[]s,
Claudio.

- Original Message -
From: niski [EMAIL PROTECTED]
To: [EMAIL PROTECTED]
Sent: Monday, May 10, 2004 1:56 PM
Subject: [obm-l] Fatoração II


 Vamos ver agora.
 Fatore x^6 + x^3 + 1

 Obs. Para evitar respostas do tipo
 1*(x^6 + x^3 + 1) ou sobre o que realmente significa fatorar,
 eu cheguei numa expressao do tipo
 (f(x) - Ax + B)(f(x) - Cx + B)(f(x) - Dx + C) onde A,B,C,D sao
 constantes e f é uma funcao...

 Depois eu coloco exatamente qual foi a minha resposta. Mas sera que
 alguem sabe um jeito simples de fazer?

 Abraços

 --
 Niski - http://www.linux.ime.usp.br/~niski

 [upon losing the use of his right eye]
 Now I will have less distraction
 Leonhard Euler

 =
 Instruções para entrar na lista, sair da lista e usar a lista em
 http://www.mat.puc-rio.br/~nicolau/olimp/obm-l.html
 =

=
Instruções para entrar na lista, sair da lista e usar a lista em
http://www.mat.puc-rio.br/~nicolau/olimp/obm-l.html
=


Re: [obm-l] Demonstracoes no ensino medio

2004-05-10 Por tôpico Cláudio \(Prática\)



Longe de mim discordar da sabedoria daqueles que 
elaboram o currículo de umcurso de matemática. Afinal, eusou apenas 
um amador. Mas me parece que colocar teoria dos números como matéria apenas 
optativa numa graduaçãoem matemática é um absurdo. Até porque 2 dos 7 
problemas do milênio são dessa área. Já era ruim o suficiente o assunto não ser 
mencionado no ensino médio...

[]s,
Claudio.


  - Original Message - 
  From: 
  Johann Peter Gustav Lejeune 
  Dirichlet 
  To: [EMAIL PROTECTED] 
  Sent: Monday, May 10, 2004 4:24 PM
  Subject: Re: [obm-l] Demonstracoes no 
  ensino medio
  
  Como quiser chamar, Teoria dos Numeros. Essa materia e opcional aqui na 
  USP Sao Carlos para o curso de Matematica.E ate divertioda, mas esperar a 
  faculdade para fazer a OBM nivel 3 ja mostra como a coisa ta 
  andando...niski [EMAIL PROTECTED] wrote: 
  Aritimética 
dos Inteiros... Quase ninguem da minha geracao tem conhecimentos basicos 
do assunto. O motivo, acredito que é das duas uma; Ou isso foi ensinado 
em uma epoca errada (i.e o cerebro do aluno, em media, nessa idade, nao 
esta preparado para tal refinamento de ideias) ou isso foi ensinado no 
modo bitolacao. Eu só fui estudar algoritmo de Euclides no 
cursinho. Eu sou professor na PUC, e garanto a você que a 
quase totalidade dos alunos, mesmo os melhores, tem muito a aprender 
com um curso destes. Para tomar um exemplo bem concreto: o conceito 
de mdc e as formas de calculá-lo são matéria de primário mas acho 
que nem um por cento dos calouros saberia descrever o algoritmo de 
Euclides com demonstração.  -- Niski - 
http://www.linux.ime.usp.br/~niski[upon losing the use of his ! 
right eye]"Now I will have less distraction"Leonhard 
Euler=Instruções 
para entrar na lista, sair da lista e usar a lista 
emhttp://www.mat.puc-rio.br/~nicolau/olimp/obm-l.html=r/~nicolau/olimp/obm-l.html=
  
  TRANSIRE SVVM PECTVS MVNDOQVE POTIRI CONGREGATI EX TOTO ORBE 
  MATHEMATICI OB SCRIPTA INSIGNIA TRIBVERE Fields Medal(John Charles 
  Fields)
  
  N.F.C. (Ne Fronti Crede)
  
  
  Yahoo! 
  Messenger - Fale com seus amigos online. Instale 
  agora!


[obm-l] Re: [obm-l] Re: [obm-l] envoltória convexa e conjuntos compactos

2004-04-30 Por tôpico Cláudio \(Prática\)

 Se X eh um conjunto qualquer de objetos e definimos uma metrica em X que
nao
 o faca completo, eh entao verdade que existe um espaco metrico completo
 contendo X como subespaco?

 Artur

Bom, isso eu já não sei dizer porque topologia não é nem de longe a minha
praia, mas me parece que se tornarmos Q (corpo dos racionais) um espaço
métrico com a distância usual d(x,y) = |x - y|, a existência de um espaço
métrico completo que o contenha só é estabelecida por meio do axioma do
supremo.

No caso geral pode ser que você também tenha que postular a existência de um
espaço métrico completo contendo um dado espaço.

De qualquer jeito, acho melhor esperar pela resposta de alguém mais
gabaritado...

[]s,
Claudio.


=
Instruções para entrar na lista, sair da lista e usar a lista em
http://www.mat.puc-rio.br/~nicolau/olimp/obm-l.html
=


[obm-l] p(n+1) - p(n)

2004-04-27 Por tôpico Cláudio \(Prática\)
O que se sabe sobre a seqûencia d(n) = p(n+1) - p(n), onde p(n) = n-ésimo
primo?

O problema abaixo mostra que limsup d(n) = +infinito.

Existe alguma cota inferior conhecida para liminf d(n)?

[]s,
Claudio.

- Original Message -
From: [EMAIL PROTECTED]
To: [EMAIL PROTECTED]
Sent: Saturday, April 24, 2004 10:55 PM
Subject: Re: [obm-l] DUVIDA - Primo


  Alguem pode me dar uma ajuda nesta questão:
 
  Seja p(n) o n-ésimo número primo ( p(1) = 2, p(2) = 3, p(3) = 5 ...).
  Demonstrar que o conjunto formado pelas diferenças p(n + 1) - p(n)
  possui um numero infinito de elementos.
  [...]

 Note que isto equivale a provar que o conjunto das diferenças p(n+1)-p(n)
 contém números arbitrariamente grandes, i.e. para todo N natural, existem
 N naturais compostos consecutivos.

 []s,

 --
 Fábio ctg \pi Dias Moreira


 =
 Instruções para entrar na lista, sair da lista e usar a lista em
 http://www.mat.puc-rio.br/~nicolau/olimp/obm-l.html
 =

=
Instruções para entrar na lista, sair da lista e usar a lista em
http://www.mat.puc-rio.br/~nicolau/olimp/obm-l.html
=


[obm-l] Re: [obm-l] DUVIDA - funçao

2004-04-27 Por tôpico Cláudio \(Prática\)



Outra dúvida:

- Uma função f : A -- B (em que A é o conjunto dos numeros reais 
positivos não - nulos e B o conjunto dos reais)é estritamente crescente e 
para "x" e "y" pertencentes a A temos: f (x.y) = f(x) + f(y) . Sabe-se ainda que 
f(1) = 0 e f(2) = 1. Demonstrar que f(3) é irracional.

É fácilprovar (por exemplo, por indução) que, 
para n inteiro positivo, vale f(x^n) = n*f(x).
Em particular, f(2^n) = n*f(2) = n.
Como f é estritamente crescente, f é 
injetiva.
Logo, f(x) é inteiro positivo == x = 2^n, 
com n inteiro positivo.

Suponhamos que f(3) seja racional, ou seja, f(3) = 
p/q com p,q inteiros positivos primos entre si.
(podemos supor que p e q são ambos positivos porque 
f(3)  f(2)  0).

f(3^q) = q*f(3) = q*(p/q) = p = inteiro positivo 
==
3^q = 2^n para algum inteiro n ==
contradição ao teorema fundamental da aritmética 
==
f(3) é irracional

[]s,
Claudio.


Re: [obm-l] p(n+1) - p(n)

2004-04-27 Por tôpico Cláudio \(Prática\)



Muito obrigado! 
Eu até conhecia essa referência mas nunca tinha 
lido essa parte em particular.

[]s,
Claudio.


  - Original Message - 
  From: 
  Johann Peter Gustav Lejeune 
  Dirichlet 
  To: [EMAIL PROTECTED] 
  Sent: Tuesday, April 27, 2004 12:49 
  PM
  Subject: Re: [obm-l] p(n+1) - p(n)
  
  Bem, o livro do Gugu e do Saldanha, comm o titulo grande, fala um pouco 
  disso...
  
  http://www.mat.puc-rio.br/~nicolau/papers/mersenne/mersenne.htmlCláudio_(Prática) 
  [EMAIL PROTECTED] 
  wrote:
  O 
que se sabe sobre a seqûencia d(n) = p(n+1) - p(n), onde p(n) = 
n-ésimoprimo?O problema abaixo mostra que limsup d(n) = 
+infinito.Existe alguma cota inferior conhecida para liminf 
d(n)?[]s,Claudio.- Original Message -From: 
<[EMAIL PROTECTED]>To: <[EMAIL PROTECTED]>Sent: Saturday, 
April 24, 2004 10:55 PMSubject: Re: [obm-l] DUVIDA - 
Primo  Alguem pode me dar uma ajuda nesta 
questão:   Seja p(n) o n-ésimo número primo ( p(1) = 
2, p(2) = 3, p(3) = 5 ...).  Demonstrar que o conjunto formado 
pelas diferenças p(n + 1) - p(n)  possui um numero infinito de 
elementos.  [...] Note que isto equivale a 
provar que o conjunto das diferenças p(n+1)-p(n) contém números 
arbitrariamente grandes, i.e. para todo N nat! ural, existem N 
naturais compostos consecutivos. []s, 
-- Fábio "ctg \pi" Dias Moreira 
= 
Instruções para entrar na lista, sair da lista e usar a lista em 
http://www.mat.puc-rio.br/~nicolau/olimp/obm-l.html 
==Instruções 
para entrar na lista, sair da lista e usar a lista 
emhttp://www.mat.puc-rio.br/~nicolau/olimp/obm-l.html=
  
  
  TRANSIRE SVVM PECTVS MVNDOQVE 
  POTIRI
  CONGREGATI EX TOTO ORBE MATHEMATICI OB 
  SCRIPTA INSIGNIA TRIBVERE
  Fields Medal(John Charles 
  Fields)
  
  
  Yahoo! 
  Messenger - Fale com seus amigos online. Instale 
  agora!


[obm-l] Re: [obm-l] DUVIDA - funçao

2004-04-27 Por tôpico Cláudio \(Prática\)

- Original Message -
From: Ricardo Bittencourt [EMAIL PROTECTED]
To: [EMAIL PROTECTED]
Sent: Tuesday, April 27, 2004 1:52 PM
Subject: Re: [obm-l] DUVIDA - funçao


 João Silva wrote:

  - Uma função f : A -- B (em que A é o conjunto dos numeros reais
  positivos não - nulos e B o conjunto dos reais) é estritamente crescente
  e para x e y pertencentes a A temos: f (x.y) = f(x) + f(y) . Sabe-se
  ainda que f(1) = 0 e f(2) = 1. Demonstrar que f(3) é irracional.

 f(sqrt(2)*sqrt(2))=2*f(sqrt(2))=f(2)=1
 logo f(sqrt(2))=1/2

 Daí fica claro que uma função f é o log na base 2 né? Pois:

 log2 (1) = 0
 log2 (sqrt(2))=1/2
 log2 (2) = 1

 log2 (ab) = log2 (a) + log2 (b)

 Então resta provar que log2(3) é irracional.
 Pra isso acontecer, 3=2^(p/q) com p,q inteiros.

 Mas então 3^q=2^p, e com p,q inteiros isso é impossível.

 Hum.. resta provar que log2 é a única função f que
 satisfaz o enunciado, isso eu não sei fazer.

Oi, Ricardo:

Acho que o fato de f ser monótona crescente e satisfazer a f(xy) = f(x) +
f(y) implica que f é contínua.
Além disso, da mesma forma que na minha mensagem anterior, podemos provar
que se r é um racional qualquer, então:
f(2^r) = r*f(2) = r.
Finalmente, o conjunto dos números da forma 2^r (r racional) é denso em
(0,+infinito).
(dados a e b com 0  a  b, tome n = menor inteiro positivo tal que 2^(1/n)
 b/a e, uma vez fixado n, tome m = menor inteiro tal que 2^(m/n)  a.
Então, 2^((m-1)/n)  a  2^(m/n) = 2^((m-1)/n)*2^(1/n)  a*2^(1/n)  a*(b/a)
= b ).

Se g:(0,+infinito) - R é tal que:
g é estritamente crescente;
g(xy) = g(x) + g(y) para quaisquer x, y em (0,+infinito);
g(1) = 0 e g(2) = 1,
então, da mesma forma, g é contínua e g(2^r) = r para todo racional r.

Assim, a função F:(0,+infinito) - R dada por:
F(x) = f(x) - g(x)
é uma função contínua que se anula num subconjunto denso em (0,+infinito).
Logo, F é identicamente nula e, portanto, f é única.

[]s,
Claudio.


=
Instruções para entrar na lista, sair da lista e usar a lista em
http://www.mat.puc-rio.br/~nicolau/olimp/obm-l.html
=


Re: [obm-l] Teoria Analitica Elementar dos Numeros (dois problema s legais!!)

2004-04-23 Por tôpico Cláudio \(Prática\)



Acho que o que o Dirichlet quer é que se prove isso:

Existe um numero real"a" euma sequência (f(n)) com a seguinte 
propriedade:
f(0) = a;
f(n+1)=2^f(n) para n = 0;
[f(m)] é primo para m = 0,
onde [x] = maior inteiro que é menor ou igual que x.

[]s,
Claudio.



[obm-l] Re: [obm-l] provas do IME(ROGÉRIO) ?

2004-04-19 Por tôpico Cláudio \(Prática\)



Deixe-me entender: oSérgio tem o maior 
trabalho pra coletar, resolver e digitar as provas. Depois disso ele as coloca à 
disposição de todos na lista e tudo o que você tem a dizer é que a digitação "TÁ 
MUITO CEBOZA"? Francamente...

[]s,
Claudio.


  - Original Message - 
  From: 
  TSD 
  To: [EMAIL PROTECTED] 
  Sent: Sunday, April 18, 2004 2:19 
PM
  Subject: [obm-l] provas do IME(ROGÉRIO) 
  ?
  
  A DIGITAÇÃO TÁ MUITO CEBOZA. NUM TERIA COMO 
  MELHORAR A QUALIDADE DA ESCRITA NOS ENUNCIADOS E RESOLUÇÕES ?
  


Re: [obm-l] GEOMETRIA ESPACIAL

2004-04-19 Por tôpico Cláudio \(Prática\)
Sejam A e B os vértices, P e Q os centros das bases dos cones maior e menor,
respectivamente.
As retas AB e PQ se encontram no ponto M que, de acordo com o enunciado,
estará sobre a borda da base do cone mais baixo, uma vez que a reta BM,
suporte da geratriz do cone menor, passa por A.

Teremos que:
AP = h = altura do cone mais alto;
BQ = k = altura do cone mais baixo;
PM = r + 2s;
QM = s.

Os triângulos APM e BQM são semelhantes. Logo:
AP/PM = BQ/QM ==
h/(r+2s) = k/s ==
h/k = (r+2s)/s = r/s + 2   (1).

Mas também sabemos que os volumes dos dois cones são iguais, de forma que:
(1/3)*Pi*r^2*h = (1/3)*Pi*s^2*k ==
s^2/r^2 = h/k   (2)

(1) e (2) ==
s^2/r^2 = r/s + 2.

Fazendo a = r/s, obtemos:
1/a^2 = a + 2 ==
a^3 + 2a^2 - 1 = 0 ==
(a + 1)*(a^2 + a - 1) = 0 ==
a única raiz positiva é (raiz(5)-1)/2, que é o valor desejado de r/s (a
menos que eu tenha errado alguma conta).

[]s,
Claudio.



- Original Message -
From: Márcio Barbado Jr. [EMAIL PROTECTED]
To: Lista da OBM [EMAIL PROTECTED]
Sent: Monday, April 19, 2004 12:30 PM
Subject: [obm-l] GEOMETRIA ESPACIAL


 Senhores (as)

 Vejam se podem me ajudar com o problema abaixo. Embora possua a
 resposta, não vejo como chegar a ela. A resposta segue após o enunciado.

 Um abraço e obrigado pela atenção

 Dois cones tem suas bases se tangenciando e ambas contidas no mesmo
 plano. O cone de maior altura possui raio r e o outro possui raio s.
 Encontrar o valor da relação r/s sabendo-se que os cones possuem mesmo
 volume e ainda, a reta suporte da geratriz do cone menor passa pelo
vértice
 do maior.

 RESP.: -2 + 2 . [5^(1/2)]



 =
 Instruções para entrar na lista, sair da lista e usar a lista em
 http://www.mat.puc-rio.br/~nicolau/olimp/obm-l.html
 =

=
Instruções para entrar na lista, sair da lista e usar a lista em
http://www.mat.puc-rio.br/~nicolau/olimp/obm-l.html
=


Re: [obm-l] matrizes

2004-04-19 Por tôpico Cláudio \(Prática\)

- Original Message -
From: Raphael Marx [EMAIL PROTECTED]
To: [EMAIL PROTECTED]
Sent: Monday, April 19, 2004 1:58 PM
Subject: [obm-l] matrizes


 Seja a matriz A de ordem n que admite a existêcia de sua inversa A^(-1).
 Sabendo-se que a matriz admite a seguinte propriedade abaixo:
 I e a matriz de identidade de ordem n


 item a

 encontre uma matriz 2x2 onde vale a seguinte relação:
 A + A^(-1) = I

Multiplicando por A e re-arranjando, obtemos A^2 - A + I = 0.
Seja p(x) = x^2 - x + 1, cujas raízes são r = (1+raiz(5))/2 e 1/r =
(1-raiz(5))/2.
Tome A = diag(r,1/r). A é raiz do seu polinômio mínimo, igual a p(x) e,
portanto, satisfaz a relação A + A^(-1) = I.

 item b

 b pertence ao conjunto de inteiros {-2, -1,+1,+2}
 k pertence aos naturais
 A^k  +  A^(-k) = b*I
 prove que b esta limitado somente e apenas somente àqueles valores.para
 qualquer valor de k natural

Usando a matriz A do item (a), teremos:
A^k + A^(-k) = diag( r^k + (1/r)^k , r^k + (1/r)^k ) = (r^k + (1/r)^k)*I.
Agora, basta mostrar que r^k + (1/r)^k pertence a {-2,-1,1,2}, para todo k
natural.
Uma idéia é usar indução.
Outra é encontrar uma relação de recorrência cuja solução seja:
a(k) = r^k + (1/r)^k para todo k natural.
Por exemplo, podemos tomar:
a(1) = 1, a(2) = -1 e, para k = 3, a(k) = a(k-1) - a(k-2).
Isso implica que:
a(3) = -1 - 1 = -2;
a(4) = -2 - (-1) = -1;
a(5) = -1 - (-2) = 1;
a(6) = 1 - (-1) = 2;
a(7) = 2 - 1 = 1 = a(1);
a(8) = 1 - 2 = -1 = a(2)
A partir daí, fica fácil ver que os valores de a(k) se repetem com período
6, de modo que, para todo k natural, a(k) pertence a {-2,-1,1,2}.


[]s,
Claudio.



=
Instruções para entrar na lista, sair da lista e usar a lista em
http://www.mat.puc-rio.br/~nicolau/olimp/obm-l.html
=


Re: [obm-l] inteiros e quadrados perfeitos...

2004-04-12 Por tôpico Cláudio \(Prática\)
Dê uma olhada em:
http://www.kalva.demon.co.uk/imo/imo88.html

É o problema B3.

[]s,
Claudio.

- Original Message -
From: niski [EMAIL PROTECTED]
To: [EMAIL PROTECTED]
Sent: Monday, April 12, 2004 1:54 PM
Subject: [obm-l] inteiros e quadrados perfeitos...


 Eu não sei fazer. Alguem sabe? Como?

 Mostre que dados a,b números naturais então se (a2 + b2)/(ab+1) é um
 numero inteiro, então tambem é um quadrado perfeito

 obrigado
 --
 Niski - http://www.linux.ime.usp.br/~niski

 [upon losing the use of his right eye]
 Now I will have less distraction
 Leonhard Euler

 =
 Instruções para entrar na lista, sair da lista e usar a lista em
 http://www.mat.puc-rio.br/~nicolau/olimp/obm-l.html
 =

=
Instruções para entrar na lista, sair da lista e usar a lista em
http://www.mat.puc-rio.br/~nicolau/olimp/obm-l.html
=


[obm-l] Re: [obm-l] Funções

2004-04-12 Por tôpico Cláudio \(Prática\)

- Original Message -
From: rickufrj [EMAIL PROTECTED]
To: obm-l [EMAIL PROTECTED]
Sent: Monday, April 12, 2004 2:28 PM
Subject: [obm-l] Funções


 SE ALGUEM PUDER AJUDAR A RESOLVER OS SEGUINTES
 PROBLEMMAS:

 1) UMA FUNÇÃO f EM R É DITA PERIÓDICA SE EXISTE T PARA
 TODO X PERTENCENTE A R f(X+T)=f(X).O MENOR T COM ESSA
 PROPRIEDADE É O PERÍODO DA FUNÇÃO . SE f(X) É
 PERIÓDICA DE PERÍODO T , DETERMINE O PERÍODO DE G(X) =
 f(aX + b).

Seja P o período de g. Então g(x+P) = g(x) ==
f(a(x+P) + b) = f(ax + b) ==
f(ax + b + aP) = f(ax + b) ==
aP = T ==
P = T/a.

 2)SEJA f:[a,b] - [a,b] QUALQUER . MOSTRE QUE EXISTE X
 PERTENCENTE A [a,b] TAL QUE f(X) = X.

Isso não é verdade. Tome f(x) dada por:
f(a) = b;
f(x) = a, se x  a.

 3)DETERMINE O CONJUNTO IMAGEM DE f:RxR-R DEFINIDA POR
 f(x,y) = x^2 + (x.y - 1)^2.

É óbvio que f(x,y) = 0, quaisquer que sejam x e y reais.
Qualquer real positivo pertence à imagem de f, pois se x  0, então
f(x,1/x) = x^2.
Por outro lado, não é possível termos f(x,y) = 0, pois isso implicaria que:
0 = (xy - 1)^2 = -x^2 = 0  ==  xy = 1 e x = 0, o que é impossível.
Logo, Im(f) = (0,+infinito).


[]s,
Claudio.

=
Instruções para entrar na lista, sair da lista e usar a lista em
http://www.mat.puc-rio.br/~nicolau/olimp/obm-l.html
=


[obm-l] Re: [obm-l] dúvida!!!

2004-04-12 Por tôpico Cláudio \(Prática\)
Tem um método que é infalível, apesar de ser também totalmente inútil na
prática: veja se (n-1)! é divisível por n (supondo n  4). Se for, então n é
composto. Se não for, então n é primo. Isso é consequência do teorema de
Wilson, que diz que n é primo se e somente se n divide (n-1)! + 1.

[]s,
Claudio.

- Original Message -
From: [EMAIL PROTECTED]
To: [EMAIL PROTECTED]
Sent: Friday, April 09, 2004 10:56 PM
Subject: Re: [obm-l] dúvida!!!


  como eu posso saber de certeza que certo número não primo? alguma idéia
  brilhante senhores
  [...]

 Se n é o tal número, calcule 2^(n-1) (mod n). Se o resultado *não* der 1,
 você tem certeza absoluta de que o número é composto. Por outro lado, se
 der um, você ainda não sabe nada.

 Se você ainda estiver desconfiado, você pode tentar calcular 3^(n-1) (mod
 n) -- se der diferente de um, o número é composto. Novamente, se a conta
 der um, isso não quer dizer nada.

 Você pode repetir isso quantas vezes você quiser, desde que a base da
 potência não seja um múltiplo de n. A mesma coisa vale: se der diferente
 de um, o número é composto, mas não vale a recíproca. Por exemplo, 2^340 -
 1 é divisível por 341 = 31*11.

 Pior, existem números, como o n = 561 = 51*11 tal que, se mdc(a, n) = 1,
 então a^(n-1) - 1 é divisível por n.

 Se você quiser saber mais sobre números primos, o livro Primos de
 Mersenne (e outros primos muito grandes) do Nicolau e do Gugu, disponível
 em

 http://www.mat.puc-rio.br/~nicolau/publ/publ.html

 é um ótimo começo.

 []s,

 --
 Fábio Dias Moreira


 =
 Instruções para entrar na lista, sair da lista e usar a lista em
 http://www.mat.puc-rio.br/~nicolau/olimp/obm-l.html
 =

=
Instruções para entrar na lista, sair da lista e usar a lista em
http://www.mat.puc-rio.br/~nicolau/olimp/obm-l.html
=


Re: [obm-l] Polinomio caracteristico de uma matriz

2004-03-31 Por tôpico Cláudio \(Prática\)
Oi, Artur:

A demonstração-padrão usa a matriz adjunta clássica (transposta da matriz
dos cofatores) e está aqui:
http://www.math.niu.edu/~rusin/known-math/99/ham_cayley
ou aqui:
http://mathforum.org/library/drmath/view/51991.html

Mas você vai gostar mesmo é dessa aqui:
www.math-cs.cmsu.edu/~mjms/1995.2/rosoff.ps

[]s,
Claudio.

- Original Message -
From: Artur Costa Steiner [EMAIL PROTECTED]
To: OBM [EMAIL PROTECTED]
Sent: Wednesday, March 31, 2004 10:55 AM
Subject: [obm-l] Polinomio caracteristico de uma matriz


 Eu sei que a demonstracao do que vou dizer tem em
 qualquer livro de Algebra Linear. Mas, se alguem se
 lembrar e nao for muito complicado (no momento naum
 estou lembrado dos detalhes, mas acho que naum eh
 muito trivial), seria possivel alinhavar a
 demonstracao de que, se A eh uma matriz quadrada e P
 eh seu polinomio caracteristico, entao P(A) = 0?

 Se A for diagonalizavel hah uma prova bem simples, mas
 no caso geral eh mais complicado.

 Artur

 __
 Do you Yahoo!?
 Yahoo! Finance Tax Center - File online. File on time.
 http://taxes.yahoo.com/filing.html
 =
 Instruções para entrar na lista, sair da lista e usar a lista em
 http://www.mat.puc-rio.br/~nicolau/olimp/obm-l.html
 =

=
Instruções para entrar na lista, sair da lista e usar a lista em
http://www.mat.puc-rio.br/~nicolau/olimp/obm-l.html
=


Re: [obm-l] Continuidade

2004-03-31 Por tôpico Cláudio \(Prática\)



Na 1a. use o fato de que a composta de funções 
contínuas é contínua.
Na 2a. idem, mas falta definir que f(0,0) = 
0.

  - Original Message - 
  From: 
  Marcelo Souza 
  To: [EMAIL PROTECTED] 
  Sent: Wednesday, March 31, 2004 3:49 
  PM
  Subject: [obm-l] Continuidade
  
  
  Como demonstrar que
  1. z=sen(x^2+y)
  2. z=[sen(xy)]/[sqrt(x^2+y^2)]
  
  são contínuas. Desde já agradeço
  []'s, Marcelo
  
  MSN Messenger: converse com os seus amigos online. Instale grátis. Clique aqui. 
  = 
  Instruções para entrar na lista, sair da lista e usar a lista em http://www.mat.puc-rio.br/~nicolau/olimp/obm-l.html 
  =


[obm-l] Re: [obm-l] Re:_[obm-l]_Polinômio_Irredutível

2004-03-31 Por tôpico Cláudio \(Prática\)



Até pode ser, mas você consegue dar algum 
contra-exemplo de grau = 2?

[]s,
Claudio.


  - Original Message - 
  From: 
  Johann Peter Gustav Lejeune 
  Dirichlet 
  To: [EMAIL PROTECTED] 
  Sent: Wednesday, March 31, 2004 3:50 
  PM
  Subject: Re: [obm-l] 
  Re:_[obm-l]_Polinômio_Irredutível
  
  e sempre bom tentar fatorar em C antes de se aventurar perigosamente.Mas 
  em geral nao e possivel dizer isso.Cláudio_(Prática) [EMAIL PROTECTED] 
  wrote: 
  



Eu formulei mal a minha dúvida abaixo, pois é 
claro que existem casos mais ou menos óbvios onde o resultado não é verdade. 
Por exemplo, f(x) = x - a^k, com a em F e k  1 == f(x^m) será 
redutível se mdc(m,k)  1.

A dúvida surgiu ao tentar calcular o polinômio 
mínimo de (2^(1/3)- i)^(1/2):
x = (2^(1/3)- i)^(1/2) 
==
x^2+i = 2^(1/3) ==
x^6+ 3ix^4 - 3x^2- i = 2 
==
(x^6 - 3x^2 - 2)^2 = -(1 - 3x^4)^2 
==
x^12 + 3x^8 - 4x^6 + 3x^4 + 12x^2 + 5 = 
0

Olhando essa equação em Z_3, 
obtemos:
x^12 + 2x^6 + 2 = 0.

Foi aí que surgiu a dúvida, pois f(x) = x^2 + 
2x + 2 é irredutível sobre Z_3.
A partir disso, podemos concluir que f(x^6) = 
x^12 + 2x^6 + 2 também é?

[]s,
Claudio.

  - Original Message - 
  From: 
  Cláudio (Prática) 
  To: [EMAIL PROTECTED] 
  Sent: Wednesday, March 31, 2004 12:34 
  PM
  Subject: [obm-l] Polinômio 
  Irredutível
  
  Oi, pessoal:
  
  Tenho a seguinte dúvida:
  Se F é um corpo e f(x) é um polinômio irredutível sobre F, então é 
  verdade que, para cada inteiro positivo n, o polinômio g(x) = f(x^n) 
  também é irredutível sobre F?
  
  Se for verdade, isso vale pra qualquer corpo?
  
  Agradeço qualquer ajuda.
  
  []s,
  Claudio.
  
  
  TRANSIRE SVVM PECTVS MVNDOQVE 
  POTIRI
  CONGREGATI EX TOTO ORBE MATHEMATICI OB 
  SCRIPTA INSIGNIA TRIBVERE
  Fields Medal(John Charles 
  Fields)
  
  
  Yahoo! 
  Mail - O melhor e-mail do Brasil. Abra 
  sua conta agora!


Re: [obm-l] Um limite meio chato

2004-03-31 Por tôpico Cláudio \(Prática\)



sen(x) = x - x^3/6 + O(x^5)
cos(x) = 1 - x^2/2 + O(x^4)

Assim:
sen(x)/x^3 - cos(x)/x^2 = 
1/x^2 - 1/6 + O(x^2) - 1/x^2 + 1/2 + O(x^2) 
=
1/3 + O(x^2)

Logo, o limite é igual a 1/3.

[]s,
Claudio.

  - Original Message - 
  From: 
  Johann Peter Gustav Lejeune 
  Dirichlet 
  To: [EMAIL PROTECTED] 
  Sent: Wednesday, March 31, 2004 4:11 
  PM
  Subject: [obm-l] Um limite meio 
  chato
  
  Ola pessoal!!!
  Certa feita fui desafiado a dizer o limite desta expressao quando x tende 
  a zero:
  
  sen x/x^3- cosx/x^2.
  
  Pequeno detalhe: na epoca usei L'Hopital-Bernoulli mas ai nao tinha 
  graça...
  Agora eu queria que ces me ajudassem nesse sentido:demonstrar 
  elementarmente essa coisinha.Ai pensei em usar serie de Taylor e consegui 
  resolver,mas ainda e complicado (nada que toque em derivadas nem muito 
  alem).
  Mas ai me veio uma ideia: que tal adaptar Taylor?Assim:provar que 
  x-x^3/3!+x^5/5! e a melhor aproximaçao de um polinomio de grau 5 
  desen x e depois algo parecido com xcos x, e demonstrar tudo a prtir 
  dai...
  Captaram?E entao, alguma ajuda?
  
  Ass.:Johann
  
  
  TRANSIRE SVVM PECTVS MVNDOQVE 
  POTIRI
  CONGREGATI EX TOTO ORBE MATHEMATICI OB 
  SCRIPTA INSIGNIA TRIBVERE
  Fields Medal(John Charles 
  Fields)
  
  
  Yahoo! 
  Mail - O melhor e-mail do Brasil. Abra 
  sua conta agora!


Re: [obm-l] Extensoes de Corpos

2004-03-29 Por tôpico Cláudio \(Prática\)
Oi, Duda:

Obrigado pela resposta. Ainda não sei nada sobre a teoria de Galois mas vou
dar uma pesquisada.

[]s,
Claudio.

- Original Message -
From: Eduardo Casagrande Stabel [EMAIL PROTECTED]
To: [EMAIL PROTECTED]
Sent: Monday, March 29, 2004 9:33 AM
Subject: Re: [obm-l] Extensoes de Corpos


 Oi Cláudio.

 Eu não tenho lido as mensagens da lista, e li esta sem querer.

 Se a extensão E:F é normal e separável, além de finita, existe um teorema
 (teorema da correspondência de Galois) que afirma que existe uma bijeção
 entre os corpos intermediários da extensão e o grupo de F-automorfismos de
 E, que é um grupo finito. Existe uma relação bem simples entre a dimensão
da
 extensão e o tamanho do subgrupo. Aí você procura, ao invés de corpos
 intermediários, os subgrupos de determinada ordem.

 Eu acho que, em geral, a resposta à sua pergunta é difícil.

 Abraço,
 Duda.

 - Original Message -
 From: Claudio Buffara [EMAIL PROTECTED]
 To: Lista OBM [EMAIL PROTECTED]
 Sent: Sunday, March 28, 2004 7:20 PM
 Subject: [obm-l] Extensoes de Corpos


  Oi, pessoal:
 
  Com relacao a minha mensagem anterior, minha duvida eh mais geral:
 
  Sejam um corpo F, de caracteristica 0, e uma extensao E tal que [E:F] =
n.
  Se m divide n, quais as condicoes para que exista um corpo K tal que:
  F = K = E, e [K:F] = m?
 
  []s,
  Claudio.
 
 
 
 
=
  Instruções para entrar na lista, sair da lista e usar a lista em
  http://www.mat.puc-rio.br/~nicolau/olimp/obm-l.html
 
=
 
 

 =
 Instruções para entrar na lista, sair da lista e usar a lista em
 http://www.mat.puc-rio.br/~nicolau/olimp/obm-l.html
 =

=
Instruções para entrar na lista, sair da lista e usar a lista em
http://www.mat.puc-rio.br/~nicolau/olimp/obm-l.html
=


Re: [obm-l] Digitos de 1000!

2004-03-22 Por tôpico Cláudio \(Prática\)
Excelente! Exatamente a solução que eu achei.

Agora, vamos ver se você é bom mesmo: faça o terceiro do mesmo jeito. :-)

[]s,
Claudio.

- Original Message -
From: Qwert Smith [EMAIL PROTECTED]
To: [EMAIL PROTECTED]
Sent: Monday, March 22, 2004 2:04 PM
Subject: RE: [obm-l] Digitos de 1000!


 Como qualquer um ve sem nenhuma dificuldade 1000! =
 40238726007709377354370243392300398571937486421071463254379991042993851239
 86290205920442084869694048004799886101971960586316668729948085589013238296
 69944590997424504087073759918823627727188732519779505950995276120874975462
 49704360141827809464649629105639388743788648733711918104582578364784997701
 24766328898359557354325131853239584630755574091142624174743493475534286465
 76611667797396668820291207379143853719588249808126867838374559731746136085
 37953452422158659320192809087829730843139284440328123155861103697680135730
 42161687476096758713483120254785893207671691324484262361314125087802080002
 61683151027341827977704784635868170164365024153691398281264810213092761244
 89635992870511496497541990934222156683257208082133318611681155361583654698
 40467089756029009505376164758477284218896796462449451607653534081989013854
 4248798495995331910172336602139450399736280750137837615307127761926849
 03435262520001588853514733161170210396817592151090778801939317811419454525
 72238655414610628921879602238389714760885062768629671466746975629112340824
 39208160153780889893964518263243671616762179168909779911903754031274622289
 9880051951428201218736174599264295658174662830295557029902432415318161
 72104658320367869061172601587835207515162842255402651704833042261439742869
 33061690897968482590125458327168226458066526769958652682272807075781391858
 17888965220816434834482599326604336766017699961283186078838615027946595513
 11565520360939881806121385586003014356945272242063446317974605946825731037
 90084024432438465657245014402821885252470935190620929023136493273497565513
 95872055965422874977401141334696271542284586237738753823048386568897646192
 7383814900140767310446640259899490217659043399018860185665264850617997
 02356193897017860040811889729918311021171229845901641921068884387121855646
 12496079872290851929681937238864261483965738229112312502418664935314397013
 74285319266498753372189406942814341185201580141233448280150513996942901534
 83077644569099073152433278288269864602789864321139083506217095002597389863
 55427719674282224875758676575234422020757363056949882508796892816275384886
 33969099598262809561214509948717012445164612603790293091208890869420285106
 40182154399457156805941872748998094254742173582401063677404595741785160829
 23013535808184009699637252423056085590370062427124341690900415369010593398
 38357779394109700277534720
 00
 00
 

 logo o numero de zeros e 249 e ultimo digito nao nulo e 2

 :),
 Auggy


 From: Cláudio \(Prática\) [EMAIL PROTECTED]
 Reply-To: [EMAIL PROTECTED]
 To: [EMAIL PROTECTED]
 Subject: [obm-l] Digitos de 1000!
 Date: Mon, 22 Mar 2004 12:08:54 -0300
 
 HelpOi, pessoal:
 
 Já que o assunto é potências de primos que dividem n!, aqui vai um
 bonitinho:
 
 1) (clássico) Por quantos zeros termina a representação decimal de 1000!
 
 2) (menos conhecido e mais difícil) Qual o último algarismo não nulo na
 representação decimal de 1000!
 
 3) (generalização) Qual o último algarismo não nulo de n!?  Dica não
muito
 útil: é sempre par se n  1.
 
 
 []s,
 Claudio.
 

 _
 Check out MSN PC Safety  Security to help ensure your PC is protected and
 safe. http://specials.msn.com/msn/security.asp

 =
 Instruções para entrar na lista, sair da lista e usar a lista em
 http://www.mat.puc-rio.br/~nicolau/olimp/obm-l.html
 =

=
Instruções para entrar na lista, sair da lista e usar a lista em
http://www.mat.puc-rio.br/~nicolau/olimp/obm-l.html
=


[obm-l] Re: [obm-l] Álgebra linear - Problema interessante

2004-03-19 Por tôpico Cláudio \(Prática\)
O que são cifra de Hill e matriz codificadora?

E não seria NIGHT, com H antes do T?

[]s,
Claudio.

- Original Message -
From: Daniel Silva Braz [EMAIL PROTECTED]
To: [EMAIL PROTECTED]
Sent: Thursday, March 18, 2004 11:33 PM
Subject: [obm-l] Álgebra linear - Problema interessante


 Obtenha a cifra de Hill da mensagem DARK NIGTH para
 cada uma das matrizes codificadoras:

 (a) | 1  3 |
 | 2  1 |

 (b) | 4  3 |
 | 1  2 |


 __

 Yahoo! Mail - O melhor e-mail do Brasil! Abra sua conta agora:
 http://br.yahoo.com/info/mail.html
 =
 Instruções para entrar na lista, sair da lista e usar a lista em
 http://www.mat.puc-rio.br/~nicolau/olimp/obm-l.html
 =

=
Instruções para entrar na lista, sair da lista e usar a lista em
http://www.mat.puc-rio.br/~nicolau/olimp/obm-l.html
=


Re: [obm-l] Ordem nos Reais

2004-03-17 Por tôpico Cláudio \(Prática\)

- Original Message -
From: Nicolau C. Saldanha [EMAIL PROTECTED]
To: [EMAIL PROTECTED]
Sent: Wednesday, March 17, 2004 3:14 PM
Subject: Re: [obm-l] Ordem nos Reais


 On Wed, Mar 17, 2004 at 03:00:15PM -0300, Cláudio (Prática) wrote:
  Mas o que acontece se a ordem for diferente?
 
  Por exemplo, suponha que particionamos os reais (R) em racionais (Q) e
  irracionais (R - Q) e definimos uma ordem (#) tal que:
  1) se x, y pertencem a Q ou x, y pertencem a R - Q, então:
  x # y == x  y (ordem usual)
  2) se x pertence a R - Q e y pertence a Q, então x # y.
 
  Ou seja, cada irracional é menor do que cada racional e dois irracionais
ou
  dois racionais são comparados da forma usual.
 
  Agora considere o conjunto A = { -raiz(2)/n | n é inteiro positivo}.
Cada
  elemento de A é irracional. Logo, A é limitado superiormente (por
exemplo,
  por cada racional).  Pergunta: Qual o supremo de A?

 Não tem supremo, claro.

 Mas o que você fez foi um pouco violento demais. Você definiu uma ordem
 que não respeita as operações + e *: dessa forma, a única coisa que sobra
 é a cardinalidade e você pode botar um monte de ordens completamente
 diferentes nos reais. Você pode, por exemplo, fazer com que R fique
 bem ordenado (todo subconjunto não vazio tem mínimo).

 No caso dos reais, a única relação de ordem que faz de R um corpo ordenado
 é a usual.
Ou seja, com qualquer outra ordem, você não consegue obter um conjunto P
fechado em relação a + e *?
É fácil demonstrar isso?

 Isto também acontece para o conjunto dos reais algébricos mas
 não acontece para, por exemplo Q[sqrt(2)]. Neste outro corpo existe uma
outra
 ordem (além da definida pela inclusão em R) que também satisfaz os axiomas
 de corpo ordenado, e acho que vocês não terão dificuldade em encontrá-la.
Isso é interessante.

Uma ordem é definida com base no conjunto P = {a + b*raiz(2) | a + b*raiz(2)
é real positivo} e é meio óbvio de ver que se x, y pertencem a P, então x+y
e x*y pertencem a P.

A outra é definida com base em P* = {a + b*raiz(2) | a - b*raiz(2) pertence
a P}

Pra simplificar, vamos fazer w = raiz(2).

a + bw = 0 == a = b = 0 == a - bw = 0.

Suponha que a + bw  0 mas não pertence a P*.
Então a - bw não pertence a P ==
-(a - bw) = -a - (-b)w pertence a P ==
-(a + bw) = -a + (-b)w pertence a P*.
Logo, se x pertence a Q[raiz(2)], então x = 0, ou x pertence a P* ou -x
pertence a P* e estas três alternativas são mutuamente exclusivas.

Sejam a + bw e c + dw pertencentes a P*.
Então, a - bw e c - dw pertencem a P.
Logo:
(a - bw) + (c - dw) = (a + c) - (b + d)w pertence a P ==
(a + c) + (b + d)w = (a + bw) + (c + dw) pertence a P*
Também:
(a - bw)*(c - dw) = (ac + bd) - (ad + bc)w pertence a P ==
(ac + bd) + (ad + bc)w = (a + bw)(c + dw) pertence a P*.
Ou seja, a ordem definida com base em P* também preserva as operações + e *.

Obrigado pela explicação.

[]s,
Claudio.

=
Instruções para entrar na lista, sair da lista e usar a lista em
http://www.mat.puc-rio.br/~nicolau/olimp/obm-l.html
=


Re: [obm-l] Ideal Maximal

2004-03-01 Por tôpico Cláudio \(Prática\)
Obrigado, Nicolau!

Eu estava assumindo implicita e erroneamente que todo ideal de Z_4[x] eh
principal, mas checando meus alfarrábios, vejo que A[x] só será um PID se A
for um corpo. Aliás, o mesmo exemplo com Z ao invés de Z_4 mostra que mesmo
que A seja um domínio de integridade (mas não um corpo), A[x] não será
necessariamente um PID (apesar de ser um domínio de integridade).

Aos poucos estas idéias vão se assentando...

Um abraço,
Claudio.

- Original Message -
From: Nicolau C. Saldanha [EMAIL PROTECTED]
To: [EMAIL PROTECTED]
Sent: Monday, March 01, 2004 1:11 PM
Subject: Re: [obm-l] Ideal Maximal


 On Mon, Mar 01, 2004 at 11:09:44AM -0300, Claudio Buffara wrote:
  Sejam:
  Z_4 = anel dos inteiros mod 4
  e
  Z_4[x] = anel dos polinomios com coeficientes em Z_4.
  O ideal x^2 + 1 de Z_4[x] eh maximal?
 
  Eu diria que sim, dado que x^2 + 1 eh irredutival sobre Z_4, mas nesse
caso,
  Z_4[x]/x^2 + 1 seria um corpo, o que nao eh verdade, pois contem o
  elemento 2 + x^2 + 1, o qual eh um divisor de zero.
 
  Onde estah o meu erro?

 O ideal J que você descreveu não é maximal, ele está contido no ideal
 J1 = x^2 + 1, 2. Aliás J1 também não é maximal, ele está contido
 em J2 = x+1, 2 (pois x^2+1 = (x+1)^2 - 2x); J2 sim é maximal, e o
 quociente é Z/(2).

 O fato do polinômio p em A[x] ser irredutível não prova que o ideal p
 é maximal se A for um anel, isto só dá certo se A for um corpo.

 []s, N.
 =
 Instruções para entrar na lista, sair da lista e usar a lista em
 http://www.mat.puc-rio.br/~nicolau/olimp/obm-l.html
 =

=
Instruções para entrar na lista, sair da lista e usar a lista em
http://www.mat.puc-rio.br/~nicolau/olimp/obm-l.html
=


Re: [obm-l] Forma canonica...

2004-02-18 Por tôpico Cláudio \(Prática\)
Maximizar x*y dado que x + y = 8.

Escreva y = 8 - x, de modo que você quer maximizar f(x) = x*(8 - x) = 8x -
x^2.

A idéia agora é completar o quadrado, ou seja, reescrever f(x) como sendo:
f(x) = -16 + 8x - x^2 + 16 = 16 - (4 - x)^2

Agora fica fácil (espero) ver que f(x) será máximo e igual a 16 quando x =
4, já que um quadrado é sempre não negativo.

Um abraço,
Claudio.

- Original Message -
From: Rick [EMAIL PROTECTED]
To: [EMAIL PROTECTED]
Sent: Sunday, February 18, 2001 2:03 PM
Subject: [obm-l] Forma canonica...


 Antes de mais nada, gostaria de perguntar ao prof. Nicolau (ou a qualquer
 outro membro que possa responder), se questoes de fisica (geralmente a
parte
 mais matematica) sao bem vindas na lista. Se nao, alguem poderia me
indicar
 uma outra lista ou site em que possa tirar as minhas duvidas sobre fisica.
 Obrigado.

 Deixando de enrolacao [ :) ], vamos ao assunto desse topico.

   Aproveitando o ensejo, estou com mais uma duvida
   neste exercicio:
   1)Dentre todos os numeros reais de soma 8 determine
   aqueles cujo produto é
   maximo.
   O livro respondeu isso atribuindo o seguinte
   sistema:
   x+z=8  (I)y=x.z (II)
  De fato, precisamos ter x+z =8. Hah sem duvida uma
  infinidade de reais x e z que satiafazem a esta
  igualdade. A cada par (x,z), temos associado um
  produto x.z, que o seu livro chamou de y.
 Realmente, existem uma infinidade de numeros x e z que a satisfazem.
Porem,
 minha duvida era outra. Desta maneira temos dois termos (x e z), mas seria
 possivel obter um produto maior utilizando mais termos cuja soma totalize
em
 8. Por exemplo, temos os numeros reais -6, -2, +16. A soma deles eh 8. Ja
o
 produto eh 192 (imensamente maior que o 16, valor atribuido como produto
 maximo). Portanto, ja vemos que existem outros produtos superiores quando
 usamos mais termos, por isso questionei o uso do livro de apenas dois
 termos.


  Tecnicamente, temos uma funcao de R^2 em R dada por
  f(x,z) = y = xz. Queremos calcular seu minimo quando x
  e z satifazem aa particularidade de que x+z = 8, isto
  eh, queremos calcular o minimo de f quando a mesma
  esta restrita ao subconjunto de R^2 dado por {(x,z) :
  x+z =8}, o qual eh uma reta em R^2.
 Hum... nao entendi muito bem o conjunto R^2. O que isso quer dizer, que os
 valores do dominio dessa funcao estarao todos elevados ao quadrado?


 Neste caso, fica bem facil transformar este problema bidimensional em
  um unidimensional. De x+z =8, temos que z = 8-x e que
  y = xz = x(8-x). Vemos agora que y pode ser visto como
  uma funcao so de x, que no caso eh um trinomio do
  segundo grau. Jah temos a forma fatorada do mesmo e
  vemos que suas raizes sao 0 e 8. Vemos tambem que o
  coeficiente o 2o grau deste trinomio eh negativo, o
  que nos diz que ele tem um maximo relativo que, no
  caso de trinomios do segundo grau, eh maximo global.
 Meus conhecimentos matematicos ainda desconhecem a existencia de outros
 maximos, alem daquele que eh representado pela ordenada do vertice da
 parabola. ..Rs..

  Sabemos que o maximo de um trinomio com tais condicoes
  ocorre para x* = semi soma das raizes. Logo, x* =
  (0+8)/2 = 4. Isto nos conduz a z* = 8-4 =4 e a y* =
  x*.y* = 16. Esta conclusao eh geral, isto eh, se x+z
  =S, S0, entao o maior valor do produto x.z ocorre
  para x* = z* = S/2. Este problema eh um classico e eh
  muitas vezes enunciado da seguinte forma: dentre todos
  os retangulos de mesmo perimetro, qual o de maior
  area? A resposta, pelo que vimos, eh o quadrado.
 Bastante interessante esta conclusao, valeu mesmo!.

  Bom, para provarmos que a solucao a que chegamos e de
  fato a maxima, usamos normalmente o conceito de
  deriaada, do Calculo Diferencial. Você já  chegou a
  estudar este assunto?
 Ainda nao, o unico contato que tive com derivadas foi em um livro de
fisica.
 Para explicar a velocidade instantanea, ele utilizou o conceito de limite
e
 em seguida abordou a expressao an.t^(n-1), isto em uma funcao do tipo
 f(t)=a.t^n, onde a variavel era o tempo. Fora a explicacao superficial, e
o
 entendimento que tenho sobre limite, eu nao tenho a minima ideia do que
 seja. Uma duvida: esse eh um assunto geralmete estudado na universidade ou
 no ensino medio? Aqui na Bahia,  nao cheguei a ve-lo. (eu jah conclui)

  Sugestao: Outro problema bonito - dentre todos os
  retangulos de mesma area, qual o de menor perimetro?
  Dentre este retangulos, existe algum de maior
  perimetro?
 Esse eu ainda tow pensando, assim que responder eu mando pra ca... :D

 =
 Instruções para entrar na lista, sair da lista e usar a lista em
 http://www.mat.puc-rio.br/~nicolau/olimp/obm-l.html
 =

=
Instruções para entrar na lista, sair da lista e usar a lista em
http://www.mat.puc-rio.br/~nicolau/olimp/obm-l.html

Re: [obm-l] Um problema de logica muito confuso

2004-02-16 Por tôpico Cláudio \(Prática\)
Não tenho nem 1/10 do conhecimento matemático do Nicolau, mas de uma coisa
estou certo: se esse professor realmente apresentou esta questão aos alunos
e deu esta resposta com esta justificativa, então ele merece ser demitido
por justa causa.

Para cada um dos itens, existe uma pergunta cuja resposta é aquele item e
nenhum outro (supondo que I = R - Q).

Por exemplo:
Qual o conjunto numérico (dentro os 5 citados) que contêm 1/2 mas não contém
raiz(2)? Resposta (a)

Um abraço,
Claudio.

- Original Message -
From: [EMAIL PROTECTED]
To: [EMAIL PROTECTED]
Sent: Monday, February 16, 2004 3:10 PM
Subject: [obm-l] Um problema de logica muito confuso


 Caros amigos da lista, por favor, poderiam me ajudar nesta duvida cruel,
 referente a seguinte questão :

 Em um teste há cinco opções, porém somente uma é correta, sendo elas :

 a)Q   b)I   c)Z   d)R   e)C

 Qual destas opções é correta ?

 A resposta do professor e dos alunos foi a e), porque, se a resposta
certa
 fosse por exemplo a opção c) então a opção a) tambem seria correta já
 que Q contém Z, gerando duas opções corretas, porém o enunciado da questão
 diz que há apenas uma opção correta, então ABSURDO!!! Usando raciocinio
 analogo a todas as opções chegaram a opção e) como correta.

 Porém eu não concordei... disse que a resposta da pergunta varia de acordo
 com a pergunta do teste em questão(vamos admitir que todo teste tem que
ter
 pergunta). Se a pergunta do teste fosse : Qual o conjunto que é formado
por
 todos os numeros que podem ser escritos da forma(+ ou -)x, tal que x
 pertence aos naturais ? A resposta correta seria c).

 Acho que todos cairam numa espécie de pegadinha, inclusive o professor que
 por mais que eu tentasse lhe convencer da minha opinião ele não aceitava.
Ou
 simplismente eu estou errado.

 Seria imensuravel a ajuda que voces me dariam postando sua opinião na
lista,
 gostaria de pedir especialmente a opinião do Nicolau Saldanha, pois se ele
 concordar comigo talvez meu professor me dê mais atenção, se ele não
 concordar será uma opinião também muito válida para mim.

 Desde já muito agradecido,

 Adenilson Junior
 Fortaleza-CE

 _
 Voce quer um iGMail protegido contra vírus e spams?
 Clique aqui: http://www.igmailseguro.ig.com.br
 Ofertas imperdíveis! Link: http://www.americanas.com.br/ig/

 =
 Instruções para entrar na lista, sair da lista e usar a lista em
 http://www.mat.puc-rio.br/~nicolau/olimp/obm-l.html
 =

=
Instruções para entrar na lista, sair da lista e usar a lista em
http://www.mat.puc-rio.br/~nicolau/olimp/obm-l.html
=


Re: [obm-l] Numeros algebricos e transcendentes

2004-02-12 Por tôpico Cláudio \(Prática\)

- Original Message -
From: Artur Costa Steiner [EMAIL PROTECTED]
To: [EMAIL PROTECTED]
Sent: Thursday, February 12, 2004 10:20 AM
Subject: [obm-l] Numeros algebricos e transcendentes


 Alguem poderia indicar algum material ou algum site sobre numeros
algebricos
 e transcendentes?
 Especificamente, alguem tem uma demonstracao de que a soma de um
 transcendente com um  algebrico eh trancendente e o produto de um
 transcendente por um algebrico nao nulo eh transcendente?
 Obrigado
 Artur

Oi, Artur:

Se a eh algebrico não nulo e t eh transcendente, então a+t e a*t são ambos
transcendentes, pois se fossem algébricos, então:
t = (a+t)-a = (a*t)*(1/a) também seria algébrico (pois o conjuunto dos
algébricos é um corpo) == contradição.

De qualquer forma, existem notas de aula on-line sobre o assunto aqui:
http://www.math.sc.edu/~filaseta/gradcourses/Math785/main785.html

E o melhor (do seu ponto de vista) é que o único pré-requisito é análise
(além de alguns fatos básicos de álgebra e teoria dos números)!

Um abraço,
Claudio.

=
Instruções para entrar na lista, sair da lista e usar a lista em
http://www.mat.puc-rio.br/~nicolau/olimp/obm-l.html
=


Re: [obm-l] Ajuda

2004-02-12 Por tôpico Cláudio \(Prática\)



Resolva a inequação 
abaixo

9^x - 6^x - 4^x  0

eu não consigo desenvolver essa questão...

tentei da seguinte maneira.

3^(2x) - 2^(x). 3^(x) - 2^(2x)  0

substituindo
3^x por y
2^x por x == má escolha de variável. Com tanta letra dando sopa você 
foi logo escolher a mesma? Pode dar confusão no final. Vou mudar pra 
z.
terei
y^2 - zy - z^2  0

Isso é igual a:
y^2 - zy + z^2/4 - z^2/4 - z^2  0 ==
(y - z/2)^2  5z^2/4 ==
| y - z/2 |  |z|*raiz(5)/2 = z*raiz(5)/2, poisz = 2^x  0 
==
y - z/2  z*raiz(5)/2 ou y - z/2  - 
z*raiz(5)/2 ==
y  z*(1+raiz(5))/2 ou y  
z*(1-raiz(5))/2.

Mas y = 3^x  0. Logo não pode ser y  z*(1-raiz(5))/2  0.
Assim, só pode ser:
y  z*(1+raiz(5))/2 ==
3^x  2^x*(1+raiz(5))/2 ==
(3/2)^x  (1+raiz(5))/2 ==

x  log((1+raiz(5)/2)/log(3/2).

Um abraço,
Claudio.
 




Re: [obm-l] Numeros de Lucas

2004-02-12 Por tôpico Cláudio \(Prática\)



Sua informação é quase inútil se você não 
especificar pelo menos o número da Eureka (e de preferência a página ou o nome 
da seção/artigo em que você viu o problema)

  - Original Message - 
  From: 
  Johann Peter Gustav Lejeune 
  Dirichlet 
  To: [EMAIL PROTECTED] 
  Sent: Thursday, February 12, 2004 1:33 
  PM
  Subject: Re: [obm-l] Numeros de 
  Lucas
  
  Se nao me engano ja vi isto na Eureka!Claudio Buffara 
  [EMAIL PROTECTED] 
  wrote: 
  Oi, 
pessoal:Considere a sequencia (L(n)) dada por:L(0) = 2L(1) = 
1L(n) = L(n-1) + L(n-2) para n = 2.Prove que existe um 
inteiro positivo m tal que a soma de quaisquer m termosconsecutivos 
dessa sequencia eh divisivel por 2004.Prove tambem que se mudarmos 
as condicoes iniciais para L(0) = 0 e L(1) = 1(de forma que a sequencia 
passa ser a de Fibonacci), entao para qualquerinteiro positivo m vai 
existir um termo da sequencia divisivel por m.Um 
abraco,Claudio.=Instruções 
para entrar na lista, sair da lista e usar a lista 
emhttp://www.mat.puc-rio.br/~nicolau/olimp/obm-l.html=
  
  
  Yahoo! 
  GeoCities: 15MB de espaço grátis para criar seu web 
site!


Re: [obm-l] Problema para Artur

2004-02-12 Por tôpico Cláudio \(Prática\)



Oi, Bruno:

Existe um teorema (provado por Gelfond e Schneider) 
quediz que se a e b são algébricos, com a  0, a  1 e b 
irracional, então a^b é transcendente. Uma demonstração disso está contida nas 
notas de aula que eu recomendei pro Artur numa mensagem anterior.

Por outro lado, não conheço nenhum resultado geral 
sobre a natureza de a^b quando a e b são transcendentes (o que não quer dizer 
absolutamente nada - um tal resultado pode muito bem existir! Você conhece 
algum?).

Um argumento de cardinalidade mostra que x^x deve 
assumir pelo menos um valor transcendente para algum x transcendente. 

Considere a partição do intervalo [1,+inf) = A U T, 
onde A = conjunto dos algébricos no intervalo e T = conjunto dos transcendentes 
no intervalo.
Seja F: [1,+inf) -- [1,+inf) dada por F(x) = 
x^x. F é claramente uma bijeção.
Como A é enumerável, F(A) é 
enumerável.
Se F(T) só contém números algébricos, então 
F(T)também é enumerável.
Logo, (1,+inf] = F((1,+inf]) = F(A U T) = F(A) U 
F(T) é enumerável == 
contradição ==
F(T) deve conter algum transcendente (de fato, uma 
infinidade não-enumerável deles).

A questão que permanece é: existe algum 
transcendente x tal que x^x é algébrico?

O caso x = raiz(2) não parece ajudar muito, pois 
raiz(2) é algébrico. 
De qualquer forma, o teorema de Gelfond-Schneider 
com a = b = raiz(2) mostra que raiz(2)^raiz(2) é transcendente.

Um abraço,
Claudio.

  - Original Message - 
  From: 
  Bruno 
  Lima 
  To: OBM lISTA 
  Sent: Thursday, February 12, 2004 1:49 
  PM
  Subject: [obm-l] Problema para 
Artur
  
  Agora que vc esta pensando sobre numeros algebricos e transcendentes 
  uma questao interessante é a seguinte: seja x transcendente, entao x 
  elevado a x é algebrico ou transcendente??
  pense primeiro no caso x= raiz qudrada de 
2.


Re: [obm-l] Numero de Napier

2004-02-10 Por tôpico Cláudio \(Prática\)
Eu posso estar enganado mas acho que menos de 1% dos matemáticos se referem
ao e como número de Napier ou número de Euler. Este último, inclusive,
empresta seu nome a uma outra constante - justamente a constante de Euler -
igual a lim(n-inf) (1 + 1/2 + ... + 1/n - ln(n)) e chamar o e de número
de Euler pode dar confusão.

Para calcular o valor numérico de e sem usar série de Taylor, acho que a
melhor forma é usar frações contínuas:
e = [2;1,2,1,1,4,1,1,6,1,1,8,...]

Para uma demonstração (arquivo pdf), entre em:
http://research.microsoft.com/~cohn/publications.html
e vá até o final da página.

Um abraço,
Claudio.

- Original Message -
From: Henrique Patrício Sant'Anna Branco [EMAIL PROTECTED]
To: [EMAIL PROTECTED]
Sent: Tuesday, February 10, 2004 1:12 PM
Subject: Re: [obm-l] Numero de Napier


  numero de Napier é o mesmo que numero de Euler? Defina numero de Napier

 Sim, é o mesmo número irracional e ~ 2.71828...

 Henrique.

 =
 Instruções para entrar na lista, sair da lista e usar a lista em
 http://www.mat.puc-rio.br/~nicolau/olimp/obm-l.html
 =

=
Instruções para entrar na lista, sair da lista e usar a lista em
http://www.mat.puc-rio.br/~nicolau/olimp/obm-l.html
=


Re: [obm-l] Problema Interessante

2004-02-10 Por tôpico Cláudio \(Prática\)
Oi, Artur:

Tem também um teorema que diz que se x e cos(pi*x) são ambos racionais,
então x = k/2 ou x = k/3 para algum k inteiro, mas não se aplica a este
problema pois (raiz(5)-1)/2 é irracional.

Mesmo não sendo aplicável, acho que é um resultado interessante por si mesmo
e cuja demonstração não é difícil.
A idéia é mostrar, por indução, que cos(n*pi*x) pode ser expresso como um
polinômio:
p(t) = a_0 + a_1*t + ... + a_n*t^n,
de grau n, em cos(pi*x) tal que:
a_n = 2^(n-1)
e
para 2 = k = n, 2^(k-1) divide a_k.

Isso implica que 2*cos(pi*x) é raiz de um polinômio mônico de grau n e
coeficientes inteiros. Logo, se 2*cos(pi*x) é racional, então 2*cos(pi*x) só
pode ser inteiro (pelo bom e velho teorema das raízes racionais) ==
cos(pi*x) = -1, -1/2, 0, 1/2 ou 1 == x = k/2 ou x = k/3 com k inteiro.

Um corolário que eu acho interessante é que se as medidas dos lados e dos
ângulos (em graus) de um triângulo são todas racionais, então esse triângulo
é equilátero.

*

De qualquer forma talvez dê pra aproveitar alguma idéia do teorema acima pra
resolver o problema do Márcio.

Um abraço,
Claudio.

- Original Message -
From: Artur Costa Steiner [EMAIL PROTECTED]
To: [EMAIL PROTECTED]
Sent: Tuesday, February 10, 2004 12:50 PM
Subject: Re: [obm-l] Problema Interessante


 O Marcio Cohen propos, no ultimo domingo, um problema interessante, provar
 que x = arccos[((sqrt(5)-1)/2]/pi eh irracional. Eu achei que isto poderia
 ter solucao por fracoes continuas ou com base na divisao aurea. Mas por
aih
 nao cheguei a nada.

 Depois eu notei que (sqrt(5)-1))/2 eh uma das dua raizes da equacao do
 segundo grau, de coeficientes inteiros, 2x^2 + x -1 =0, de modo que
 (sqrt(5)-1))/2 eh algebrico. Observamos ainda que cos(pi*x)
=(sqrt(5)-1))/2,
 o que implica em que a parte real de e^(pi*x) seja Re[^(pi*x)] =
 sqrt(5)-1))/2. Neste ponto eu me lembrei que parece que hah um teorema
(mao
 estou abolutamente certo) o qual diz que, com excecao de -1, 0 e 1, as
 partes reais das raizes inteiras da unidade sao transcendentes. Se alguem
se
 lembrar deste teorema, caso efetivamente exista, e puder apresentar ou
mesmo
 rascunhar uma prova, eu gostaria.

 Bom, assumindo-se que o citado teorema efetivamente exista, concluimos que
 Re[e^(pi*x)] ek algebrico e que, desta forma, e^(pi*x*i) nao eh raiz da
 unidade. Se x for racional, entao existem inteiros p0 e q0 tais que x
 =p/q. Logo pi*x*i = pi* p/q *i e e^(pi*x*i)= cos(p*pi/q) + i *
sen(p*pi/q).
 Logo, [e^(pi*x*i]*q = cos(p*pi) + i * sen(p*pi). Mas eh sempre possivel
 escolhermos p/q = x de modo que p seja par e que, consequentemente,
 cos(p*pi)  = 1 e sen(p*pi) =0. Isto nos mostra que existe q inteiro  tal
que
 [e^(pi*x*i]*q  =1 . A conclusao eh que se x eh racional entao e^(pi*x*i)
eh
 raiz da unidade para algum inteiro p.
 dado que, no caso prooposto, e^(pi*x*i) nao eh raiz da unidade, segue-se
que
 x eh iracional.
 Supondo-se, eh claro, que o teorema que citei existe...Vou tentar
 demonstra-lo, se possivel.
 Artur


 
 OPEN Internet
 @ Primeiro provedor do DF com anti-vírus no servidor de e-mails @


 =
 Instruções para entrar na lista, sair da lista e usar a lista em
 http://www.mat.puc-rio.br/~nicolau/olimp/obm-l.html
 =

=
Instruções para entrar na lista, sair da lista e usar a lista em
http://www.mat.puc-rio.br/~nicolau/olimp/obm-l.html
=


Re: [obm-l] Problema

2003-11-26 Por tôpico Cláudio \(Prática\)



Calcule o numero de partições do conjunto {1,2,3,...,n^2} em n 
conjuntos de n elementos cada, contando de duas maneiras o número de permutações 
dos elementos do conjunto.

Um abraço,
Claudio.




- Original Message - 

  From: 
  Benedito 
  To: [EMAIL PROTECTED] 
  Sent: Wednesday, November 26, 2003 1:16 
  PM
  Subject: [obm-l] Problema
  
  

  
Problema
Use um argumento combinatóriopara mostrar que o número 
(n^2)! é divisível por
(n!)^(n+1).

Benedito


  

  
  


 IncrediMail - O mundo do correio eletrônico 
  finalmente desenvolveu-se - Clique 
  aqui
IMSTP.gif

Re: [obm-l] Probabilidade 1/3

2003-11-19 Por tôpico Cláudio \(Prática\)
 Mas mais seriamente não acho óbvio como calcular
 P(cara), P(coroa) ou P(em pé) para um cilindro.

Abstraindo de todas as complicações físicas e de engenharia (atrito, choque
elástico vs inelástico, imperfeições do cilindro e da superfície onde ele é
jogado, etc.), me parece que um cilindro homogêneo com h = d*raiz(3) (e não
h = d/raiz(3) como eu havia escrito antes) tem as três probabilidades
iguais.

Pra mim é claro que P(em pé) - 0 (1) quando h - 0 (+infinito). Logo,
supondo que P(em pé) é uma função contínua de h (hipótese que não me parece
tão absurda), deve haver h tal que P(em pé) = 1/3.

Para um cilindro de altura = h e diâmetro = d, eu diria que P(em pé) = 1 -
(2/Pi)*arctg(h/d).  Eu simplesmente determinei o ângulo de contato entre o
cilindro e a superfície para o qual o reta unindo o centro de massa ao ponto
de contato é perpendicular à superfície.

Um abraço,
Claudio.


=
Instruções para entrar na lista, sair da lista e usar a lista em
http://www.mat.puc-rio.br/~nicolau/olimp/obm-l.html
=


Re: [obm-l] Geometria

2003-11-11 Por tôpico Cláudio \(Prática\)
Title: Re: [obm-l] Geometria



Seja P em DE tal que IP seja paralelo a 
AE.
Então:AE = IP = a.

Seja DIE = x
IA = AE*tg(IEA) = a*tg(50)
DE = AE*tg(DAE) = a*tg(60)
DP = IP*tg(DIP) = a*tg(x - 50)

Mas DP = DE - PE = DE - IA ==
a*tg(x - 50) = a*tg(60) - a*tg(50)

Cancelando a ...

Um abraço,
Claudio.


  - Original Message - 
  From: 
  Bruno Souza 
  
  To: [EMAIL PROTECTED] 
  Sent: Tuesday, November 11, 2003 2:03 
  PM
  Subject: Re: [obm-l] Geometria
  
  Cláudio,
  Esse ângulo achado pode ser 
coerente.
  Como chegou em tal equação??
  Obrigado
  "Bruno S
  
- Original Message - 
From: 
Claudio Buffara 
To: [EMAIL PROTECTED] 
Sent: Monday, November 10, 2003 10:45 
PM
Subject: Re: [obm-l] Geometria
on 10.11.03 17:43, Bruno Souza at [EMAIL PROTECTED] wrote:
Olá a todos,Há muito tempo 
  tenho esse problema que não consigo resolver.Gostaria de qualquer 
  ajuda ou sugestão.Penso, utilmamente, que esse ângulo 
  não eh determinado, porém não consigo provar.P.S: Esse problema "parece" 
  elementar.Até,BrunoFazendo 
m(EID) = x, eu obtive a equacao:tg(x - 50) = tg(60) - tg(50), o que 
implica que x = 78,38 graus.Serah que eu errei alguma conta? 



Re: [obm-l] Prova do IME

2003-11-05 Por tôpico Cláudio \(Prática\)

- Original Message -
From: Paulo Santa Rita [EMAIL PROTECTED]
To: [EMAIL PROTECTED]
Sent: Wednesday, November 05, 2003 2:04 PM
Subject: Re: [obm-l] Prova do IME



 Quem faz a questao 2, com solucao diferente da do GPI ? Para que todos
 possam participar, voces aceitam que uma pessoa so possa fazer uma questao
(
 nao duas ou mais ) ?

Questão:
P(x) = x^3 + ax + b (b  0) tem 3 raízes reais. Prove que a  0.

A solução do GPI usou as relações de Girard.

Aqui vai uma solução alternativa:
Se a = 0, então P(x) tem uma única raiz real, igual a (-b)^(1/3).

Se a  0, então P'(x) = 3x^2 + a  0, para todo x ==
P(x) é estritamente crescente ==
Como lim(x--inf) P(x) = -inf e lim(x - +inf) P(x) = +inf, P(x) tem uma
única raiz real.

Logo, só pode ser a  0.

Um abraço,
Claudio.

=
Instruções para entrar na lista, sair da lista e usar a lista em
http://www.mat.puc-rio.br/~nicolau/olimp/obm-l.html
=


[obm-l] Re: [obm-l] Geometria Analítica

2003-10-28 Por tôpico Cláudio \(Prática\)



Oi, chará:

Não, pois este ponto não é o baricentro (em geral), 
mas sim o circumcentro.

Talvez seja mais fácil calcular o ponto de 
interseccção das mediatrizes de dois dos lados do triângulo que tem estes pontos 
como vértices.

Um abraço,
Cláudio.

  - Original Message - 
  From: 
  Claudio 
  
  To: [EMAIL PROTECTED] 
  Sent: Tuesday, October 28, 2003 9:33 
  AM
  Subject: [obm-l] Geometria 
Analítica
  
  Pessoal nesta questão simples de GA, posso usar o 
  baricentro para calcular o ponto equidistante?
  
  Veja.
  
  O Unico ponto que é equidistante de (0,0) (1,2) e 
  (3,-1) é?
  
  Desde ja 
agradeço.


Re: [obm-l] analise combinatoria

2003-10-27 Por tôpico Cláudio \(Prática\)



Um outro jeito eh deduzir do número total de 
permutações circulares dos algarismos (9!) o número destas em que o 0 e o 5 
ficam diametralmente opostos:

Uma vez colocado o 0, há 1 maneira de se colocar o 
5. Em seguida, permutam-se os 8 algarismos restantes. Total = 8!.

Logo, o número desejado é 9! - 8! = 8!*(9-1) = 
8!*8.


  - Original Message - 
  From: 
  Domingos Jr. 
  
  To: [EMAIL PROTECTED] 
  Sent: Monday, October 27, 2003 10:14 
  AM
  Subject: Re: [obm-l] analise 
  combinatoria
  
  acho que está certo.
  
  fixe 0 numa posição, então o5 pode possuir 
  qualquer posição, exceto a diametralmente oposta,havendo 8 posições 
  possíveis, depois os 8 demais números podem ser permutados 
  livremente.
  não estamos considerando rotações das numerações 
  (o que eu acho correto para esse problema, já que ele o polígono é regular e 
  os vértices não possuem nomes).
  
- Original Message - 
From: 
Silvio Borges 

To: [EMAIL PROTECTED] 
Sent: Monday, October 27, 2003 8:42 
AM
Subject: [obm-l] analise 
combinatoria

Gostaria que me ajudassem nesta questao, eu fiz 
mas tenho duvidas 
quanto a resposta encontrada.
Muito obrigado

Silvio.

A questao e a seguinte :

De quantas maneiras podemos dispor os numeros 
de 0 a 9, nos 
vertices de umdecagono regular, de modo 
que o 0 e o 5 nao fiquem
diametralmente opostos ?


eu encontrei 8 * 8!





Re: [obm-l] SOMA(n=1) (1/n)*((2+sen(n))/3)^n

2003-10-23 Por tôpico Cláudio \(Prática\)

 S = somatorio(1 ate +INF) de i^[ - r(i) ] , r( i )  1, converge ? Para
mim,
 e evidente que sim.

Oi, Paulo:

Infelizmente isso não é verdade.
Por exemplo, para cada n = 3, tome r(n) = 1 + ln(ln(n))/ln(n)  1.
Isso resulta em n^r(n) = n*ln(n) ==
SOMA(n=3) n^(-r(n)) =  SOMA(n =3) 1/(n*ln(n)), que diverge, pelo teste da
integral.

*

O problema do Duda parece ser bem mais complicado.
Por exemplo, um bom começo seria determinar se a sequência x(n) = sen(n)^n é
convergente ou não.

Um abraço,
Claudio.


=
Instruções para entrar na lista, sair da lista e usar a lista em
http://www.mat.puc-rio.br/~nicolau/olimp/obm-l.html
=


Re: [obm-l] Problemas de Divisibilidade

2003-10-17 Por tôpico Cláudio \(Prática\)



Não é verdade que se 2^n divide x^2 então 2^n 
divide x.

  - Original Message - 
  From: 
  Daniel Melo 
  Wanzeller 
  To: [EMAIL PROTECTED] 
  Sent: Thursday, October 16, 2003 10:28 
  AM
  Subject: Re: [obm-l] Problemas de 
  Divisibilidade
  
  A da raiz fiz o seguinte:
  
   (3 + raiz)^n + (3 - raiz) ^n = 
  x
   Elevando ao quadrado dos dois 
  lados, tem-se:
   (3 + raiz)^2n + ( 3 - raiz)^2n 
  + 2[(3+raiz)(3 - raiz)]^n = x^2
   Desenvolvendo:
   (14+6*raiz)^n + (14 - 
  6*raiz)^n + 2*(2)^2n = x^2
   Colocando o 2^n em evidencia 
  nos dois primeiros termos, tem-se:
   [(7 + 3*raiz)^n]*(2^n) + [(7 - 
  3*raiz)^n]*(2^n)+ 2*2^(2n) = x^2
   Assim, se x^2 é divisivel por 
  2^n x também o é.


Re: [obm-l] Problemas de Divisibilidade

2003-10-13 Por tôpico Cláudio \(Prática\)

- Original Message -
From: Carlos Maçaranduba [EMAIL PROTECTED]
To: [EMAIL PROTECTED]
Sent: Sunday, October 12, 2003 6:32 PM
Subject: [obm-l] Problemas de Divisibilidade


 II-Se n 1 e impar = 1^n + 2^n + ... (n -1)^n é
 divisivel por n.

Usando congruências mod n, teremos:
1 == -(n-1)
2 == -(n-2)
...
(n-1)/2 == -(n+1)/2

Elevando essas (n-1)/2 congruências ao expoente n (que é ímpar), obteremos:
1^n == -(n-1)^n
2^n == -(n-2)^n
...
((n-1)/2)^n == -((n+1)/2)^n

Somando tudo, ficaremos com:
1^n + 2^n + ... + ((n-1)/2)^n == -(n-1)^n - (n-2)^n - ... - ((n+1)/2)^n

Ou seja:
1^n + 2^n + ... + (n-2)^n + (n-1)^n == 0 (mod n)

O que quer dizer que:
n divide 1^n + 2^n + ... + (n-1)^n.

Um abraço,
Claudio.

=
Instruções para entrar na lista, sair da lista e usar a lista em
http://www.mat.puc-rio.br/~nicolau/olimp/obm-l.html
=


Re: [obm-l] Adivinhe o seu bit

2003-10-06 Por tôpico Cláudio \(Prática\)

- Original Message -
From: Nicolau C. Saldanha [EMAIL PROTECTED]
To: [EMAIL PROTECTED]
Sent: Monday, October 06, 2003 4:04 PM
Subject: Re: [obm-l] Adivinhe o seu bit


 On Mon, Oct 06, 2003 at 03:25:46PM -0300, Cláudio (Prática) wrote:
  São dadas n pessoas, cada uma com um bit (0 ou 1) escrito em sua testa
de
  forma aleatória e independente. Cada pessoa pode ver os n-1 bits
escritos nas
  testas das outras pessoas, mas não o seu próprio bit. O seguinte jogo é
  jogado: cada pessoa escolhe ou PASSAR ou CHUTAR O SEU BIT, e isso é
feito
  simultaneamente por todas as n pessoas. Diremos que esse grupo de
pessoas
  VENCEU o jogo se pelo menos uma pessoa decidiu chutar o seu bit e todas
as
  pessoas que chutaram o seu bit acertaram.
 
  Mostre que:
  1) Para todo n = 3 existe uma estratégia E(n) tal que:
  Prob(vencer com E(n))  1/2
 
  2) Para todo n = 1 existe uma estratégia E(n) tal que:
  Prob(vencer com E(n)) -- 1 quando n -- infinito

 Vejamos se eu entendi bem. As pessoas no grupo colaboram (ou todos ganham
 ou todos perdem).  Elas podem combinar uma estratégia com antecedência
 mas uma vez iniciado o jogo elas não podem mais se comunicar (exceto pelas
 jogadas, que são públicas). A estratégia é escolhida antes do sorteio dos
bits.

 É isso?

 Para n = 3 uma estratégia possível é a seguinte.
 Se os bits dos seus dois companheiros forem iguais
 chute que o seu é o oposto do deles. Assim se os três bits
 forem iguais o grupo perde na primeira jogada com três chutes errados;
 isto ocorre com probabilidade 1/4.
 Caso contrário o grupo ganha na primeira jogada, com um único chute
(certo);
 isto ocorre com probabilidade 3/4.

 []s, N.

Oi, Nicolau:

É isso mesmo. Eu devia ter deixado mais explícito no enunciado, mas se elas
pudessem se comunicar livremente o problema seria trivial.

Um abraço,
Claudio.




=
Instruções para entrar na lista, sair da lista e usar a lista em
http://www.mat.puc-rio.br/~nicolau/olimp/obm-l.html
=


Re: [obm-l] Mais IME...

2003-10-03 Por tôpico Cláudio \(Prática\)

 (IME 98)
 Uma embarcação deve ser tripulada por oito homens, dois dos quais só
 remam do lado direito e apenas um, do lado esquerdo. Determine de
 quantos modos esta tripulação pode ser formada, se de cada lado deve
 haver quatro homens
 Observação: A ordem dos homens em cada lado distingue a tripulação.

Pra começar, coloque os dois destros na direita e o canhoto na esquerda.
Escolha dos outros 2 pra direita: Binom(5,2) = 10.
Escolha dos outros 3 pra esquerda: Binom(3,3) = 1
Permutação dos 4 em cada lado: 4!*4! = 576

Número total de maneiras de formar a tripulação: 5760.

*

 Tem só mais uma aki:
 (IME 98)
 Resolva e interprete, geometricamente, o sistema matricial abaixo em
 função de a e b.

 | 1   -23 | | x || -4 |
 | 5   -67 | | y || -8 |
 | 6 8a | | z ||  b |
 Só não sei fazer a parte da interpretação geométrica (resolver sistema
 linear pelo amor de Deus...)

Nesse caso, vou ter que concordar com seu comentário sobre as questões do
IME serem mal-formuladas. Afinal, o que se deve entender por interpretar
geometricamente um sistema de equações?

A existência de uma solução para este sistema significa que o vetor
(-4,-8,b) pertence ao subespaço de R^3 gerado pelos vetores (1,5,6),
(-2,-6,-8) e (3,7,a).
Resolvendo o sistema, você acha o seguinte:
Se a  -22, então o sistema tem uma solução única:
Se a = -22 e b = 36, então o sistema tem uma infinidade de soluções
Se a = -22 e b  36, então o sistema não tem solução.

Em termos geométricos, isso significa que:
a  -22 == (1,5,6), (-2,-6,-8) e (3,7,a) formam uma base para o R^3.
a = -22 e b = 36 == (1,5,6), (-2,-6,-8) e (3,7,-22) são L.D. e geram apenas
um subespaço de dimensão 2 do R^3 (um plano passando pela origem), o qual
contém (-4,-8,36)
a = -22 e b  36 == (1,5,6), (-2,-6,-8) e (3,7,-22) são L.D. e geram
apenas um subespaço de dimensão 2 do R^3, o qual não contém (-4,-8,b)

Nao sei se isso é o que eles queriam, mas...

Um abraço,
Claudio.

=
Instruções para entrar na lista, sair da lista e usar a lista em
http://www.mat.puc-rio.br/~nicolau/olimp/obm-l.html
=


[obm-l] Baricentros

2003-10-03 Por tôpico Cláudio \(Prática\)



Oi, Alexandre:

Tentei responder às suas indagações, que aliás são bem interessantes e 
pertinentes.

Em analítica a média aritmética entre a e b = [a+b]/2
*** Sim, é o ponto médio do segmento ab.
O baricentro do triangulo ABC = [a+b+c]/3o baricentro do 
tetraedro ABCD=[a+b+c+d]/4 (no r3)
*** Sim e sim.O baricentro de um tetraedro não regular seria 
[a+b+c+d]/4 também?
*** Sim. Tome um dos vértices de um tetraedro regular ABCD (digamos A) como 
sendo a origem do seu sistema de coordenadas.
Um tetraedro qualquer pode ser obtido de ABCD por meio de uma transformação 
linear T, que leva os pontos A, B, C e D em T(A) (= A= lembre-se de 
que A é a origem), T(B), T(C) e T(D), respectivamente. Seja M o baricentro do 
tetraedroregular inicial. Então, T(M) será o baricentro do 
tetraedroresultante. Mas M = (A+B+C+D)/4 e T é linear == 
T(M) = T((A+B+C+D)/4) = (T(A)+T(B)+T(C)+T(D))/4.
e o baricentro de uma pirâmide 
de base quadrada seria [a+b+c+d+e]/5 ?
*** Não. O baricentro de qualquer pirâmide fica a uma distância da base 
igual a 1/4 da altura da pirâmide.
A melhor forma de ver isso é considerar uma pirâmide cuja base é um n-gono 
regular pertencente ao plano xy.
Prosseguindo na sua analogia, o baricentro seria o ponto:
(a_1+a_2+...+a_n+v)/(n+1) (v = vértice da pirâmide)
Mas os vértices a_1, ..., a_n têm todos coordenada z = 0. 
Assim, a coordenada z do baricentro seria igual a v/(n+1), que tende a zero 
quando n tende a infinito (caso em que teríamos um cone circular).
Mas o baricentro de um cone circular fica a uma distância da base igual a 
1/4 da altura desse cone (e não no plano da base, como indica a sua 
analogia).
e o baricentro de um cubo seria [a+b+c+d+e+f+g+h]/8?
*** Sim. É o centro geométrico do cubo.
e o baricentro de um paralelepípedo seria [a+b+c+d+e+f+g+h]/8 
também?
*** Sim. Um paralelepípedo é imagem de um cubo por alguma transformação 
linear.

Um abraço,
Claudio.



Re: [obm-l] Baricentros

2003-10-03 Por tôpico Cláudio \(Prática\)



Só pra complementar:

Você pode ver que o baricentro de uma pirâmide 
qualquer está a 1/4 da altura simplesmente decompondo a base em triângulos. Isso 
induz uma decomposição da pirâmide em várias pirâmides de base triangular, todas 
da mesma altura.Como o baricentro de cada uma dessas pirâmides 
(tetraedros, de fato) está a 1/4 da altura,você pode concluirque o 
baricentro da pirâmide original também estará, pois a sua coordenada z é uma 
média ponderada das coordenadas z de cada um dos baricentros das pirâmides 
triangularese estas coordenadas são todas iguais.

Um abraço,
Claudio.



  - Original Message - 
  From: 
  Cláudio (Prática) 
  To: [EMAIL PROTECTED] 
  Sent: Friday, October 03, 2003 2:50 
  PM
  Subject: [obm-l] Baricentros
  
  Oi, Alexandre:
  
  Tentei responder às suas indagações, que aliás são bem interessantes e 
  pertinentes.
  
  Em analítica a média aritmética entre a e b = [a+b]/2
  *** Sim, é o ponto médio do segmento ab.
  O baricentro do triangulo ABC = [a+b+c]/3o baricentro do 
  tetraedro ABCD=[a+b+c+d]/4 (no r3)
  *** Sim e sim.O baricentro de um tetraedro não regular seria 
  [a+b+c+d]/4 também?
  *** Sim. Tome um dos vértices de um tetraedro regular ABCD (digamos A) 
  como sendo a origem do seu sistema de coordenadas.
  Um tetraedro qualquer pode ser obtido de ABCD por meio de uma 
  transformação linear T, que leva os pontos A, B, C e D em T(A) (= A= 
  lembre-se de que A é a origem), T(B), T(C) e T(D), respectivamente. Seja M o 
  baricentro do tetraedroregular inicial. Então, T(M) será o baricentro do 
  tetraedroresultante. Mas M = (A+B+C+D)/4 e T é linear == 
  T(M) = T((A+B+C+D)/4) = (T(A)+T(B)+T(C)+T(D))/4.
  e o baricentro de uma 
  pirâmide de base quadrada seria [a+b+c+d+e]/5 ?
  *** Não. O baricentro de qualquer pirâmide fica a uma distância da base 
  igual a 1/4 da altura da pirâmide.
  A melhor forma de ver isso é considerar uma pirâmide cuja base é um 
  n-gono regular pertencente ao plano xy.
  Prosseguindo na sua analogia, o baricentro seria o ponto:
  (a_1+a_2+...+a_n+v)/(n+1) (v = vértice da pirâmide)
  Mas os vértices a_1, ..., a_n têm todos coordenada z = 0. 
  Assim, a coordenada z do baricentro seria igual a v/(n+1), que tende a 
  zero quando n tende a infinito (caso em que teríamos um cone circular).
  Mas o baricentro de um cone circular fica a uma distância da base igual a 
  1/4 da altura desse cone (e não no plano da base, como indica a sua 
  analogia).
  e o baricentro de um cubo seria [a+b+c+d+e+f+g+h]/8?
  *** Sim. É o centro geométrico do cubo.
  e o baricentro de um paralelepípedo seria 
  [a+b+c+d+e+f+g+h]/8 também?
  *** Sim. Um paralelepípedo é imagem de um cubo por alguma transformação 
  linear.
  
  Um abraço,
  Claudio.
  


[obm-l] Re: [obm-l] Dúvidas

2003-09-22 Por tôpico Cláudio \(Prática\)



Dois probleminhas:

1) Qual é a soma dos algarismos do produto em que 
os fatores são um número constituído por 45 algarismos iguais a 9 e o outro, um 
número cosntituído por 45 algarismos iguais a 5?

Considere o polinômio:
P(x) = [9*(x^44 + x^43 + ... + x + 1)]*[5*(x^44 + 
x^43 + ... + x + 1)] =
45*(x^44 + x^43 + ... + x + 1)^2 

O valor do produto pedido é igual a P(10) e a soma 
dos seus algarismos é P(1).

Logo, Soma = P(1) = 45*(1 + 1 + ... + 1 + 1)^2 = 
45*45^2 = 45^3 = 91125.

*

2)Há 50 bolas brancas e 50 bolas pretas que 
serãodistribuídasem duas urnas idênticas. Em seguida, um prisioneiro 
escolherá (aleatoriamente) uma dessas urnas e dela retirará uma bola. Se esta 
for preta,o prisioneiroé condenado à morte. Caso contrário,o 
prisioneiroé libertado. Que distribuição de bolas pelas urnas dará ao 
prisioneiro a menor probabilidade de ele ser condenado à morte?

A distribuição ótima é: 
Urna 1: 1 bola branca e nenhuma preta.
Urna 2: 49 bolas brancas e 50 bolas 
pretas.


Um abraço,
Claudio.


[obm-l] Sequência Equidistribuída

2003-09-19 Por tôpico Cláudio \(Prática\)
Oi, Salvador:

Esse teorema é bem interessante.

Acho que ele está relacionado ao seguinte fato:
Na sequência x(n) = 2^n, a probabilidade do algarismo da esquerda da
representação decimal de x(n) ser igual a k (1=k=9) é igual a
log_10((k+1)/k). Ou seja, nessa sequência, pouco mais de 30% dos termos
começam com o algarismo 1. Por outro lado, menos de 5% deles começam com
9. Essa é a tal lei de Benford.

Claro que, como log_10(2) é irracional, a sequência y(n) = log_10(x(n)) mod
1 = n*log_10(2) mod 1 é equidistribuida.



Será que x(n) = cos(n) é equidistribuída?

Um abraço,
Claudio.

- Original Message -
From: Salvador Addas Zanata [EMAIL PROTECTED]
To: Lista OBM [EMAIL PROTECTED]
Sent: Friday, September 19, 2003 11:56 AM
Subject: Re: [obm-l] Valores de aderencia



 Oi amigos,

 Existe um troco chamado teorema da equidistribuicao de Weyl, que diz o
 seguinte: Se uma sequencia a_n em [0,1] por exemplo, satisfizer uma serie
 de relacoes, entao ela eh equidistribuida.

 Por exemplo, a_n=n.w mod 1, com w irracional eh equidistribuida, o que
 quer dizer que ela se espalha uniformemente em [0,1].

 Se alguem quiser, posso dar as condicoes precisas que a seq. deve
 satisfazer. Eh claro que dizer que uma seq. eh equidistribuida eh muito
 mais forte que dizer que ela eh densa.


 Um abraco,

 Salvador




 On Thu, 18 Sep 2003, Claudio Buffara wrote:

  Oi, pessoal:
 
  Sabemos que x(n) = cos(n) tem subsequencias que convergem para qualquer
  ponto no intervalo [-1,1].
 
  Pergunta:
  O que eh que a funcao cosseno tem de especial para que isso aconteca, ou
  seja, que propriedade(s) uma funcao real precisa ter para gerar
sequencias
  com subsequencias convergindo para qualquer ponto da imagem da funcao?
 
  Sobre a funcao cosseno eu consigo pensar em 4 coisas:
  1) Ela eh limitada;
  2) Ela eh periodica de periodo irracional;
  3) Ela eh continua;
  4) Ela eh uma sobrejecao em [-1,1].
 
  O meu chute eh que (1) e (3) sao irrelevantes, que (2) eh uma condicao
  suficiente mas nao necessaria, pois acho que y(n) = cos(n^2) tambem tem
  subsequencias convergindo para qualquer ponto de [-1,1], e que (4) eh
uma
  condicao necessaria mas nao suficiente, pois f(x) = sen(pi*x) tambem eh
uma
  sobrejecao em [-1,1] mas z(n) = sen(pi*n) eh constante e igual a zero.
 
  Como sempre, qualquer ajuda serah bem-vinda.
 
  Um abraco,
  Claudio.
 
 
=
  Instruções para entrar na lista, sair da lista e usar a lista em
  http://www.mat.puc-rio.br/~nicolau/olimp/obm-l.html
 
=
 

 =
 Instruções para entrar na lista, sair da lista e usar a lista em
 http://www.mat.puc-rio.br/~nicolau/olimp/obm-l.html
 =

=
Instruções para entrar na lista, sair da lista e usar a lista em
http://www.mat.puc-rio.br/~nicolau/olimp/obm-l.html
=


Re: [obm-l] Valores de aderencia de cos(n)

2003-09-19 Por tôpico Cláudio \(Prática\)
Oi, Artur:

De fato, a continuidade da função cosseno é essencial (pelo menos na
demonstração que eu obtive).

Acho que dá pra provar o seguinte:
Seja X contido em R tal que X contém todos os inteiros positivos.
Seja f: R - R uma função contínua, par (f(-x) = f(x)) e periódica com
período irracional.
Então, a sequência (f(n)) tem subsequências que convergem para qualquer
ponto de f(R).

Seja p = período de f. Como p é irracional, A = {r + s*p; r,s: inteiros} é
denso em R e tal que, para cada inteiro positivo n, existem inteiros  r(n)
e  s(n)  tais que:
0  | r(n) + s(n)*p |  1/n.

Tomemos b em f(R).
Seja a em R tal que f(a) = b.
Para cada n, tomamos um ponto z(n) do conjunto:
A inter ( a -1/n , a + 1/n )
(o qual é sempre não-vazio já que A é denso em R).
Obtemos assim uma sequencia z(n) de pontos de A que converge para a.

Seja z(n) = r(n) + s(n)*p, onde r(n) e s(n) são inteiros.
Temos que f(z(n)) = f(r(n) + s(n)*p) = f(r(n)) = f(|r(n)|).

Como z(n) - a e f é contínua, temos que f(z(n)) = f(|r(n)|) - f(a) = b.

Agora, basta tomar uma subsequência não-decrescente |r(n_i)| da sequência
|r(n)|.
|r(n_i)| também será uma subsequencia não-decrescente da sequência y(n) = n
==
f(|r(n_i)|) será uma subsequência de (f(n)) que converge para b.



=
Instruções para entrar na lista, sair da lista e usar a lista em
http://www.mat.puc-rio.br/~nicolau/olimp/obm-l.html
=


[obm-l] Re: [obm-l] Como resolvê-las???

2003-09-17 Por tôpico Cláudio \(Prática\)

2^x - 3^(1/x)=1

Seja F: R-{0} - R dada por F(x) = 2^x - 3^(1/x) - 1
Voce quer justamente os zeros de F.

F é contínua e diferenciável em todo o seu domínio (R - {0})
com
F'(x) = 2^x*ln(2) + (1/x^2)*3^(1/x)*ln(3)

F'(x)  0 para todo x em R - {0} ==
F é crescente em todo o seu domínio.

Repare que lim(x - 0-) F(x) = 0 ==
F(x)  0 para todo x  0 ==
F não tem zeros negativos

F(1) = -2  0 e F(2) = 3 - raiz(3)  0 ==
F tem um zero entre 1 e 2 e pela positividade de F', este zero é único.
Com uma planilha eu achei x = 1,58496250071 com precisão de 11 casas
decimais.

Um abraço,
Claudio.

=
Instruções para entrar na lista, sair da lista e usar a lista em
http://www.mat.puc-rio.br/~nicolau/olimp/obm-l.html
=


[obm-l] Re: [obm-l] Como resolvê-las???

2003-09-17 Por tôpico Cláudio \(Prática\)



Oi, Dirichlet:

Imagino que sua idéia tenha sido multiplicar a 
equação por 3^x.

Assim: 
2^x - 3^(1/x) = 1 ==
6^x - 3^(x+1/x) = 3^x ==
6^x - 3^x = 3^(x+1/x) e não 6^x - 3^x = 
1.

Ou seja, 3^x*3^(1/x) não é igual a 1 
(de fato, para nenhum valor real de x, pois isso 
implicaria em x + 1/x = 0 == x^2 + 1 = 0)

Um abraço,
Claudio.


  - Original Message - 
  From: 
  Johann Peter Gustav Lejeune 
  Dirichlet 
  To: [EMAIL PROTECTED] 
  Sent: Wednesday, September 17, 2003 1:19 
  PM
  Subject: Re: [obm-l] Como 
  resolvê-las???
  
  6^x-1=3^x ou 6^x-3^x=1.Como o lado esquerdo cresce mais rapido que o 
  direito, basta testar as soluçoes ate um certo ponto.leonardo 
  mattos [EMAIL PROTECTED] 
  wrote: 
  sen(2x-a) 
- Ksen(a)=02^x - 
3^(1/x)=1_MSN 
Hotmail, o maior webmail do Brasil. http://www.hotmail.com=Instruções 
para entrar na lista, sair da lista e usar a lista 
emhttp://www.mat.puc-rio.br/~nicolau/olimp/obm-l.html=
  
  
  Desafio 
  AntiZona: participe do jogo de perguntas e respostas que vai dar1 
  Renault Clio, computadores, câmeras digitais, videogames e muito 
mais!


[obm-l] Imagem densa

2003-09-17 Por tôpico Cláudio \(Prática\)
Oi, Salvador:

Em essência eu acho que é isso, apesar de você ter omitido alguns passos
facilmente formalizáveis.

Uma pergunta que me ocorre é: que propriedade de f(x) = cos(x) você usou?
Apenas que f é uma sobrejeção de [0,2Pi] em [-1,1]?
Será que o fato de que f é contínua também é relevante?
Que tal cos(m + n*2Pi) = cos(|m|) com m, n inteiros?

Mais geralmente, a minha pergunta é a seguinte:
Dado um conjunto X contido em R e uma função f: X - R, se A é um
subconjunto qualquer de X tal que A é denso em X, qual a condição (sobre X e
f) para que f(A) seja denso em f(X)?

Um abraço,
Claudio.

- Original Message -
From: Salvador Addas Zanata [EMAIL PROTECTED]
To: Lista OBM [EMAIL PROTECTED]
Sent: Wednesday, September 17, 2003 2:18 PM
Subject: Re: [obm-l] Valores de aderencia de cos(n)




 A parte do n que importa eh n mod 2.pi, que eh denso no intervalo
 [0,2.pi], porque n/2.pi eh irracional. Logo cos(n) eh denso em
 cos([0,2.pi])=[-1,1]. Acho que eh so isso.


 Abraco,

 Salvador



 On Tue, 16 Sep 2003, Claudio Buffara wrote:

  E pra completar a serie de problemas sobre conjuntos densos em R, aqui
vai
  mais um problema do livro Curso de Analise - vol. 1 do Elon (cap. IV -
ex.
  46 da 6a. edicao):
 
  Prove que o conjunto dos valores de aderencia da sequencia x(n) = cos(n)
eh
  o intervalo fechado [-1,1].
 
  OBS: a eh valor de aderencia de x(n) == a eh limite de alguma
subsequencia
  de x(n).
 
  Sugestao: Use o fato de que se b eh irracional, entao o conjunto {m +
n*b;
  m,n: inteiros} eh denso em R (o que uma coisa tem a ver com a outra???)
 
  Um abraco,
  Claudio.
 
 
 
=
  Instruções para entrar na lista, sair da lista e usar a lista em
  http://www.mat.puc-rio.br/~nicolau/olimp/obm-l.html
 
=
 

 =
 Instruções para entrar na lista, sair da lista e usar a lista em
 http://www.mat.puc-rio.br/~nicolau/olimp/obm-l.html
 =

=
Instruções para entrar na lista, sair da lista e usar a lista em
http://www.mat.puc-rio.br/~nicolau/olimp/obm-l.html
=


Re: [obm-l] Fatorial Quadrado

2003-09-17 Por tôpico Cláudio \(Prática\)



Oi, Ed:

Infelizmente, você só pode dizer que d(m*n) = 
d(m)*d(n) se m e n forem primos entre si, o que não é válido no caso de 
n!se n = 4, pois mdc(4,2) = 2 (e de fato d(4!) = d(24) = 8, mas 
d(4)*d(3)*d(2)*d(1) = 3*2*2*1 = 12).

Mas valeu pela atenção ao problema.

Um abraço,
Claudio.


  - Original Message - 
  From: 
  Eduardo Azevedo 
  To: [EMAIL PROTECTED] 
  Sent: Wednesday, September 17, 2003 3:05 
  PM
  Subject: Re: [obm-l] Fatorial  
  Quadrado
  
   Oi, pessoal: Alguem conhece alguma demonstracao de 
  que nenhum fatorial  1 eh quadrado perfeito que nao use o 
  postulado de Bertrand?
  É só a gente ver que os quadrados são os números 
  que tem uma quantidade ímpar de divisores. Afinal, os divisores de n vem em 
  pares n e n/d. A única exceção é, se existir, raiz de n.
  
  Agora, se chamarmos de d(n) o número de divisores 
  de n temos
  
  d(n!) = d(n)*d(n-1)*...d(2)*d(1), que é par pois 
  d(2) é par. Então n! não pode ser quadrado.
  
  
  abrc
  
  -ed


Re: [obm-l] Juros....

2003-09-15 Por tôpico Cláudio \(Prática\)



(1 + 0,1025/2)^2 = 1,105126...

exp(0,1020) = 1,107383...  
1,105126...

Logo, um investimento a 10,2% aa com juros 
compostos continuamente é melhor.

Um abraço,
Claudio.

  - Original Message - 
  From: 
  [EMAIL PROTECTED] 
  To: [EMAIL PROTECTED] 
  Sent: Sunday, September 14, 2003 12:26 
  AM
  Subject: [obm-l] Juros
  Qual o melhor investimento10,25% ao ano, com juros 
  compostos semestralmente ou 10,20% ao ano com juros compostos 
  continuamenteUm cara me perguntou isso hoje, não tenho certeza sobre o 
  enunciado, mas ele me disse que viu esse problema em um livro do 
  ElonAlguém já ouviu falar?? Será esse o enunciado correto??? 
   Um 
  abraço, 
  Crom 


[obm-l] Definição de conjunto denso em R

2003-09-10 Por tôpico Cláudio \(Prática\)
Estou usando a seguinte definição:
Um subconjunto X de R é denso em R == todo intervalo aberto de R contém
algum elemento de X.

(eu falo em intervalo aberto pra excluir o caso de um intervalo fechado
degenerado [a,a] = {a}).

- Original Message -
From: Domingos Jr. [EMAIL PROTECTED]
To: [EMAIL PROTECTED]
Sent: Tuesday, September 09, 2003 11:11 PM
Subject: Re: [obm-l] Conjunto denso em R



 (**) uma questão chata agora é provar que sempre existe p, q que tornem e

 0, pois aí teríamos 0  na + m  1/q.
 pra mim isso parece verdade pois seria extremamente bizarro haver apenas
 aproximações por cima com a precisão denominador²!


 nossa, agora que percebi, isso é completamente desnecessário...
 tome x  y em B, então para algum q inteiro positivo tq 1/q  y - x.

 se -1/q²  e  0, então
 -1/q  na + m  0
 x  y + na + m  y, e segue que existe um elemento entre x, y em B.

 no caso de 0  na + m  1/q tomamos x  x + na + m  y.

 uma pergunta: eu conheci a definição de conjunto denso com base no que
você
 (Cláudio) me disse, é assim mesmo que se prova que um conjunto é denso ou
 existe alguma condição adicional?

 vou pensar na questão dos pontos de acumulação...

 [ ]'s

 =
 Instruções para entrar na lista, sair da lista e usar a lista em
 http://www.mat.puc-rio.br/~nicolau/olimp/obm-l.html
 =

=
Instruções para entrar na lista, sair da lista e usar a lista em
http://www.mat.puc-rio.br/~nicolau/olimp/obm-l.html
=


[obm-l] Pontos de acumulação

2003-09-10 Por tôpico Cláudio \(Prática\)
Oi, Salvador:

Acho que o seu argumento também é válido.

O que eu tinha pensado é que, dado qualquer real positivo b, existe apenas
um número finito de elementos de C em (0,b) e em (b,2b). Assim, b não pode
ser ponto de acumulação de C.

Um abraço,
Claudio.


- Original Message -
From: Salvador Addas Zanata [EMAIL PROTECTED]
To: [EMAIL PROTECTED]
Sent: Wednesday, September 10, 2003 11:16 AM
Subject: Re: [obm-l] Conjunto denso em R - Domingos



 Se x for um ponto de acumulacao de C, entao existe uma seq. de elementos
 distintos de C convergindo para x. Mas qualquer seq. de elementos de C vai
 para infinito, ne? Logo me parece que nao temos pontos de acumulacao.

 Abraco,

 Salvador


 
  Agora, uma questao interessante:
  Se a eh um irracional positivo e C = {n*a + m; m,n: inteiros
nao-negativos},
  serah que C tem algum ponto de acumulacao ou todos os seus pontos sao
  isolados?
 
  Um abraco,
  Claudio.
 
 
 
=
  Instruções para entrar na lista, sair da lista e usar a lista em
  http://www.mat.puc-rio.br/~nicolau/olimp/obm-l.html
 
=
 

 =
 Instruções para entrar na lista, sair da lista e usar a lista em
 http://www.mat.puc-rio.br/~nicolau/olimp/obm-l.html
 =

=
Instruções para entrar na lista, sair da lista e usar a lista em
http://www.mat.puc-rio.br/~nicolau/olimp/obm-l.html
=


Re: [obm-l] Infinitos Primos.

2003-08-16 Por tôpico Cláudio \(Prática\)
A demonstração clássica do caso 4k + 1 leva em conta que, para todo primo p
e todo inteiro N, se p divide N^2 + 1, então p = 2 ou p é da forma 4k + 1.

Sejam p1, p2, ..., pn todos os primos da forma 4k + 1.

Então, os fatores primos de (2*p1*p2*...*pn)^2 + 1 são todos da forma 4k + 3
==
contradição.

*

Existe uma demonstração da infinitude dos primos da forma Nk + 1 para
qualquer N que usa polinômios ciclotômicos. É essa que você conhece?

Um abraço,
Claudio.

- Original Message -
From: Frederico Reis Marques de Brito [EMAIL PROTECTED]
To: [EMAIL PROTECTED]
Sent: Saturday, August 16, 2003 10:56 AM
Subject: [obm-l] Infinitos Primos.




 Pessoal, como todos devem saber dada em toda  progressão aritméticaem
 que a razão e o termo inicial são coprimos existe uma quantidade infinita
de
 primos. Este é o conhecido Teorema de Dirichlet, cuja demonstração  é
 bastante complexa. Alguns casos especiais são facilmente demonstrados como
 4k+3   ou   6k+5 e já foram tratados nesta lista.  Proponho então a
 demonstração  dos seguintes casos:
 10K +1e4k +1 , especialmente o primeiro deles, poias embora
conheça
 as demonstrações gostaria de obter provas mais simples das de que tenho
 conhecimento.
 Se alguém tiver uma idéia, por favor escreva-me.

 Abraços,
 Frederico.

 _
 MSN Messenger: converse com os seus amigos online.
 http://messenger.msn.com.br

 =
 Instruções para entrar na lista, sair da lista e usar a lista em
 http://www.mat.puc-rio.br/~nicolau/olimp/obm-l.html
 =

=
Instruções para entrar na lista, sair da lista e usar a lista em
http://www.mat.puc-rio.br/~nicolau/olimp/obm-l.html
=


[obm-l] Re: [obm-l] Opção_de_Compra

2003-08-14 Por tôpico Cláudio \(Prática\)

 Olá Claudio!

 Interessante isso! :)
 Se o comprador pega a ações emprestado, ele terá que
 devolver daqui um ano a ações mais 100*a*0,2 (20% do
 valor do empréstimo), ou seja, a ações + 20a

Na verdade, 20% é a taxa de juros para empréstimos e aplicações de reais e
não de ações (desculpe-me se eu não deixei isso claro antes). Para este
problema, vamos assumir que você toma ações emprestadas a 0% aa.

 se ele vender essa ação emprestada, ele embolsará
 100*a reais
 então, supondo que x  100a:
 comprando a opção, sobrará 100a-x, o que ele poderá
 investir, obtendo (100a-x)*1,2 em 1 ano.
 ai temos dois casos:

 (i) a ação está valendo R$50
 ele deverá comprar a ações por 50*a, para devolver a
 ações + 20a para o cara que emprestou as ações.
 nesse caso, ele gastará R$ 70a
 o que ele deve ter, para não ter prejuíso, então:
 (100a-x)*1,2 = 70a = 100a-x = 84a = 16a = x =
 a = x/16

 (ii) a ação está valendo R$ 200
 ele exerce a opção e compra 1 ação por R$ 110,
 mas ele tem que devolver a ações, e ainda pagar 20a
 reais.
 então lhe sobra 200 - 200a - 20a = 200-220a
 para ele não ter prejuíso: 200-220a = 0 =
 200 = 220a = 20 = 22a = 10/11 = a
 então:

 x/16 = a = 10/11
 mas para o vendedor não ter prejuíso, x = 40
 será que o vendedor também deve pegar emprestado a
 ação, em vez de emprestar dinheiro do banco, para
 depois comprar a ação?

 Abraços,
 Hélder

Fora o detalhe do custo do aluguel de ações, eu diria que você pegou o
espírito da coisa.
Tanto que eu vou dar a minha solução pro problema.

Vamos assumir o ponto de vista do vendedor da opção:

Na data inicial ele:
- Vende 1 opção == Recebe  x
- Compra a ações == Paga 100a
- Toma o saldo emprestado == Recebe (100a - x)
Saldo Líquido = Zero

Dentro de 1 ano, teremos 2 situações possíveis:
1) Ação vale R$ 200:
- Repaga empréstimo == Paga (100a - x)*1,2
- Compra (1-a) ações == Paga 200*(1 - a)
- Vende 1 ação contra exercício da opção == Recebe 110
Saldo Líquido: Recebe 110 - 200*(1 - a) - (100a - x)*1,2

2) Ação vale R$ 50:
- Repaga empréstimo == Paga (100a - x)*1,2
- Vende a ações na bolsa == Recebe 50a
Saldo Líquido: Recebe 50a - (100a - x)*1,2

Como na data inicial, ele não pagou nem recebeu nada (saldo = zero), a fim
de evitar a possibilidade de uma arbitragem (lucro garantido com risco
zero), dentro de 1 ano ele também não deverá pagar ou receber nada.
Ou seja, em ambos os cenários o seu saldo deverá ser Zero. Isso implica que:

110 - 200*(1 - a) - (100a - x)*1,2 = 0
50a - (100a - x)*1,2 = 0 ==

80a + 1,2x = 90
-70a + 1,2x = 0  ==

a = 3/5  e  x = 35

Ou seja, o preço justo da opção é R$ 35,00 e a fim de se proteger o vendedor
deve, após vendê-la, comprar 0,6 ações.

Repare que o preço justo da opção independe da distribuição de probabilidade
dos preços futuros da ação. Essa é a beleza da sacada do Black e do
Scholes: existe uma quantidade de ações (o a acima) que o vendedor da
opção pode comprar a fim de exatamente neutralizar o risco da opção.

Um abraço,
Claudio.




=
Instruções para entrar na lista, sair da lista e usar a lista em
http://www.mat.puc-rio.br/~nicolau/olimp/obm-l.html
=


Re: [obm-l] Problemas em Aberto - Algarismos

2003-08-14 Por tôpico Cláudio \(Prática\)
Vou mais longe:

Os candidatos são os quadrados da forma:
(3*10^m + A)*10^(2n)
onde A pertence a {1,4,6} e m e n são inteiros não negativos.

Até agora, só encontrei números do tipo:
36, 3600, 36, ..., 36*10^(2n), ...
mas não consegui provar que são os únicos.

Um abraço,
Claudio.

- Original Message -
From: Johann Peter Gustav Lejeune Dirichlet
[EMAIL PROTECTED]
To: [EMAIL PROTECTED]
Sent: Tuesday, August 05, 2003 1:42 PM
Subject: Re: [obm-l] Problemas em Aberto - Algarismos


 Retorno do Abertos da lista?
 Que tal a gente achar quadrados perfeitos do tipo
 3*10^k+6*10^l?
 O tres nao pode vir no final.Talvez
 modulo...Depois eu penso...
  --- Claudio Buffara
 [EMAIL PROTECTED] escreveu:  Caros
 colegas:
 
  Aqui vao dois problemas que ainda estao em
  aberto na lista. O primeiro foi
  enviado pelo Duda Stabel. O segundo eh da
  olimpiada iraniana, se nao me
  engano.
 
  1) Determinar o conjunto de números inteiros
  positivos que satisfazem à duas
  condições: (i) todo número possui exatamente
  dois algarismos não-nulos,
  sendo um deles o três(3), (ii) todo número é
  quadrado perfeito.
 
  2) Prove ou disprove: existe uma potencia de 2
  tal que ao se permutar os
  algarismos de sua representacao decimal
  obtem-se uma outra potencia de 2.
 
  Esse segundo tem uma solucao aparentemente
  simples, mas esta solucao exclui
  o caso de potencias de 2 com algarismos 0
  internos (ou seja, numeros do
  tipo abcdefg).
 
  Um abraco,
  Claudio.
 
 
 

 ___
 Conheça o novo Cadê? - Mais rápido, mais fácil e mais preciso.
 Toda a web, 42 milhões de páginas brasileiras e nova busca por imagens!
 http://www.cade.com.br
 =
 Instruções para entrar na lista, sair da lista e usar a lista em
 http://www.mat.puc-rio.br/~nicolau/olimp/obm-l.html
 =

=
Instruções para entrar na lista, sair da lista e usar a lista em
http://www.mat.puc-rio.br/~nicolau/olimp/obm-l.html
=


Re: [obm-l] Ajuda

2003-08-14 Por tôpico Cláudio \(Prática\)
Oi, Thyago:

A solução padrão pra esse tipo de problema realmente envolve complexos e
polinômios.

Tentando resolver outros problemas similares, você vai perceber que
complexos e polinômios são uma forma de resolução bastante natural.

Os resultados básicos são os seguintes:
1) Todo número complexo pode ser representado na forma R*(cos(a) +
i*sen(a)), onde R é um real não negativo e a é um real qualquer (mas
normalmente limitado ao intervalo [0, 2pi) ou então (-pi,pi]);
2) e^(i*a) = cos(a) + i*sen(a): essa é a definição da função exponencial
complexa, que permite, por exemplo, que você transforme sequências de senos
e cossenos de números reais em PA em sequências de complexos em PG, que as
vezes são mais fáceis de manipular;
3) Um polinômio com coeficientes reais pode ser expresso como o produto de
binômios da forma (x - b) e/ou trinômios da forma (x^2 - 2*R*cos(a)*x +
R^2), onde a e b são números reais quaisquer e R é um real positivo.

Um abraço,
Claudio.


- Original Message -
From: dex [EMAIL PROTECTED]
To: [EMAIL PROTECTED]
Sent: Monday, August 11, 2003 11:05 AM
Subject: [obm-l] Ajuda


 Olá pessoal

 Gostaria de saber uma boa demonstração para o exercício abaixo

 P = sen(pi/n) . sen(2pi/n) . sen(3pi/n) . ... . sen[(n-1)pi/n]
 com n Inteiro positivo

 A resposta é P = n/[2^(n-1)], mas cheguei até este resultado de uma
maneira
 muito pouco prática, nada natural para uma questão de matemática (de
 vestibular). Consegui prová-la utilizando o resultado de uma outra
questão,
 que versava sobre polinômios e complexos. Ou seja, se eu não tivesse visto
 esta outra questão não conseguiria provar nada!

 Atneciosamente
 ¡Thyago!


=
Instruções para entrar na lista, sair da lista e usar a lista em
http://www.mat.puc-rio.br/~nicolau/olimp/obm-l.html
=


Re: [obm-l] Re: [obm-l] Problemas em Aberto - Algarismos

2003-08-14 Por tôpico Cláudio \(Prática\)
Exatamente.

O enunciado da Olimpiada Iraniana de 1999 está aqui:
http://www.geocities.com/CollegePark/Lounge/5284/math/99.html
e não fala nada sobre zeros ou número de algarismos.

Ainda estou tentando...

Um abraço,
Claudio.

- Original Message -
From: [EMAIL PROTECTED]
To: [EMAIL PROTECTED]
Sent: Tuesday, August 05, 2003 12:45 PM
Subject: [obm-l] Re: [obm-l] Problemas em Aberto - Algarismos


 Esse segundo problema caiu na OBM 2000, numa versão mais fácil.
  Acho que foi essa versão a que vc resolveu, jah que ele dizia que as duas
 potências têm que ter o mesmo número de algarismos, de modo que os zeros
 não modificavam a quantidade de algarismos.
  Ateh mais,
  Yuri

 -- Mensagem original --

 Caros colegas:
 
 Aqui vao dois problemas que ainda estao em aberto na lista. O primeiro
 foi
 enviado pelo Duda Stabel. O segundo eh da olimpiada iraniana, se nao me
 engano.
 
 1) Determinar o conjunto de números inteiros positivos que satisfazem à
 duas
 condições: (i) todo número possui exatamente dois algarismos não-nulos,
 sendo um deles o três(3), (ii) todo número é quadrado perfeito.
 
 2) Prove ou disprove: existe uma potencia de 2 tal que ao se permutar os
 algarismos de sua representacao decimal obtem-se uma outra potencia de
 2.
 
 Esse segundo tem uma solucao aparentemente simples, mas esta solucao
exclui
 o caso de potencias de 2 com algarismos 0 internos (ou seja, numeros
 do
 tipo abcdefg).
 
 Um abraco,
 Claudio.
 
 
 

 []'s, Yuri
 ICQ: 64992515


 --
 Use o melhor sistema de busca da Internet
 Radar UOL - http://www.radaruol.com.br



 =
 Instruções para entrar na lista, sair da lista e usar a lista em
 http://www.mat.puc-rio.br/~nicolau/olimp/obm-l.html
 =

=
Instruções para entrar na lista, sair da lista e usar a lista em
http://www.mat.puc-rio.br/~nicolau/olimp/obm-l.html
=


[obm-l] Re: [obm-l] Questões da ESAEx Por favor!

2003-08-14 Por tôpico Cláudio \(Prática\)



Ficaram faltando estas três:

3) O produto das distâncias de um ponto qualquer de 
uma hipérbole de equação
 (x^2/a^2) - (y^2/b^2) = 1 às suas 
assíntotas é ?

Assíntotas: 
1) ay- bx = 
0 
2) ay+ bx = 0.

Seja o ponto P = (r,s) pertencente à 
hipérbole.
A distancia de P até a assíntota 1 é (fórmula da 
distância de ponto a reta):
| as - br |/raiz(a^2+b^2)

A distancia de P até a assíntota2 
é:
| as+ br |/raiz(a^2+b^2)

Portanto, Produto = |a^2s^2 - 
b^2r^2|/(a^2+b^2)

Mas P pertence à hipérbole. Logo:
r^2/a^2 - s^2/b^2 = 1 ==
b^2r^2 - a^2s^2 = a^2b^2 ==
|a^2s^2 - b^2r^2| = a^2b^2 ==

Produto = a^2b^2/(a^2+b^2) = 
constante.

*

7) Considerando um sistema linear de 10 equações e 
10 incógnitas, o número de multiplicações e divisões necessárias para resolvê-lo pela regra de Cramer é igual a ?

Serão calculados 11 determinantes 10x10 e depois 
serão necessarias 10 divisões (uma pra cada variável).
Cada determinante tem 10! termos e cada termo 
envolve o produto de 10 numeros == 
cada determinante envolve 10*10! multiplicações 
== 
o numero total de multiplicações será 11*10*10! = 
10*11!

Assim, serão efetuadas 10*11! multiplicações e 11 
divisões.

Moral da história: use eliminação 
gaussiana.

*

9) A matriz da transformação que passa xy = 1 para 
a forma canônica (x^2) / 2 + (y^2)/2 = 1 é ?

xy = 1 é equação de uma hipérbole equilátera tendo 
os eixos coordenados como assíntotas, com semi-eixos medindo raiz(2) e focos 
sobre a reta y = x;

x^2/2 + y^2/2 = 1 é equação de uma circunferência 
de raio raiz(2) centrada na origem. 

Assim, não existe nenhuma transformação linear que 
faça esta passagem.

Por outro lado, se a segunda equação fosse x^2/2 - 
y^2/2 = 1, ela representaria uma hipérbole equilátera cujas assíntotas são as 
retas y = x e y = -x, com semi-eixos medindo raiz(2) e focos sobre o 
eixo-x.

Nesse caso, a transformação seria uma rotação de 
-pi/4 (ou 7pi/4) em torno da origem.
Matriz =|cos(-pi/4) 
sen(-pi/4) | = (1/raiz(2)) * | 1 -1 |
 
| -sen(-pi/4) cos(-pi/4) 
| 
| -1 1 |


Um abraço,
Claudio.


Re: [obm-l] Problemas em Aberto - Algarismos

2003-08-08 Por tôpico Cláudio \(Prática\)
Oi, e_lema (qual o seu nome?):

Meus comentários estão ao longo da sua mensagem.

Um abraço,
Claudio.

- Original Message -
From: [EMAIL PROTECTED]
To: [EMAIL PROTECTED]
Sent: Wednesday, August 06, 2003 8:21 PM
Subject: Re: [obm-l] Problemas em Aberto - Algarismos


 Cláudio obrigado pelas correções, e aqui vai a solução, gostaria
procurasse
 erros nela, ou tentasse simplificá-la.

 Não há quadrado perfeito que termine em 3, logo o 3 deverá ser o 1º alg.
da
 esq. p/ dir.
 Sendo assim os números do tal conjunto deverão ser da forma 300...0n00...0
 ou
 W=3*10^(p+2q+1)+n*10^(2q) ; Sendo todas as incógnitas inteiras
 não-negativas, onde n
 só poderá assumir os valores de: 1, 4, 5, 6, 9

Até aqui, estou 100% de acordo.
De fato, numa mensagem anterior você provou que o único quadrado com n = 6 é
o 36.
Além disso, usando congruência mod 9, também eliminamos o 5 e o 9, da
seguinte forma:

Os quadrados (mod 9) são: 0, 1, 4 e 7.
Como W é quadrado e W == 3+n (mod 9), teremos que:
3+n == 0, 1, 4 ou 7 (mod 9) ==
n == 6, 7, 1 ou 4 (mod 9) ==
(dado que n pertence a {1,4,5,6,9}) n só pode ser 1, 4 ou 6 ==
(em virtude da sua análise do caso n = 6) n só pode ser 1 ou 4.

Resumindo, o problema é provar que não existem quadrados da forma:
3*10^p + 1   e   3*10^p + 4.

 Provemos agora que p só pode ser zero.

 W=300...0n*10^(2q)=K*K, K inteiro :. 300...0n=q*q, q inteiro, logo:
 q*q=3*10^(p+1)+n=30*10^(p)+n :.

Você deveria ter escolhido outra letra que não q, pois esta já estava
sendo usada pra representar o número de zeros à direita (em 10^(2q)), mas
tudo bem...

O problema começa a partir daqui, onde você introduz expoentes possivelmente
irracionais (o que é um pouco inusitado para este problema, mas pode até dar
certo no final) e a formatação/tabulação está bem difícil de entenderSe
você puder dar uma limpada no argumento e na formatação eu agradeceria.

 q*q-n=(q+n^0,5)(q-n^0,5)=[3*10^(t)]*[10^(s)], t,s reais
   q+n^0,5=3*10^t  q+n^0,5=10^s
 q=3*10^t+n^0,5
 temos dois casos: t=s :.  e outs :.  e ;
  ou
   q-n^0,5=10^sq-n^0,5=3*10^t
 q=10^s+n^0,5

 a-)q*q=9*10^(2t)+6*n^(0,5)*10^(t)+n   ou
b-)q*q=10^(2s)+2*n^(0,5)*10^(s)+n

 a-)10^(2t)=10^(a)/3, a inteiro positivo, pois 9*10^(2t)=3*10^(a)=300...0
 Desse jeito q*q=3*10^(a)+6*((n*10^(a))/3)^(0,5)+n=3*10^(a)+2(n*3*10^(a))+n
 q*q só será inteiro se n*3*10^(a) o for também. Mas nehum dos valores
 possíveis de n
 faz essa condição ser obedecida. Daí, hipótese a é falsa.

 b-)10^(2s)=3*10^(a), a inteiro positivo, pois 10^(2s)=3*10^(a)=300...0
 Desse jeito q*q=3*10^(a)+2*(n*3*10^(a))+n, ora recaímos no caso anterior,
 logo a
 hipótese b também é falsa

 Disso concluímos que no número W=300...0n00...0, entre 3 e n não deve
haver
 zeros, com isso
 W=3n00...0=3n*10^(2q)=K*K :. 3n=q*q :. n=6

 Logo a resposta será:

3600...0, onde o nº de zeros é par, ou  3,6*10^(2q+1); q=0 e q inteiro



=
Instruções para entrar na lista, sair da lista e usar a lista em
http://www.mat.puc-rio.br/~nicolau/olimp/obm-l.html
=


Re: Re:[obm-l] Algumas duvidas

2003-08-05 Por tôpico Cláudio \(Prática\)
Oi, Duda:

Por favor não leve o que eu disse tão a sério.

Talvez a maior utilidade desse problema seja realmente como passatempo (em
salas de espera ou viagens de avião).

No mais, acabei de pensar num outro contexto onde esse tipo de coisa
aparece: ao invés de quatro quatros, considere (n+1) números quaisquer
fixos: a_0, a_1, ..., a_n - coeficientes de um polinômio de grau n. Que tipo
de técnicas poderíamos usar para demonstrar que as raízes desse polinômio
podem (ou não podem) ser representadas por meio de certas operações
definidas com os a_k? Por exemplo, se nos restringirmos às quatro operações
básicas mais a extração de raízes, cairemos na teoria de Galois. Será que se
incluirmos mais algumas operações, poderemos representar as raízes de
qualquer polinômio?

Um abraço,
Claudio.


- Original Message -
From: Eduardo Casagrande Stabel [EMAIL PROTECTED]
To: [EMAIL PROTECTED]
Sent: Monday, August 04, 2003 10:33 PM
Subject: Re: Re:[obm-l] Algumas duvidas


 Olá Cláudio!

 Fui infeliz no meu comentário...

 O que me veio à cabeça, na hora em que disse que o problema dos quatro
 quatros era inútil, foi que dificilmente ele apareceria em algum resultado
 matemático. Por exemplo, durante a demontração do TFA nunca precisaremos
de
 tal decomposição. Neste sentido, ele me pareceu inútil. Pensando melhor,
 acho que ele pode gerar algum fruto. Por exemplo, que tipo de técnicas
 poderíamos usar para demonstrar que um número não pode ser representado
com
 quatro quatros e certas operações definidas?

 Mas a verdade é que ele é um problema legal, uma boa diversão. Deveria ter
 me limitado a somente ter dado a solução ao problema.

 Concluo dizendo que foi um prazer conhecê-lo, também. E um prazer conhecer
o
 Luís Lopes e o Morgado! Espero que o pessoal da lista se reúna em maior
 número em oportunidades futuras.

 Abração,
 Duda.


  From: claudio.buffara

 Oi, Duda:

 Discordo da sua avaliacao de que o problema dos quatro quatros eh o mais
 inutil ja inventado.

 Alem de ser um passatempo matematico equivalente ao Logodesafio
(publicado
 em varios jornais e cujo objetivo eh formar o maior numero possivel de
 palavras com um dado conjunto de letras), acho que tambem eh um bom
 exercicio de criatividade - vide este caso extremamente nao trivial do 33.

 Mais ou menos relacionado a esse problema tem aquele da calculadora com a
 tecla de multiplicacao quebrada: Como multiplicar dois numeros numa
 calculadora usando apenas as teclas +, - e 1/x (alem das teclas
 numericas, claro)?

 Depois, o conceito de utilidade em matematica pura eh um tanto quanto
 polemico - lembre-se das discussoes geradas aqui na lista por um certo
 ex-participante que felizmente parece que foi embora de vez...

 No mais, foi um prazer conhece-lo pessoalmente no Rio.

 Um grande abraco,
 Claudio.

 De:[EMAIL PROTECTED]

 Para:obm-l

 Cópia:

 Data:Sat, 2 Aug 2003 23:38:03 -0300

 Assunto:Re:[obm-l] Algumas duvidas



 
  Olá!
 
  Acho que você é colega novo na lista, não lembro de já ter lido alguma
 mensagem sua. Se for, seja muito bem vindo!
 
  Quando eu estava no segundo grau, tinha um amigo que junto comigo pensou
 neste problema (importantíssimo!). Na época, lembro que nós permitimos que
 se usasse o ponto, daí podíamos representar o número .4 = 2/5. Este caso
do
 33, lembro que foi um dos que deu mais trabalho. O que eu encontrei foi o
 seguinte:
 
  [ sqrt(sqrt(sqrt(4^(4! + sqrt(4) ] / sqrt(4)
  = [ (4^24)^(1/8) + 2 ] / 2 = [ 4^3 + 2 ] / 2 = [ 64 + 2 ] / 2 = 33
 
  Este é, quase sem dúvida, o problema de matemática mais inútil que
alguém
 já inventou. Estou bastante convencido disso... ainda não pude demonstrar,
 mas é questão de tempo. ;)
 
  Abração,
  Duda.
 
  PS. Agora que li que não pode usar radiciação com índice oculta, ou
seja,
 não se pode usar a operação raiz quadrada, que invalida essa minha
solução.
 Deixa para lá, envio ela mesmo assim para te dar as boas vindas...
 
 
  De:[EMAIL PROTECTED]

  Para:[EMAIL PROTECTED]

  Cópia:

  Data:Sat, 02 Aug 2003 15:16:48 -0300

  Assunto:[obm-l] Algumas duvidas

 

   Ola pessoal... tenho algumas duvidas e queria saber se vcs poderiam me
   ajudar...
  
   1. Seja n alguma permutação de 123456789. Diz-se que um algarismo está
 no
   lugar certo se o 1 for o 1° digito, o 2 no 2°, 3 no 3°, etc.
Espera-se
 que
   quantos algarismos estejam no lugar certo?
  
   2. se vcs jah leram o homem que calculava, devem conhecer um
problema
 em
   que, usando quatro digitos 4 se escreve todos os numeros de 0 a 100.
Por
   exemplo, 0 = 44 - 44; 1 = 44/44, 2 = 4/4 + 4/4, etc
   Dizem q eh possível escrever todos ate o 100, mas para tanto tem q se
 fazer
   uso do fatorial (4!=24). Naum consigo fazer o 33; alguem pode me
ajudar?
   (vale usar soma, subtração, frações, multiplicação, potência,
 parenteses,
   fatorial e radiciação desde q o índice naum esteja oculto).
  
   _
   MSN Messenger: instale grátis e converse com seus 

Re: [obm-l] altura

2003-08-05 Por tôpico Cláudio \(Prática\)
Aqui vai uma sugestão:
Por cada vértice do triângulo trace uma reta paralela ao lado oposto.

As três retas assim obtidas formarão um triângulo maior tal que as alturas
do triângulo original serão mediatrizes dos lados deste triângulo maior
(prove isso - dica: vão aparecer vários paralelogramos).

Agora, é só provar que as mediatrizes de um triângulo são concorrentes, o
que é mais fácil. Basta lembrar que o ponto de intersecção de duas
mediatrizes é o circuncentro do triângulo.

Um abraço,
Claudio.

- Original Message -
From: denisson [EMAIL PROTECTED]
To: [EMAIL PROTECTED]
Sent: Tuesday, August 05, 2003 12:41 PM
Subject: [obm-l] altura


 como provar que as tres alturas de um triangulo qualquer
 concorrem no mesmo ponto?

 obrigado

 Denisson


 __
 Acabe com aquelas janelinhas que pulam na sua tela.
 AntiPop-up UOL - É grátis!
 http://antipopup.uol.com.br/


 =
 Instruções para entrar na lista, sair da lista e usar a lista em
 http://www.mat.puc-rio.br/~nicolau/olimp/obm-l.html
 =

=
Instruções para entrar na lista, sair da lista e usar a lista em
http://www.mat.puc-rio.br/~nicolau/olimp/obm-l.html
=


Re: [obm-l] Geometria Plana

2003-06-16 Por tôpico Cláudio \(Prática\)
Oi, Marcio e Eduardo:

Esse problema tem uma generalização interessante.

Inicialmente, dado um n-gono (n = 3) inscrito A1A2...An, considere a soma
das medidas (em radianos) dos arcos A1A2, A2A3, ..., AnA1. O valor desta
soma sempre será um múltiplo de 2Pi, digamos 2Pi*m, onde m é um inteiro tal
que 1 = m  n/2. O polígono será convexo se e somente se m = 1.

A generalização é a seguinte:
Dados n segmentos (n = 3) de comprimentos L1, L2, ..., Ln, sempre existirá
um real positivo R tal que os n segmentos formam os lados de um polígono
inscritível num círculo de raio R. Além disso, se ficarmos restritos a uma
valor fixo de m, podemos tomar os segmentos em qualquer ordem que o polígono
formado por eles ainda será inscritível no mesmo círculo.

A primeira parte decorre dos seguintes fatos:
1) Dado um circulo de raio R, uma corda de comprimento L estará compreendida
por um ângulo central de medida 2x, onde x = arcsen(L/(2R)) e 0  x  Pi/2.

2) A função f: (L,+infinito) -- (0,Pi/2), definida por f(x) = arcsen(L/x) é
uma bijeção contínua e monótona decrescente.

Seja L = max(L1,L2,...,Ln).

A função g:(L,+infinito) -- (0,n*Pi/2), definida por:
g(x) = arcsen(L1/x) + arcsen(L2/x) + ... + arcsen(Ln/x)
também será contínua e monótona decrescente (logo, injetiva).
No entanto, g só será sobrejetiva, e portanto uma bijeção, se L1 = L2 = ...
= Ln = L.

Logo, pelo TVI, deverá existir um (único) real positivo R, tal que g(2R) =
m*Pi, onde m = 1 se estivermos interessados em polígonos convexos e 2 = m 
n/2 caso contrário (repare que como n = 3, Pi pertence ao contra-domínio de
g).

*

A 2a. parte decorre do fato de que a definição de g independe da ordem em
que as parcelas arcsen(Lk/x) são somadas.

*

Repare que se usarmos valores distintos de m, obteremos valores distintos de
R. Por exemplo, tome n = 5 e L1 = L2 = ... = L5 = L.

Nesse caso, teremos g: (L,+infinito) -- (0,5Pi/2)
g(x) = 5*arcsen(L/x)  ==

m = 1 == g(2R) = 5*arcsen(L/(2R)) = Pi ==
L = 2R*sen(Pi/5) = lado do pentágono regular convexo inscrito

m = 2 == g(2R) = 58arcsen(L/(2R)) = 2*Pi ==
L = 2R*sen(2Pi/5) = lado do pentágono regular estrelado inscrito.

*

No caso do problema do Eduardo, teremos n = 6, L1 = L2 = L3 = 3 e L4 = L5 =
L6 = 5.

Assim, g:(5,+infinito) -- (0,3Pi) será dada por:
g(x) = 3*(arcsen(3/x) + arcsen(5/x))

m = 1 == g(2R) = Pi ==
arcsen(3/(2R)) + arcsen(5/(2R)) = Pi/3 ==
( fazendo x = 1/(2R) )
arcsen(5x) = Pi/3 - arcsen(3x) ==
( tomando senos e usando a identidada p/ o seno da diferença )
5x = (raiz(3)/2)*cos(arcsen(3x)) - (1/2)*sen(arcsen(3x)) ==
10x = raiz(3)*raiz(1 - (3x)^2) - 3x ==
(13x)^2 = 3*(1 - (3x)^2) ==
169x^2 = 3 - 27x^2 ==
x^2 = 3/196 ==
x = raiz(3)/14  (lembre-se de que x = 1/(2R)  0) ==
R = 1/(2x) = 7/raiz(3) = 7*raiz(3)/3, o que bate com a resposta dop Márcio
(ainda bem!!!)

*

No entanto, também temos que considerar o caso m = 2 (hexágono não convexo).

Nesse caso, teremos:
g(2R) = 2Pi ==
arcsen(3/(2R)) + arcsen(5/(2R)) = 2Pi/3 ==
( fazendo x = 1/(2R) )
arcsen(5x) = 2Pi/3 - arcsen(3x) ==
( tomando senos e usando a identidada p/ o seno da diferença )
5x = (raiz(3)/2)*cos(arcsen(3x)) + (1/2)*sen(arcsen(3x)) ==
10x = raiz(3)*raiz(1 - (3x)^2) + 3x ==
(7x)^2 = 3*(1 - (3x)^2) ==
49x^2 = 3 - 27x^2 ==
x^2 = 3/76 ==
x = raiz(57)/38  (lembre-se de que x = 1/(2R)  0) ==
R = 1/(2x) = 19/raiz(57) = raiz(57)/3.

Assim, o problema do Eduardo admite uma segunda solução R = raiz(57)/3, que
corresponde juatamente ao caso em que o hexágono inscrito é não-convexo.

*

Um abraço,
Claudio.

- Original Message -
From: Marcio [EMAIL PROTECTED]
To: [EMAIL PROTECTED]
Sent: Saturday, June 14, 2003 3:54 PM
Subject: Re: [obm-l] Geometria Plana


 Ligue o centro O do círculo aos 6 vértices. Chame de 2x cada angulo
com
 vertice em O que aparece nos triangulos de base 5, e de 2y os angulos em O
 nos triangulos de base 3.
 Temos 6x+6y = 360  donde x = 60-y.
 No triangulo de base 5, temos
 (i)  5 = 2R * sen(x)  (para ver isso, voce pode tracar a altura
 relativa a base nesse triangulo isosceles) e no de base 3, temos:
 (ii) 3 = 2R * sen(y).
 Eliminando R dessas equacoes: 5seny = 3 senx
 Como x = 60-y, senx = sen(60-y) = [sqrt(3)cosy - seny]/2
 Juntando:
 10seny = 3sqrt(3)cosy - 3seny,  donde tan(y) = 3sqrt(3)/13 e
 portanto, sen(y) = 3sqrt(3)/14.
 Essa ultima conclusao, junto com (ii), permite determinar R:
 3 = 2R * 3sqrt(3)/14, donde R = 7sqrt(3) / 3.

 Obs: Eh legal notar que os lados 3,3,3,5,5,5 podem estar dispostos em
 qualquer ordem que o resultado nao muda.

 Marcio


 - Original Message -
 From: Eduardo Quintas da Silva [EMAIL PROTECTED]
 To: [EMAIL PROTECTED]
 Sent: Saturday, June 14, 2003 8:05 AM
 Subject: [obm-l] Geometria Plana


  Um hexágono inscrito num círculo de raio R, tem 3 lados medindo 3 cm e 3
  lados medindo 5 cm. Calcule R



=
Instruções para entrar na lista, sair da 

[obm-l] Primos com média 27 - Correção

2003-06-13 Por tôpico Cláudio \(Prática\)



Caros colegas da lista:

O problema original era:
Determine o maior primo que pode pertencer a um 
conjunto de primos distintos cuja média aritmética é 27.

A minha resposta (137) está errada (veja 
abaixo).

O Dirichlet achou aresposta correta, que é 
139. A solução dele está reproduzida a seguir:

Contra-exemplo:3+5+7+11+13+17+19+23+29+31+139=27*11

Ponha os primos crescentemente:p1+p2+p3+...+pn=27np1 nao 
pode ser 2 e isso voce ja fez.Bem,os primos inferiores a 27 sao 
3,5,7,11,13,17,19,23.Assim 
pn=27-p1+27-p2+27-p3+...+27-p(n-1)=27+27-3+...=27-23=145.E ja que a joça 
e prima pn=139.
*

Um abraço,
Claudio.


- Original Message - 
From: "Cláudio (Prática)" [EMAIL PROTECTED]
To: [EMAIL PROTECTED]
Sent: Friday, March 14, 2003 2:34 PM
Subject: [obm-l] Primos com média 27(141 e primo? - 
Não, mas 137 é!)

 - Original Message - From: 
[EMAIL PROTECTED] To: [EMAIL PROTECTED] Sent: Friday, March 14, 2003 1:24 PM Subject: 
[obm-l] Re: [obm-l] Primos com média 27(141 e primo?)   
 Mas desde quando 141=3*47 e primo?   -- Mensagem 
original --Ops! Que mancada 
Obrigado, JP.  Veja a solução revisada a seguir - é exatamente 
igual à anterior, só que em vez de parar em 141, que não é primo, vai um 
único passo além e chega em 137, que é primo com certeza!!! 
 Suponha que existem n primos: P1  P2  ...  Pn. 
 Então, teremos: P1 + ... + Pn = 27*n, e queremos achar Pn. 
 Os primos menores que 27 são 2, 3, 5, 7, 11, 13, 17, 19 e 23. 
Vamos chamá-los de "primos inferiores". Todos os demais serão "primos 
superiores".  A fim de "maximizar" Pn, devemos ter a média 
composta do maior número possível de primos inferiores e do menor número 
possível de primos superiores.

*** O erro está justamente na afirmativa acima 
***

Assim, vamos ver se damos a sorte de ter 
todos os 9 primos inferiores e apenas um primo (Pn) superior incluído na 
média.  27*10 = 2+3+...+23+Pn = 100+Pn == Pn = 170 == 
não é primo  Em seguida, podemos eliminar um primo inferior de 
cada vez, começando com o mais alto (23):  27*9 = 
2+3+...+19+Pn = 77+Pn == Pn = 166 == não é primo  Além 
disso, a má notícia é que eliminando um único primo inferior ímpar, nós 
sempre acharemos um valor par para Pn. Logo, se tivermos que eliminar um 
primo inferior, ele só pode ser o 2. Vejamos:  27*9 = 
3+5+...+23+Pn = 98+Pn == Pn = 145 == não é primo.  
O passo seguinte é eliminar dois primos inferiores de cada vez. 
Começando com os dois mais altos (19 e 23), teremos:  
27*8 = 2+3+5+...+17+Pn = 60+Pn == Pn = 156 == não é primo 
 Além disso, da mesma forma que acima, concluímos que eliminando 
qualquer par de primos ímpares resultará em Pn par. Logo, 2 terá que ser 
necessariamente eliminado.  Vamos eliminar 2 e 
23: 27*8 = 3+5+...+19+Pn = 75+Pn == Pn = 141 == não é primo 
(como o JP bem observou)  2 e 19: 27*8 = 
3+5+...+17+23+Pn = 79+Pn == Pn = 137 == esse é primo 
 A fim de completar a análise, devemos considerar o caso em que há 2 ou 
mais primos superiores compondo a média. Suponhamos que a média 
tenha m primos inferiores e n primos superiores (n = 2). 
Então:  27*(m+n) = m*Minf + n*Msup (Minf (Msup) = média 
dos primos inferiores (superiores) ) == Msup = (27*(m+n) - 
m*Minf)/n = (27 - Minf)*m/n + 27  Não é difícil ver que o maior 
valor possível de (27 - Minf)*m ocorre justamente quando todos os 9 
primos inferiores estão presentes == (27 - Minf)*m = 27*m - Minf*m = 
27*9 - (2+3+...+23) = 243 - 100 = 143  Logo, o valor máximo de 
Msup qundo há n primos superiores é menor ou igual a 143/n + 27 == 
uma função decrescente de n.  Com n = 2 ( o menor valor 
permitido de n), teremos que Msup = 143/2 + 27 = 98,5  
137.  Logo, com 2 ou mais primos, Msup será menor do que 137 
== a sequencia de primos distintos com média igual a 27 tem apenas 
um primo superior, igual a 137.   Um 
abraço, Claudio. 


Re: [obm-l] motorista

2003-06-13 Por tôpico Cláudio \(Prática\)

- Original Message -
From: Rafael [EMAIL PROTECTED]
To: OBM [EMAIL PROTECTED]
Sent: Friday, June 13, 2003 1:16 PM
Subject: [obm-l] motorista


 O Sr. Santos chega todo dia à estação do metrô às
 cinco horas da tarde. Neste exato instante, seu
 motorista o apanha e o leva para casa. Um belo dia, o
 Sr. Santos chegou à estação às quatro horas da tarde e
 ao invéns de esperar pelo motorista até as cinco horas
 da tarde resolveu ir andando para casa. No caminho,
 ele encontra com o seu motorista que o apanha e o leva
 de carro para casa e chegam em casa vinte minutos mais
 cedo do que de costume. Algumas semanas mais tarde,
 num outro belo dia , o Sr. Santos chegou à  estação do
 metrô as 4:30h da tarde e, novamente ao invés de
 esperar pelo seu motorista ele resolve ir andando para
 casa e encontra o seu motorista no caminho. Este
 prontamente o apanha e o leva para casa de carro.
 Desta vez, o Sr. Santos chegou em casa mais cedo:
 a) 15minb) 10 minc)5min d)4mine)3min

Oi, Rafael:

Sejam:
d = distancia da estação à casa do Sr. Santos;
v = velocidade do Sr. Santos a pé;
V = velocidade do carro.

Se no primeiro dia, o carro encontrou o Sr. Santos a uma distância x da
estação, então no segundo dia, o encontro foi a uma distância igual a x/2,
já que a velocidade relativa dos dois é constante (e igual a v + V) nos dois
dias, e no segundo dia o adianto do Sr. Santos foi metade do adianto no 1o.
dia.

Logo, no 1o. dia, o tempo do trajeto do carro de volta pra casa foi de (d -
x)/V e no 2o. dia de (d - x/2)/V
Também sabemos que, num dia normal, o trajeto da estação até a casa dura
d/V.

Assim, temos que d/V - (d - x)/V = x/V = 20 min.

Logo, d/V - (d - x/2)/V = x/(2V) = 10 min == alternativa (b).


Um abraço,
Claudio.

=
Instruções para entrar na lista, sair da lista e usar a lista em
http://www.mat.puc-rio.br/~nicolau/olimp/obm-l.html
=


[obm-l] Re: [obm-l] equaçao

2003-06-13 Por tôpico Cláudio \(Prática\)
- Original Message -
From: Rafael [EMAIL PROTECTED]
To: OBM [EMAIL PROTECTED]
Sent: Friday, June 13, 2003 2:04 PM
Subject: [obm-l] equaçao


 O gabarito que eu tenho dessa questão é alternativa d)
 3, mas eu não consigo enxergar mais um valor real para
 m, será que o nosso amigo gabarito está errado?

 Para quantos valores de m, a expressão
 m^2x^2 + 2(m-1)x + 4 é o quadrado de uma expressão
 do primeiro grau em x?(908) resposta:d
 a)0b)1c)2d)3e)4

 Seja a equação do segundo grau:
 m²x² + 2(m - 1)x + 4 = 0

 Para que essa equação seja igual a algo do tipo:
 (ax + b)² = 0

 Isso significa que ela só terá duas raízes reais
 iguais! E sabemos que uma equação do 2º grau tem duas
 raízes iguais se o determinante é igual a zero:
 D = [2(m - 1)]² - 4m².4
 D = 4(m² - 2m + 1) - 16m²
 D = 4(m² - 2m + 1 - 4m²)
 D = 4(-3m² - 2m + 1)

 Então temos:
 4(-3m² - 2m + 1) = 0
 -3m² - 2m + 1 = 0
 (3m - 1).(-m - 1) = 0
 m = 1/3 ou m = -1

 Resposta: Alternativa c) 2.

Oi, Rafael:

Tentando pelo método ingênuo eu acho a mesma coisa...

(ax+b)^2 = a^2x^2 + 2abx + b^2 = m^2x^2 + 2(m-1)x + 4

Igualando coeficientes:
a^2 = m^2 == a = m   ou   a = -m
ab = (m - 1)
b^2 = 4 == b = 2   ou   b = -2

Considerando os quatro casos e usando a 2a. equação:
b = 2, a = m   ou   b = -2, a = -m  == 2m = m - 1 == m = -1
b = 2, a = -m   ou   b = -2, a = m  == -2m = m - 1 == m = 1/3

Um abraço,
Claudio.

=
Instruções para entrar na lista, sair da lista e usar a lista em
http://www.mat.puc-rio.br/~nicolau/olimp/obm-l.html
=


Re: [obm-l] Primos em PA

2003-06-13 Por tôpico Cláudio \(Prática\)
Oi, Gugu:

Agora entendi!  Se toda PA (Kx + L) com mdc(K,L) = 1 contiver um primo,
então o teorema de Dirichlet é verdadeiro.

Mas ainda acho que o enunciado original do problema poderia ser melhor
redigido...

De qualquer forma, muito obrigado.

Um abraço,
Claudio.

=
Instruções para entrar na lista, sair da lista e usar a lista em
http://www.mat.puc-rio.br/~nicolau/olimp/obm-l.html
=


[obm-l] Integração x Derivação

2003-06-12 Por tôpico Cláudio \(Prática\)
Oi, Duda:

Obrigado pela dica. Vou procurar referências a esse algoritmo na internet.

Achei a sua idéia de global x local bem interessante, apesar da minha
dúvida ser sobre o cálculo de integrais indefinidas. Naturalmente, integrais
indefinidas e definidas são relacionadas via o teorema fundamental do
cálculo. Assim, pode ser que você tenha realmente acertado na mosca. Vou
pensar mais a respeito...

Um abraço,
Claudio.

- Original Message -
From: Eduardo Casagrande Stabel [EMAIL PROTECTED]
To: [EMAIL PROTECTED]
Sent: Thursday, June 12, 2003 12:18 PM
Subject: Re: [obm-l] integral de sec x


 Oi Cláudio.

 Agradeço por esta mensagem e pela outra sobre o teorema de Bezout. As duas
 foram muito claras e me auxiliaram a compreender os resultados.

 Queria te indicar uma mensagem do Paulo Santa Rita

 http://www.mail-archive.com/[EMAIL PROTECTED]/msg07483.html

 onde ele comenta sobre o algoritmo de Risch, que tem a ver com como
integrar
 funções de modo algorítmico assim com aprendemos a diferenciá-las. Ontem à
 noite, pensei um pouco sobre por que temos mais dificuldade em integrar do
 que em diferenciar. Minha resposta - mesmo que não seja boa - é que a
 integração envolve uma quantidade cada vez maior de parcelas numa soma
(que
 pode ser difícil de calcular) da quebra de um intervalo em várias partes,
 enquanto a diferenciação envolve um limite que é tomada numa vizinhança
tão
 pequena quanto se queira em torno de um ponto. Me parece que a integração
é
 algo global, e a diferenciação algo local. Ou seja, você precisa
considerar
 todo um intervalo [0,x] e os valores que a função assume em todo ele para
 poder integrá-la. Para diferenciar, você precisa sober apenas a tendência
de
 comportamento numa vizinhança próxima de um ponto (x-e, x+e).

 Obrigado e um abraço,
 Duda.


=
Instruções para entrar na lista, sair da lista e usar a lista em
http://www.mat.puc-rio.br/~nicolau/olimp/obm-l.html
=


[obm-l] Re: [obm-l] Re:_[obm-l]_olimpíadas_ao_redor_do_mundo.....

2003-06-12 Por tôpico Cláudio \(Prática\)



Bom, se você não entendeu o que eu queria dizer, 
então acho que não temos mais nada a conversar.

Claudio.

  - Original Message - 
  From: 
  Johann Peter Gustav Lejeune 
  Dirichlet 
  To: [EMAIL PROTECTED] 
  Sent: Thursday, June 12, 2003 1:33 
  PM
  Subject: Re: [obm-l] 
  Re:_[obm-l]_olimpíadas_ao_redor_do_mundo.
  
  Nao sei direito o que tu queres dizer com isso,mas eu coleciono soluçoes 
  de problemas a dar com o pau.Mas eu nao faço distinçao entre otimizar 
  soluçoes.Soluçao e soluçao e prontoNao da pra dizer qual e a mais otima so 
  olhando,tem que ver mesmo e a historia do problema.ai sim tem graça ficar 
  procurando varias.Mas e ai?Sera que essa mensagem do Crom nao era 
  ambigua?
  E eu as vezes junto as minhas soluçoes as 
  nao-minhas.Cláudio_(Prática) 
  [EMAIL PROTECTED] wrote:
  



Infelizmente você está sendo grosseiro e 
arrogante (mesmo sem querer).
Eu não vejo problema algum em uma pessoa querer 
saber se há soluções alternativas para um problema que ela já 
resolveu.
E isso não tem nada a ver com auto-confiança. 
Acho que as pessoas fazem parte da lista pra aprender e se aperfeiçoar e não 
pra se mostrar e fazer grosserias, que parece ser o seu caso.

Claudio Buffara.


  - Original Message - 
  From: 
  Johann Peter Gustav Lejeune 
  Dirichlet 
  To: [EMAIL PROTECTED] 
  Sent: Wednesday, June 11, 2003 12:59 
  PM
  Subject: Re: [obm-l] olimpíadas ao 
  redor do mundo.
  
  So uma pergunta:voce nao confia em si mesmo?Sem querer ser 
  grosseiro,claro...[EMAIL PROTECTED] 
  wrote: 
  Resolvi o problema abaixo, mas gostaria de ver( se 
possível ) a solução de outros da lista e poder concluir se a minha é a 
mais otimizada ou não ( ficou grande ). Problema:Eduardo 
escreveu todos os produtos, todas as somas e todos os valores absolutos 
das diferenças dos inteiros positivos a_1,a_2,a_3,.,a_100 tomados 
dois a dois. Qual o maior número de inteiros ímpares obtidos por 
Eduardo??ps-Cheguei numa função f(n), que dá o maior número possível 
de inteiros ímpares obtidos por Eduardopara conseguir esse número 
máximo de ímpares é necessário que na sequência de cem números inteiros 
positivos existam 66 ou 67 ímparesSerá que errei nos 
cálculos??? Um 
abraço,am! 
p;nb! sp; Crom 
  
  
  Yahoo! Mail Mais 
  espaço, mais segurança e gratuito: caixa postal de 6MB, antivírus, 
  proteção contra spam.
  
  
  Yahoo! Mail Mais espaço, 
  mais segurança e gratuito: caixa postal de 6MB, antivírus, proteção contra 
  spam.


[obm-l] Bolinhas em Gavetas, PA de Primos e Média de Primos

2003-06-11 Por tôpico Cláudio \(Prática\)
 Neste caso, por serem distintos, os anéis colocados num mesmo dedo
obedecem
 a uma certa ordem. E se, em vez de anéis, tivéssemos seis bolinhas
numeradas
 de 1 a 6 e quatro gavetas numeradas de 1 a 4? (Bolinhas colocadas numa
mesma
 gaveta não obedeceriam a ordem alguma).

 Estou muito tempo ausente, por isso predoem-me se já circulou pela lista
os
 seguintes problemas:

 1) Qual o número máximo de termos que pode ter uma PA cujos termos são
todos
 números primos?


 2) A média aritmética de n números primos é 20. Qual é o maior desses
 números?

 []s, Josimar


Oi, Josimar:

Primeiro as bolinhas...

O no. de maneiras de colocar as 6 bolinhas nas 4 gavetas seria 4^6, pois
haveria 4 gavetas possiveis para se colocar
cada uma das 6 bolinhas.

*

2) O enunciado estah ambiguo, pois ha diversas solucoes. Por exemplo, com n
= 2 os primos poderiam ser {3,37}, {11,29} e {17,23}; com n = 3, teriamos
{2,5,53}, {2,11,47}, {2,17,41}, etc.

No entanto, se a pergunta for Qual o maior primo que pode pertencer a um
conjunto de primos distintos com média aritmética igual a 20?, entao
teremos o
seguinte:

Sejam P1  P2  ...  Pr os primos menores que 20 e Q1  Q2  ... Qs os
primos maiores que 20 que compoem a media. Naturalmente, teremos r + s = n.

A fim de que maximizar Qs, a media deverah ter o maior numero possivel de
Pi's e o menor numero possivel de Qj's.

Os Pi pertencem a {2, 3, 5, 7, 11, 13, 17, 19} ==
r = 8  e  Soma(Pi) = 2 + 3 + ... + 19 = 77

No melhor caso, teriamos r = 8 e s = 1 ==
Media = (77 + Q1)/9 = 20 ==
Q1 = 103, que eh primo.

Logo, o maior primo possivel eh 103.

*

1) Esse problema eh o mais interessante mas, infelizmente, parece estar em
aberto.
Conjectura-se que existem PA's arbitrariamente longas cujos termos sao todos
primos.
No entanto, a PA desse tipo mais longa que se conhece tem apenas 22 termos.
Veja o site:
http://mathworld.wolfram.com/PrimeArithmeticProgression.html
e também:
http://www.utm.edu/research/primes/lists/top20/ArithmeticProg2.html

*

Um abraco,
Claudio.

=
Instruções para entrar na lista, sair da lista e usar a lista em
http://www.mat.puc-rio.br/~nicolau/olimp/obm-l.html
=


[obm-l] Re: [obm-l] olimpíadas ao redor do mundo.....

2003-06-11 Por tôpico Cláudio \(Prática\)



Infelizmente você está sendo grosseiro e arrogante 
(mesmo sem querer).
Eu não vejo problema algum em uma pessoa querer 
saber se há soluções alternativas para um problema que ela já 
resolveu.
E isso não tem nada a ver com auto-confiança. Acho 
que as pessoas fazem parte da lista pra aprender e se aperfeiçoar e não pra se 
mostrar e fazer grosserias, que parece ser o seu caso.

Claudio Buffara.


  - Original Message - 
  From: 
  Johann Peter Gustav Lejeune 
  Dirichlet 
  To: [EMAIL PROTECTED] 
  Sent: Wednesday, June 11, 2003 12:59 
  PM
  Subject: Re: [obm-l] olimpíadas ao redor 
  do mundo.
  
  So uma pergunta:voce nao confia em si mesmo?Sem querer ser 
  grosseiro,claro...[EMAIL PROTECTED] wrote: 
  Resolvi o problema abaixo, mas gostaria de ver( se 
possível ) a solução de outros da lista e poder concluir se a minha é a mais 
otimizada ou não ( ficou grande ). Problema:Eduardo escreveu 
todos os produtos, todas as somas e todos os valores absolutos das 
diferenças dos inteiros positivos a_1,a_2,a_3,.,a_100 tomados dois a 
dois. Qual o maior número de inteiros ímpares obtidos por 
Eduardo??ps-Cheguei numa função f(n), que dá o maior número possível de 
inteiros ímpares obtidos por Eduardopara conseguir esse número máximo de 
ímpares é necessário que na sequência de cem números inteiros positivos 
existam 66 ou 67 ímparesSerá que errei nos 
cálculos??? Um 
abraço,nb! 
sp; Crom 
  
  
  Yahoo! Mail Mais espaço, 
  mais segurança e gratuito: caixa postal de 6MB, antivírus, proteção contra 
  spam.


[obm-l] Re: [obm-l] Re: [obm-l] Recomendação de Filme e Hipercubo

2003-06-06 Por tôpico Cláudio \(Prática\)
Oi, Nicolau:

Uma retificação: quando eu disse que não adianta visualizar um hipercubo no
R^4 eu estava me referindo apenas à minha pessoa. Geometria pra mim sempre
foi um inferno e admito publicamente minha admiração (e também uma certa
inveja) por quem consegue vislumbrar aquelas construções auxiliares mágicas.

Não tenho dúvida de que haja gente por aí que entende perfeitamente 4 ou
mais dimensões (senão não existiriam muitos topologistas, não é mesmo?)

A seção do cubo pendurado é um hexágono regular, certo? A minha pergunta é
justamente a mesma para um 4-hipercubo pendurado (se bem que o conceito de
pendurado em R^4 é meio problemático pra mim) . Acho que um bom começo pra
começar a pensar neste problema é aquele seu artigo sobre as coordenadas dos
vértices dum icosaedro e outros poliedros - nem sempre a posição padrão do
poliedro é a mais conveniente.

Um abraço,
Claudio.

- Original Message -
From: Nicolau C. Saldanha [EMAIL PROTECTED]
To: [EMAIL PROTECTED]
Sent: Friday, June 06, 2003 10:50 AM
Subject: [obm-l] Re: [obm-l] Recomendação de Filme e Hipercubo


 On Thu, Jun 05, 2003 at 05:47:45PM -0300, Claudio Buffara wrote:
  Nao adianta tentar visualizar um hipercubo em R^4 (por mais que os
desenhos
  de tesseracts sejam intrigantes). Pra mim, a melhor forma de tratar
  hipercubos e outros animais que existem em R^n com n  3 eh usar a boa e
  velha algebra.

 Acho essa discussão interessante mas não é matemática:
 é psicologia, pedagogia senão neurologia.

 Quando eu penso no hipercubo a minha sensação subjetiva é a de que
 estou usando a parte geométrica da cabeça, não a algébrica.
 Se me perguntam algo como quantas faces tem um sólido de 4 dimensões
 eu tenho a sensação de ver o sólido e contar as faces.
 Claro que pode-se dizer que eu estou visualizando uma projeção
 (em  4 dimensões) ou até um diagrama (discreto),
 mas eu definitivamente não escrevo mentalmente as coordenadas
 dos vértices ou as equações para as faces.

 Algumas pessoas afirmam de forma categórica (e a meu ver ingênua)
 que o ser humano é incapaz de entender geometricamente mais
 do que 3 dimensões e ficam meio chocadas quando eu contradigo.
 Talvez seja verdade que o cérebro humano tenha peças especializadas
 em tratar problemas 3d mas nenhuma peça análoga para dimensão mais alta:
 isso é mais ou menos plausível mas tanto quanto eu saiba
 isso não é ciência, é pura especulação.
 Vale notar que a capacidade de visualizar objetos 3d varia muito
 de pessoa para pessoa e mesmo entre matemáticos.
 A maioria das pessoas tem dificuldades para responder algo tipo:
 pendure um cubo por um vértice e passe um plano horizontal pelo
 centro; a interseção do plano com o cubo é um ...?
 Existem relatos muito confiáveis de pessoas com capacidades
 de visualização 3d extraordinárias.

 []s, N.

 =
 Instruções para entrar na lista, sair da lista e usar a lista em
 http://www.mat.puc-rio.br/~nicolau/olimp/obm-l.html
 =

=
Instruções para entrar na lista, sair da lista e usar a lista em
http://www.mat.puc-rio.br/~nicolau/olimp/obm-l.html
=


Re: [obm-l] problema

2003-06-06 Por tôpico Cláudio \(Prática\)
Oi, Gugu:

Eu já tinha visto essa dedução de MG = MA a partir do rearranjo mas, apesar
de interessante, é mágica demais pro meu gosto.

Eu prefiro aquela em que você vai trocando Xmin e Xmax por G e Xmin*Xmax/G
até que todos os números fiquem iguais.

Um abraço,
Claudio.


- Original Message -
From: Carlos Gustavo Tamm de Araujo Moreira [EMAIL PROTECTED]
To: [EMAIL PROTECTED]
Sent: Friday, June 06, 2003 1:27 AM
Subject: Re: [obm-l] problema


Caro Claudio,
E' interessante notar que isso da' uma prova da desigualdade das medias
 aritmetica e geometrica usando a desigualdade do rearranjo (nesse caso na
 versao que determina o menor produto interno possivel de um vetor por um
 rearranjo seu) : sejam x(1),...,x(n) positivos. Nao ha' perda de
 generalidade em supor que seu produto e' 1 (senao dividimos todos pela sua
 media geometrica). Seja a(1)=1, a(2)=x(1), a(3)=x(1).x(2), ... ,
 a(n)=x(1)x(2)...x(n-1). Note que a(n)=1/x(n). Temos entao, pela
desigualdade
 abaixo, n=a(1)/a(n)+a(2)/a(1)+a(3)/a(2)+...+a(n)/a(n-1)=
 =x(n)+x(1)+x(2)+...+x(n-1), que e' o que queriamos provar.
Abracos,
Gugu


=
Instruções para entrar na lista, sair da lista e usar a lista em
http://www.mat.puc-rio.br/~nicolau/olimp/obm-l.html
=


Re: [obm-l] problema

2003-06-06 Por tôpico Cláudio \(Prática\)

- Original Message -
From: Ricardo Prins [EMAIL PROTECTED]
To: [EMAIL PROTECTED]
Sent: Friday, June 06, 2003 11:45 AM
Subject: Re: [obm-l] problema


 Muito obrigado a todos vocês! Ontem eu acabei achando na Eureka 5 muitas
 coisas interessantes quanto à desigualdades em geral...preciso estudar
mais!
 aqui vai outro problema que eu não consegui resolver...é do lidski...

 uma sucessão infinita de números x1,x2,x3,...,xn,... (x10) para qualquer
 n=3 satisfaz à condição
 (x1²+x2²++xn-1²)(x2²+x3³+...+xn²)=(x1x2+x2x3+...+xn-1xn)²

 demonstrar que x1,x2,x3,...,xn,... são termos sucessivos de uma p.g.


Oi, Ricardo:

Aqui temos que provar que existe um no. real q tal que:
x(2) = q*x(1),  x(3) = q*x(2),  ..., x(n) = q*x(n-1),  ...

Uma idéia é usar a desigualdade de Cauchy-Schwarz relativa ao produto
escalar de 2 vetores do R^m.

Ela diz o seguinte:
Se x e y são vetores do R^m, então | x . y |^2 = || x ||^2 * || y ||^2, com
igualdade sss x e y são LI
onde:
x . y = produto escalar de x e y
| x . y | = módulo do número real  x . y
|| x || = módulo do vetor x = raiz(x(1)^2 + x(2)^2 + ... + x(m)^2)

A expressão do enunciado é justamente | x . y |^2 = || x ||^2 * || y ||^2
com:
x = ( x(1) , x(2) , ... , x(n-1) ) e y = ( x(2) , x(3) , ... , x(n) ):
vetores do R^(n-1).

Logo, concluímos que x e y são LI  ==
existe um no. real q tal que y = qx  ==
x(2) = q*x(1), x(3) = q*x(2), ..., x(n) = q*x(n-1) ==
os x(k) são termos de uma PG.

Esse artigo da Eureka 5 tem uma demonstração de Cauchy-Schwarz (lá é chamada
apensa de Cauchy - é a proposição no. 3).

Um abraço,
Claudio.

=
Instruções para entrar na lista, sair da lista e usar a lista em
http://www.mat.puc-rio.br/~nicolau/olimp/obm-l.html
=


Re: [obm-l] diferenca entre raizes

2003-06-06 Por tôpico Cláudio \(Prática\)
- Original Message -
From: Fábio Dias Moreira [EMAIL PROTECTED]
To: [EMAIL PROTECTED]
Sent: Sunday, June 01, 2003 4:04 PM
Subject: Re: [obm-l] diferenca entre raizes


 -BEGIN PGP SIGNED MESSAGE-
 Hash: SHA1

 On Monday 26 May 2003 22:51, Claudio Buffara wrote:
  [...]
  Respondendo a sua pergunta:
  Eu nao enxerguei fatoracao nenhuma. Porisso, assumi que o polinomio
  original de grau 4 se fatorava no produto de dois polinomios quadraticos
(o
  que, alias, eh sempre verdade) e, apos um pouco de braco, obtive os
  coeficientes correspondentes.
  [...]

 Um macete que o Ralph já ensinou aqui na lista também é muito útil para
 fatorar polinômios de quarto grau: procure no seu problema original
condições
 suficientes (mesmo que não sejam necessárias) para que x seja raiz.

 Por exemplo, seja f(x)=x^2-3x+3. Resolva f(f(x))=x.

 Basta que f(x)=x, logo sabemos um dos fatores de segundo grau da equação
de
 quarto grau.


Oi, Fábio:

Gostei desse macete!
De fato:
f(x) = x == f(f(x)) = f(x) = x

Mas f(x) = x ==
x^2 - 3x + 3 = x ==
x^2 - 4x + 3 = 0 ==
x = 1  ou  x = 3

f(f(x)) - x = (x^2 - 3x + 3)^2 - 3(x^2 - 3x + 3) + 3 - x  =
x^4 - 6x^3 + 12x^2 - 10x + 3 = p(x)

Dividindo p(x) por x^2 - 4x + 3, obtemos x^2 - 2x + 1 (resto = 0) ==
p(x) = (x - 1)^3*(x - 3) ==
as raízes de f(f(x)) = x são x = 1 (tripla) e x = 3.

*

O problema original era fatorar o polinômio:
p(x) = x^4 + 2x^3 - x^2 - 6x - 3

Você consegue ver alguma forma de se aplicar o macete?

Um abraço,
Claudio.

=
Instruções para entrar na lista, sair da lista e usar a lista em
http://www.mat.puc-rio.br/~nicolau/olimp/obm-l.html
=


[obm-l] Lagrange x Chinês

2003-06-05 Por tôpico Cláudio \(Prática\)
Oi, Maçaranduba:

Acho que o que você quer está aqui:
http://www.cs.berkeley.edu/~wkahan/MathH90/CRTasLIF.pdf

Um abraço,
Claudio.

  --- Carlos Maçaranduba [EMAIL PROTECTED]
 escreveu:  Fui perguntar como resolver este
 problema(abaixo) e
  meu professor só disse que eu deveria dar uma olhada
  no teorema chines do resto para inteiros e
  considerar
  para polinomios usando homomorfismo entre
  aneis.Confesso que minha cabeça entrou em parafuso e
  por isso eu perguntei a vcs qual a relaçao entre
  teorema chines do resto e a interpolaçao de
  Lagrange.Ai vai o problema:
  NOTAÇÃO:
  * - multiplicaçao.
  y_i - o i-esimo y.
  a/b - a dividido por b
  PROD_i=m,h,(X - a_i) - Produtorio de todos os
  fatores
 
  (X - a_i), sendo i começando de   m e indo ate h.
 
  (Interpolaçao de Lagrange)Seja k um corpo e n=1 um
  numero inteiro.Provar que dados dois subconjuntos de
  elementos de k ,{a_1, ... a_n+1} e {b_1, ... b_n+1},
  onde a_1,...,a_n+1 sao distintos, existe um unico
  polinomio f pertencente a k[x] de grau =n tal que
  f(a_i) = b_i ,  i = 1, ... , n+1.
  (Sugestao: Tomar q = PROD_i=1,n+1,(X - a_i)  e
   f = b_1*p_1 + ... b_n+1*p_n+1 , onde:
   p_i = q_i /(q_i*a_i)  e q_i = q /(X -a_i) )
 
 
 
 
 
 
 
 
 
 
 
 
 
 
 
 ___
  Yahoo! Mail
  Mais espaço, mais segurança e gratuito: caixa postal
  de 6MB, antivírus, proteção contra spam.
  http://br.mail.yahoo.com/
 
 =
  Instruções para entrar na lista, sair da lista e
  usar a lista em
  http://www.mat.puc-rio.br/~nicolau/olimp/obm-l.html
 
 =

 ___
 Yahoo! Mail
 Mais espaço, mais segurança e gratuito: caixa postal de 6MB, antivírus,
proteção contra spam.
 http://br.mail.yahoo.com/
 =
 Instruções para entrar na lista, sair da lista e usar a lista em
 http://www.mat.puc-rio.br/~nicolau/olimp/obm-l.html
 =

=
Instruções para entrar na lista, sair da lista e usar a lista em
http://www.mat.puc-rio.br/~nicolau/olimp/obm-l.html
=


Re: [obm-l] Duvida em Limite e PARI -IG

2003-06-04 Por tôpico Cláudio \(Prática\)
Oi, Amurpe:

Se não me engano, WinCE é um sistema operacional apenas pra palmtops.
Assim, acho que não se aplica ao seu caso.

Um abraço,
Claudio.

- Original Message -
From: amurpe [EMAIL PROTECTED]
To: [EMAIL PROTECTED]
Cc: [EMAIL PROTECTED]
Sent: Tuesday, June 03, 2003 9:19 AM
Subject: Re: [obm-l] Duvida em Limite e PARI -IG


  Ao Claudio/Pratica/

 Os Arquivos que você cita logo abaixo da
 frase:...Executable for WinCE®a deverão ser apanhados ,
 também?
 Peço desculpas, pela falta de clareza , de algumas
 mensagens, é que ainda não possuo micro ,e as vezes por
 falta de tempo e outras confusões peço a outras
 pessoas para passá-las pra mim.

 Mais uma vez muito obrigado pela sua ajuda.

 Um grande abraço

 Amurpe



 Oi, Amurpe:
 
  Primeiro o limite:
 
  n[(n/(n+1))^p - 1] =
  n[(1 - 1/(n+1))^p - 1] =
  n[1 - p/(n+1) + O(1/(n+1)^2) - 1] =
  n[-p/(n+1) + O(1/n^2)] =
  -pn/(n+1) + O(1/n)
 
  Quando n - infinito, -pn/(n+1) - -p   e   1/n - 0.
 
  Logo, o limite é igual a -p.
 
  OBS: O(1/(n+1)^2) significa uma função F
 (n) tal que, quando n - infinito, o
  quociente:
  F(n)/(1/(n+1)^2) permanece limitado.
 
  No nosso caso, F
 (n) será o resto do desenvolvimento do binômio (1 -
  1/(n+1))^p, ou seja:
  F(n) = Binom(p,2)/(n+1)^2 - Binom(p,3)/(n+1)^3 +
  Naturamente, o termo dominante (quando n-
  infinito) é da ordem de
  1/(n+1)^2.
 
  Também é mais ou menos óbvio que O(1/(n+1)^2) = O
 (1/n^2).
 
  **
 
  PARI-GP:
 
  Entre na página de downloads:
  http://www.gn-50uma.de/ftp/pari/00index.html
 
  Aí, vá até a seção de Precompiled Executables, cujo c
 onteúdo está
  reproduzido abaixo:
 
  Precompiled executables:
  ...for Windows®
 
  Cygwin version for Windows95/98/NT. Please consult the
 README and README.WIN
  files. (Cygwin, like Pari/GP, is covered by the GNU Gen
 eral Public License,
  as published by the Free Software Foundation.)
 
  gp-2-1-3.exe.gz (2.1.3), 1161023 Bytes (gzip'd),
  by FTP or by HTTP
 
  cygwin1.dll.gz, 268255 Bytes (gzip'd),
  by FTP or by HTTP
 
  README.WIN, 4546 Bytes,
  by FTP or by HTTP
 
  *** Faça o download dos três arquivos acima, mas antes
 certifique-se de que
  você tem um descompactador. Caso n~~ao tenha, vá até o
 pé da página - DOS
  Utilities - e faça o download do unzip50.exe.
 
  ...Executable for WinCE®
  pari217.exe.gz (2.0.17), 1217221 Bytes (gzip'd),
  by FTP or by HTTP
 
  ...EMX/RSX executables
  Compile-
 time and runtime support (including precompiled Readline
 library),
  for DOS®/Windows® and for OS/2®, for Pari/GP 2.1.0:
 
  Please consult the README.DOS file; the ZIP archives li
 ve in their own FTP
  subdirectory, including a ZIP archive of documentation.
 
  ...Self-Installing (.exe) for Windows® (up to XP®)
  Experimental. Please consult the README.
 
  Pari.exe (2.2.5.alpha), 4643907 Bytes,
  by FTP or by HTTP
 
  ...for Macintosh®
  gp-2.0.14.sit.bin (2.0.14), 1077760 Bytes,
  by FTP or by HTTP
 
  README file for this
 
  DOS® utilities:
  gzip-1.2.4.msdos.exe, 119146 Bytes,
  by FTP or by HTTP
 
  unzip50.exe, 115132 Bytes,
  by FTP or by HTTP
 
  
 
  Um abraço,
  Claudio.
 
  - Original Message -
  From: amurpe [EMAIL PROTECTED]
  To: [EMAIL PROTECTED]
  Sent: Tuesday, May 27, 2003 9:31 AM
  Subject: [obm-l] Duvida em Limite e PARI -IG
 
 
   Alô pessoal gostaria que voces me ajudassem a resolve
 r o
   seguinte limite.
  
   Lim n[(n/n+1)^p-1].
  
   Fui variando o valor de p e cheguei a conclusão que;
  
   quando p=2 o valor do limite foi -1.
  
   para p=3 o valor foi -2.
  
   Gostaria de ver como se chega a uma conclusão geral,
 sem
   ter que ir variando o valor de p.Fiz muitas contas ma
 s
   não tive sucesso.
  
   Aproveitando a oportunidade gostaria que o
   Claudio/pratica/ me desse uma ajuda com relação a
   conseguir o PARI-IG , fui no site mas , como não sou
   safo em inglês, vi muitos arquivos e fiquei na duvida
  de
   quais arquivos eu deveria fazer o Download .
  
   Desde já , muito obrigado.
  
   um abraço,
  
   Amurpe
  
  
   _
 _
   Seleção de Softwares UOL.
   10 softwares escolhidos pelo UOL para você e sua famí
 lia.
   http://www.uol.com.br/selecao
  
  
   =
 
   Instruções para entrar na lista, sair da lista e usar
  a lista em
   http://www.mat.puc-rio.br/~nicolau/olimp/obm-l.html
   =
 
 
  ===
 ==
  Instruções para entrar na lista, sair da lista e usar a
  lista em
  http://www.mat.puc-rio.br/~nicolau/olimp/obm-l.html
  ===
 ==
 


 __
 Seleção de Softwares UOL.
 10 softwares escolhidos pelo UOL para você e sua família.
 http://www.uol.com.br/selecao


 

Re: Re:[obm-l] integral

2003-06-04 Por tôpico Cláudio \(Prática\)

- Original Message -
From: Henrique Patrício Sant'Anna Branco [EMAIL PROTECTED]
To: [EMAIL PROTECTED]
Sent: Tuesday, June 03, 2003 12:56 AM
Subject: Re: Re:[obm-l] integral


  Ta certo isso?
 
  Derivando
  f(x) = sen(x - log(1+x)),
  eu obtive
  f'(x) = (1 - 1/(1+x))cos(x - log(1+x)) =
   = (x/(1+x))*cos(x - log(1+x))  sen(x)/(1+x)
 
  Acho que o Mathematica falhou dessa vez.
 
  Tambem nao achei essa integral em nenhuma tabela - minha
  aposta eh que ela nao pode ser expressa como uma
  combinacao de funcoes elementares conhecidas.

 Segundo o Maple...

  int(sin(x)/(1+x),x)
Si(1+x)*cos(1)-Ci(1+x)*sin(1)

 Onde Si e Ci ele define como int(sin(t)/t, t=0..x) e gamma + ln(x) +
 int((cos(t)-1)/t, t=0..x).

 Agora eu pergunto... Qual a utilidade de definir tais funções? Essa
primeira
 me lembra o limite fundamental trigonométrico, mas acho que não tem nada a
 ver... Opiniões?

 Henrique.

Oi, Henrique:

Vamos checar:
f(x) = Si(1+x)cos(1) - Ci(1+x)sen(1) ==
f'(x) = Si'(1+x)cos(1) - Ci'(1+x)sen(1)

Si'(1+x) = sen(1+x)/(1+x)
Ci'(1+x) = 1/(1+x) + [cos(1+x) - 1]/(1+x) = cos(1+x)/(1+x)

Essas duas últimas igualdades são consequências da regra da cadeia e do
seguinte fato:
d(integral(0 a x) f(t))/dx = f(x)

Assim, f'(x) = [sen(1+x)/(1+x)]cos(1) - [cos(1+x)/(1+x)]sen(1) ==

f'(x) = [sen(1+x)cos(1) - cos(1+x)sen(1)]/(1+x) = sen(1+x-1)/(1+x) =
sen(x)/(1+x).

Putz! E não é que deu certo?
Isso quer dizer que eu perdi minha apostase bem que Ci e Si estão longe
de ser funções elementares.

*

Quanto à utilidade das funções Si e Ci, eu acho que elas aparecem como
soluções de algumas equações diferenciais encontradas na física-matemática.
Também vale notar a existência da função complexa exponencial integral
(Ei), dada pela fórmula:
Ei(ix) = Ci(x) + i*Si(x)   onde i^2 = -1(isso te lembra alguma coisa?)

Um abraço,
Claudio.

=
Instruções para entrar na lista, sair da lista e usar a lista em
http://www.mat.puc-rio.br/~nicolau/olimp/obm-l.html
=


Re: [obm-l] Teorema do Rearranjo

2003-06-04 Por tôpico Cláudio \(Prática\)

- Original Message -
From: Artur Costa Steiner [EMAIL PROTECTED]
To: OBM [EMAIL PROTECTED]
Sent: Tuesday, June 03, 2003 10:24 AM
Subject: [obm-l] Teorema do Rearranjo


 Bom dia a todos!
 Alguem poderia descrever para mim o teorema do rearranjo? Eu naos sei a
que
 exatamente ele se refere.
 Obrigado.
 Artur

Oi, Artur:

Imagino que você esteja se referindo à desigualdade do rearranjo. Ela diz o
seguinte:

Sejam a(1), a(2), ..., a(n)  e b(1), b(2), ..., b(n) duas sequências de
números reais (não necessariamente positivos).
Suponhamos que: a(1) = a(2) = ... = a(n)  e  b(1) = b(2) = ... = b(n)
Seja (i_1, i_2, ..., i_n) uma permutação qualquer de (1, 2, ..., n).
Então:
a(1)*b(n) + a(2)*b(n-1) + ... + a(n)*b(1) = a(1)*b(i_1) + a(2)*b(i_2) + ...
+ a(n)*b(i_n)
e
a(1)*b(i_1) + a(2)*b(i_2) + ... + a(n)*b(i_n) = a(1)*b(1) + a(2)*b(2) + ...
+ a(n)*b(n)

Uma demonstração disso pode ser encontrada na Eureka no. 5, num artigo sobre
desigualdades, escrito pelo Antonio Caminha Muniz Neto.

Um abraço,
Claudio.


=
Instruções para entrar na lista, sair da lista e usar a lista em
http://www.mat.puc-rio.br/~nicolau/olimp/obm-l.html
=


Re: [obm-l] desigualdade

2003-06-04 Por tôpico Cláudio \(Prática\)
Title: Message



Oi, Artur:

n = 0: 0! = 1 = (1 + 0/2)^(0-1)
n = 1: 1! = 1 = (1 + 1/2)^(1-1)

Para n = 2, usando a desigualdade entre as 
médias aritmética e geométrica dos "n-1" números positivos:
2, 3, ..., n-1, n,
teremos:

[ 2 * 3 * ... * (n-1) * n ]^(1/(n-1)) = [ 2 + 3 
+ ...+ (n-1) + n ]/(n-1) ==

[n!]^(1/(n-1)) = [(n-1)*(n+2)/2]/(n-1) = (n + 
2)/2 = (1 + n/2) ==

n! = (1 + n/2)^(n-1)


Um abraço,
Claudio.


  - Original Message - 
  From: 
  Artur Costa 
  Steiner 
  To: OBM 
  Sent: Monday, June 02, 2003 4:38 PM
  Subject: [obm-l] desigualdade
  
  Mostre que n! 
  = (n/2+1)^(n-1), ocorrendo desigualdade estrita para n=3. Eh 
  interessante
  Um 
  abraco
  Artur


Re: [obm-l] Problema

2003-06-03 Por tôpico Cláudio \(Prática\)



Gostaria de ajudar o para o seguinte problema:
Mostrar que:
se o inteiro n/ 3, então ( n!)^2  n^n
Atenciosamente,
Fernando.

Oi, Fernando:

Vou dar só uma dica: use o seguinte:

(1) (n!)^2 = [1*2*...*n]*[n*(n-1)*...*1] = 
[1*n]*[2*(n-1)]*...*[n*1]

(2) Para n = 3 e 
1 = k = n, k*(n+1-k) = n, com igualdade se e somente se k = 1 ou k = 
n.
Dem:
k*(n+1-k) - n = -(k^2- (n+1)*k+ n) = 
-(k - n)*(k - 1) = (n - k)*(k - 1) = 0 para 1 = k = n.
Igualdade se e somente se n - k = 0 ou 
k - 1 = 0.

Um abraço,
Claudio.


Re: [obm-l] ajuda...

2003-06-03 Por tôpico Cláudio \(Prática\)



Oi, Celso:

Substituindo y =1/x na equação da circunferência 
resulta em:
x^2 + 1/x^2 = r^2 ==
x^4 - r^2x^2 + 1 = 0 (equação 
biquadrada)

Delta = r^4 - 4 

4 raízes reais == 
Delta = 0 == 
r^4  4 == 
r  raiz(2) 

Como 1  raiz(2)  2,o menor valor 
inteiro positivo de r é 2 == alternativa (b).

Um abraço,
Claudio.

  - Original Message - 
  From: 
  Celso Junior dos Santos 
  Francisco 
  To: [EMAIL PROTECTED] 
  Sent: Monday, June 02, 2003 1:55 PM
  Subject: [obm-l] ajuda...
  
  (UERJ)-Observe o sistema:
  
  y=1/x
  x^2+y^2=r^2
  
  O menor valor inteiro de r para que o sistema acima apresente 
  quatro soluções reais é:
  a)1 b)2 c)3 d)4
  
  Gabarito: b
  
  
  Yahoo! Mail Mais espaço, 
  mais segurança e gratuito: caixa postal de 6MB, antivírus, proteção contra 
  spam.


[obm-l] Re: [obm-l] Dúvida de Matemática (Infinitos)

2003-05-31 Por tôpico Cláudio \(Prática\)
A é um conjunto infinito == existe uma bijeção entre A e um subconjunto
próprio de A

A é um conjunto finito == A não é infinito

A é um conjunto infinito enumerável == existe uma bijeção entre A e N (N =
conjunto dos números naturais)

A é um conjunto não enumerável == A é infinito mas nenhuma função de N em
A é sobrejetora


- Original Message -
From: Thiago Luís Tezza [EMAIL PROTECTED]
To: [EMAIL PROTECTED]
Sent: Friday, May 30, 2003 8:22 AM
Subject: [obm-l] Dúvida de Matemática (Infinitos)



   Olá. Estou com uma dúvida sobre o que é:

- Infinito enumerável;
- Infinito não-enumerável;
- Infinito potencial;
- Infinito atual;

   E a distinção entre conjunto finito e conjunto infinito.

   Obrigado,

   Thiago Luís Tezza

 _
 MSN Hotmail, o maior webmail do Brasil.  http://www.hotmail.com

 =
 Instruções para entrar na lista, sair da lista e usar a lista em
 http://www.mat.puc-rio.br/~nicolau/olimp/obm-l.html
 =

=
Instruções para entrar na lista, sair da lista e usar a lista em
http://www.mat.puc-rio.br/~nicolau/olimp/obm-l.html
=


[obm-l] Re: [obm-l] Dúvida de Matemática (Infinitos)

2003-05-31 Por tôpico Cláudio \(Prática\)
Caro Thiago:

Não estou bem certo quanto aos infinitos potencial e atual (acho que são
conceitos que têm mais a ver com filosofia do que com matemática), mas o
exemplo a seguir pode ser relevante:

Considere o conjunto N, dos números naturais.

Por definição, N é um conjunto infinito (considere a bijeção f: N -- P,
onde P é o conjunto dos naturais pares, dada por:
f(n) = 2n. Como P é um subconjunto próprio de N, concluímos que N é
infinito)

Assim, N é um infinito atual, ou seja, um conjunto infinito que já vem
pronto.

*

Por outro lado, dado qualquer elemento n de N, sempre podemos achar um
elemento de N maior de que n - por exemplo, n+1.

Da mesma forma, dado qualquer subconjunto finito de N, podemos sempre achar
um elemento de N que não pertence a este subconjunto. Conclusão: nenhum
subconjunto finito de N pode ser igual a N.

Nesse caso, N é um infinito potencial, ou seja, N não aparece de imediato
como sendo infinito, mas dada a propriedade acima de sempre caber mais um,
pode-se dizer que N é potencialmente infinito.

*

Espero ter ajudado mais do que confundido...

Um abraço,
Claudio.

- Original Message -
From: Thiago Luís Tezza [EMAIL PROTECTED]
To: [EMAIL PROTECTED]
Sent: Friday, May 30, 2003 8:22 AM
Subject: [obm-l] Dúvida de Matemática (Infinitos)



   Olá. Estou com uma dúvida sobre o que é:

- Infinito enumerável;
- Infinito não-enumerável;
- Infinito potencial;
- Infinito atual;

   E a distinção entre conjunto finito e conjunto infinito.

   Obrigado,

   Thiago Luís Tezza

 _
 MSN Hotmail, o maior webmail do Brasil.  http://www.hotmail.com

 =
 Instruções para entrar na lista, sair da lista e usar a lista em
 http://www.mat.puc-rio.br/~nicolau/olimp/obm-l.html
 =

=
Instruções para entrar na lista, sair da lista e usar a lista em
http://www.mat.puc-rio.br/~nicolau/olimp/obm-l.html
=


[obm-l] É isso aí, Dirichlet

2003-05-31 Por tôpico Cláudio \(Prática\)



É isso aí, Dirichlet:

Aplicações geométricas da trigonometria são 
super-importantes e os participantes da lista deveriam ter mais interesse nesse 
assunto! Acho injusto você ter que carregar esse piano sozinho. Você tem toda a 
razão de dar voz à sua frustração. Além disso,já reparou que quando alguém 
se dá ao luxo de responderàs suas mensagens, é só pra criticar? 
Ninguémdá valorà suacontribuição pra lista.E que culpa 
você tem se os outros são mal-preparados ou mal-dotados e não entendem o que 
você quer dizer? Será que o Gauss, o Newton e o Einstein se davam ao luxo de 
explicar cada detalhezinho das teorias deles? Duvido. E acho que você tem que 
adotar a mesma postura.O Newton, por exemplo, dizem que era um sujeito 
insuportável - garanto que por pura inveja dos medíocres que o cercavam! 
Eu entendo que é chato ser um incompreendido, mas esta tem sido a sina de boa 
parte dos gênios da humanidade. E adorei você ter falado que tem oito soluções 
pro problema - acho que você tem mesmo que esfregar isso na cara dos seus 
detratores. E depois, só pra humilhar, prepara um artigo a respeito e manda pra 
Eureka - Eureka? que nada, manda pra American Mathematical Monthly - já pensou? 
"Eightfold Solution to the Isosceles Triangle Problem" by Johann Peter Gustav 
Lejeune Dirichlet. E outra coisa: como Dirichlet é só um pseudônimo, acho que 
você deveria revelar em breve sua verdadeira identidade pra lista - só pra 
chocar os que te invejam - tenho certeza que você é um desses caras que têm 
medalhas de ouro em pelo menos meia dúzia de olimpíadas. Quero ver a cara do 
Okakamo quando descobrir quem você é realmente...

Um abraço do admirador,
Claudio.


  - Original Message - 
  From: 
  Johann Peter Gustav Lejeune 
  Dirichlet 
  To: [EMAIL PROTECTED] 
  Sent: Thursday, May 29, 2003 1:43 
PM
  Subject: Re: [obm-l] triangulo 
  isósceles
  
  E por isso que eu sempre me irrito!!Sera que na face dessa terra 
  habitada por maquinas nao apareceu nenhuma mensagem que nao seja de 
  minha autoria sobre geometriacom trigonometria???Alem de uma do Villard 
  e outra com um teorema que o Eduardo Wagner resolveu cearensemente???Ou que 
  quando se pergunta uma questao de trigonometria e um somatorio chato ou uma 
  conta grande que se tem preguiça de fazerNao e querendo humilhar nem me 
  julgar maismas tinha que dizer algo.Mas e melhor eu nao discutir mais do 
  que ja pude.
  Particularmente eu tenho oito soluçoes no caso em que no lugar de 70 
  tem-se 60,apenas uma trigonometrica,que eu mostro agora:BE=BC pois o 
  triangulo EBC e isoangulo,logo isosceles.
  BD/sen 80=BC/sen 30,BD=BC*2*sen 80=BC*2*cos 10.
  BE/sen t=BD/sen (10+t) e ai 1/sen t=2*cos 10/sen(10+t),
  2*sen t*cos 10=sen(10+t) e ai 
  sen 10*cos t+sen t*cos10=2*sen t*cos 10,sen 10*cos t=sen t*cos 10,
  cotg t=cotg 10, t=10.E fim!Rafael 
  [EMAIL PROTECTED] wrote:
  Num 
triângulo ABC de ângulos  = 20º, ^B = 80º e ^C =80º, traçamos do 
vértice B um segmento que forma com olado BC um ângulo de 70º e encontra 
o lado AC no pontoD. Do vértice C traçamos um segmento que forma com 
olado BC um ângulo de 50º e encontra o lado AB no pontoE. Calcule o 
ângulo 
DÊC.___Yahoo! 
MailMais espaço, mais segurança e gratuito: caixa postal de 6MB, 
antivírus, proteção contra 
spam.http://br.mail.yahoo.com/=Instruções 
para entrar na lista, sair da lista e usar a lista 
emhttp://www.mat.puc-rio.br/~nicolau/olimp/obm-l.html=
  
  
  Yahoo! Mail Mais espaço, 
  mais segurança e gratuito: caixa postal de 6MB, antivírus, proteção contra 
  spam.


Re: [Re: [obm-l] Limite da integral]

2003-05-31 Por tôpico Cláudio \(Prática\)
Obrigado, Morgado.

O exemplo realmente não poderia ser mais simples.

Um abraço,
Claudio.

- Original Message -
From: Augusto Cesar de Oliveira Morgado [EMAIL PROTECTED]
To: [EMAIL PROTECTED]
Sent: Thursday, May 29, 2003 5:26 PM
Subject: Re: [Re: [obm-l] Limite da integral]


 f_n(x) = x^n em [0, 1] converge para f(x) = 0 se 0 menor ou igual x menor
que 1 e f(1) = 1
 A sequencia das integrais, 1/(n+1), converge para 0 e a integral do limite
da sequencia tambem vale zero.
 Nao obstante, a convergencia nao eh uniforme pois
 sup Modulo [f_n(x)-f(x)] = 1 nao tende a 0.


 Em Thu, 29 May 2003 15:48:23 -0300, Cláudio_(Prática)
[EMAIL PROTECTED] disse:

  Oi, Artur:
 
  Realmente acho que você tem razão. A condição na certa é suficiente mas
  nenhum livro que eu olhei falava que é necessária.
 
  Infelizmente, não achei nenhum exemplo de sequência não-uniformemente
  convergente para o qual integral do limite = limite da integral. Você
  conhece algum?
 
  Por outro lado, achei contra-exemplos onde integral do limite  limite
da
  integral nos casos:
  1) Sequência não-unif. conv. == f_n(x) = n*x*e^(-nx^2) em [0,1]
  e
  2) Sequência unif.-conv. mas integral imprópria == f_n(x) = e^(-x/n)/n
em
  [0,+infinito)
  (assim, parece que a condição é suficiente desde que o intervalo de
  integração seja compacto)
 
  Um abraço,
  Claudio.
 
  - Original Message -
  From: Artur Costa Steiner [EMAIL PROTECTED]
  To: [EMAIL PROTECTED]
  Sent: Tuesday, May 27, 2003 5:20 PM
  Subject: Re: [Re: [obm-l] Limite da integral]
 
 
  
Oi, Tertuliano:
   
A resposta é depende.
   
Mais formalmente, seja (f_n(x)) uma sequencia de funções que
converge
  para
   o limite f(x).
   
Você quer saber se:
lim(n-inf) INTEGRAL(x1 a x2) f_n(x)dx = INTEGRAL(x1 a x2) f(x)dx.
   
Isso só será verdade se a convergência for uniforme, ou seja:
Se dado epsilon  0, existir N tal que, para todo n  N e todo x no
  domínio
   das f_n e de f , | f(x) - f_n(x) |  epsilon.
  
   Eu nao estou absolutamente certo neste momento, mas me parece que esta
eh
  uma
   condicao suficiente, porem nao necessaria. Acho que eh possivel que a
  condicao
   desejada ocorra sem que a convergencia de f_n para f seja uniforme.
Vou
   consultar quando estiver em casa.
  
   Uma questao interessante ocorre quando fazemos uma pergunta
semelhante,
  agora
   nao para a integral e sim para derivadas. O fato de f_n convergir para
f e
  de
   que as f_n sejam diferenciaveis nao garante que f'_n convirja para f',
  ainda
   que saibamos que f' exista e e que f_n = f uniformemente. Hah porem
um
   teorema que diz: Seja f_n uma sequencia de funcoes diferenciaveis em
um
   intervalo fechado I da reta real. Se (1) a sequencia de numeros reais
  f_n(a)
   for convergente para algum a em I e (2) a sequencia f'_n convergir
   uniformemente em I para uma funcao g, entao f_n converge uniformemente
em
  I
   para uma funcao f tal que f' = g em I.
   A demonstracao deste teorema eh muito bonita. Ele estabelece uma
condicao
   suficiente, embora nao necessaria.
  
   Series de potencias tem a interessante caracteristica de que
  diferenciando-se
   ou integrando-se os seus termos no interior do circulo de
convergencia,
   obtemos uma nove serie que converge para a  derivada ou para a
integral da
   funcao limite da serie original.
   Um abraco
   Artur
  
  
  
  
=
   Instruções para entrar na lista, sair da lista e usar a lista em
   http://www.mat.puc-rio.br/~nicolau/olimp/obm-l.html
  
=
 
 
=
  Instruções para entrar na lista, sair da lista e usar a lista em
  http://www.mat.puc-rio.br/~nicolau/olimp/obm-l.html
 
=
 
 

 =
 Instruções para entrar na lista, sair da lista e usar a lista em
 http://www.mat.puc-rio.br/~nicolau/olimp/obm-l.html
 =

=
Instruções para entrar na lista, sair da lista e usar a lista em
http://www.mat.puc-rio.br/~nicolau/olimp/obm-l.html
=


Re: [obm-l] Problema

2003-05-31 Por tôpico Cláudio \(Prática\)



Oi, Fernando:

Por definição de valor absoluto:
|x| = x, se x  0
|x| = -x, se x  0
|0| = 0.

Assim:
x  0 == |x| = x  0
x  0 == |x| = -x  0
x = 0 == |x| = |0| = 0

Ou seja, |x| = 0, e é igual a zero se e somente 
se x = 0.

Você está estudando alguma coisa relacionada a 
fundamentos da matemática?

Um abraço,
Claudio.


  - Original Message - 
  From: 
  Fernando 
  To: [EMAIL PROTECTED] 
  Sent: Wednesday, December 31, 1997 11:21 
  PM
  Subject: [obm-l] Problema
  
  Gostaria de ajudar o para o seguinte problema:
  Mostrar que:
  /x/ / =0 e /x/=0 se e somente se x= 0
  Atenciosamente,
  Fernando.


[obm-l] Equações do 2o. grau com raiz comum

2003-05-31 Por tôpico Cláudio \(Prática\)
- Original Message -
From: Luís Guilherme Uhlig [EMAIL PROTECTED]
To: [EMAIL PROTECTED]
Sent: Sunday, May 25, 2003 12:07 PM
Subject: [obm-l] duvidas


 
 Uma de equação de 2º grau:
 Determine a condição para que as equações:
 ax^2 +bx +c =0 e a'x^2 +b'x +c' = 0
 tenham uma raiz comum.
 

Oi, Luís:

Ficou faltando essa da sua lista.

Antes de mais nada, vamos supor que as duas equações acima são, de fato, do
2o. grau, ou seja, que a*a'  0.
Dividindo as equações por a e a' respectivamente (a fim de, entre outras
coisas, facilitar as contas no final), caíremos em:
f(x) = 0   e   g(x) = 0
onde:
f(x) = x^2 + mx + n e g(x) = x^2 + px + q
m = b/a,  n = c/a,  p = b'/a',  q = c'/a'

Se r é a raiz comum, podemos escrever os dois polinômios como:
f(x) = (x - r)(x - u)   e   g(x) = (x - r)(x - v)

Assim, teremos:
(x - v)f(x) = (x - r)(x - u)(x - v)
e
(x - u)g(x) = (x - r)(x - u)(x - v)

Ou seja:
(x - v)f(x) = (x - u)g(x) ==

(x - v)(x^2 + mx + n) = (x - u)(x^2 + px + q) ==

x^3 + (m - v)x^2 + (n - mv)x - nv =
x^3 + (p - u)x^2 + (q - pu)x - qu

Igualando coeficientes, teremos::
m - v = p - u
n - mv = q - pu
nv = qu

Da primeira equação vem:
v = u + m - p

Substituindo esta expressão para v nas outras duas teremos:
pu = mv + q - n  ==
pu = mu + m(m - p) + q - n ==
(p - m)u =  m(m - p) + (q - n)  (*)
e
qu = nv ==
qu = nu + n(m - p) ==
(q - n)u = n(m - p)  (**)

Multiplicando (*) por (q - n) e (**) por (p - m), teremos:
(p - m)(q - n)u = m(m - p)(q - n) + (q - n)^2
e
(p - m)(q - n)u = -n(m - p)^2

Ou seja, (p - m)(q - n)u = m(m - p)(q - n) + (q - n)^2 = -n(m - p)^2 ==

m(m - p)(q - n) + (q - n)^2 + n(m - p)^2 = 0 ==

(q - n)^2 + (m - p)(mq - mn + mn - np) = 0 ==

(q - n)^2 + (m - p)(mq - np) = 0

Como todas as passagens são reversíveis, podemos concluir que:
x^2 + mx + n = 0   e   x^2 + px + q = 0  tem uma raiz comum
se e somente se
(q - n)^2 + (m - p)(mq - np) = 0

Um abraço,
Claudio.

=
Instruções para entrar na lista, sair da lista e usar a lista em
http://www.mat.puc-rio.br/~nicolau/olimp/obm-l.html
=


Re: [obm-l] Norma

2003-05-30 Por tôpico Cláudio \(Prática\)
Oi, Carlos:

Exatamente. Pensei nisso hoje de manhã quando tomava banho

Pra ser mais exato, eu estava pensando no caso em que dim(V) é infinita,
onde seria conveniente que o corpo, além de ordenado, fosse completo (e,
portanto, igual a R)

Por exemplo, se V = espaço das sequências infinitas de nos. reais, podemos
considerar o subespaço W das sequências infinitas cujas séries
correspondentes são absolutamente convergentes.

Nesse caso, se x = (x1,x2,x3,...) e y = (y1,y2,y3,...),
podemos definir uma norma em W pondo: N(x) = |x1| + |x2| + |x3| + 
(idem pra N(y))

(N1) N(x) = 0 == x = (0,0,0,...)
(N2) N(ax) = |ax1| + |ax2| + |ax3| + .. = |a|N(x)
(N3) N(x+y) = |x1+y1| + |x2+y2| + ... = |x1| + |y1| + |x2| + |y2| = N(x) +
N(y)  (dado que podemos rearranjar os termos de uma série absolutamente
convergente).

Se o corpo não for completo, vão existir elementos de W cuja norma não é
definida.

*

O que é um corpo ordenado Pitagórico? Eu já ouvi falar em corpos ordenados
Arquimedianos - aqueles nos quais N (definido como o subconjunto do corpo
que obedece aos axiomas de Peano) é ilimitado superiormente, ou
equivalentemente, dados quaisquer a, b  0, existe n em N tal que na  b.

*

De fato, pra definição de norma fazer sentido, o corpo tem que ser ordenado.
Tirando os corpos contidos em R, o único outro corpo ordenado que eu conheço
(e que, por acaso, não é arquimediano) é o das funções racionais p(x)/q(x),
onde p(x) e q(x) pertencem a R[x] e q(x)  0. Assim, acho que dá pra
imaginar um espaço vetorial sobre este corpo que tenha uma norma com valores
neste corpo.  Quanto a uma aplicação interessante disso, por enquanto eu
vou ficar devendo...mas também gostaria de ver alguma.

Um abraço,
Claudio.


- Original Message -
From: Carlos César de Araújo [EMAIL PROTECTED]
To: [EMAIL PROTECTED]
Sent: Wednesday, May 28, 2003 10:13 PM
Subject: Re: [obm-l] Norma


 Olá mais uma vez, Cláudio. Li com atenção as suas observações e
resoluções,
 especialmente a seguinte passagem:


   EXERCÍCIO. As condições (N2) e (N3) acarretam N(v)=0 para todo v em
V.
  
  (N2) com a = 0 == N(0) = N(0v) = 0N(v) = 0.
  (N3) com v = -u == 0 = N(0) = N(u +(-u)) = N(u) + N(-u)
  (N2) com a = -1 == N(-u) = N(-1u) = |-1|N(u) = N(u)
  Logo, 0 = N(u) + N(u) = 2N(u) == 0 = N(u) (desde que 2  0 em F)

 O que me intrigou aqui foi a restrição desde que 2  0 em F.
Claramente,
 você deseja trabalhar com uma definição bastante geral de norma para
 espaços vetoriais ao substituir meu IR por um corpo F (qualquer) e
 ressaltar que F deve ter CARACTERÍSTICA zero. Todavia, observe: o seu
 raciocínio JÁ pressupõe uma relação de ORDEM = em F, certo? Estaremos,
 então, trabalhando num corpo ordenado? Nesse caso, a condição 2  0 é
 desnecessária, já que, num tal corpo, 210.

 Dos textos de álgebra linear tradicionais, os mais ambiciosos começam
 falando em K-espaços (onde K é QUALQUER corpo), mas, numa certa altura,
 começam a supor K=IR ou K=C. Devido à minha obsessão pela lexicologia da
 matemática, tenho notado que o termo norma aparece na literatura
aplicado
 a diferentes estruturas -- como em anéis normados --, mas no contexto
dos
 ESPAÇOS VETORIAIS, uma norma é, via de regra, uma função com valores em
 IR_+, mesmo quando o corpo de escalares é C. Digo isto mesmo sabendo da
 existência de uma ampla e interessante teoria dos corpos ordenados, a
qual,
 porém, foi desenvolvida por volta de 1920 basicamente por motivos
algébricos
 (tentativas de provar o 17o problema de Hilbert). Alguns dos teoremas mais
 importantes da álgebra linear não se generalizam a corpos arbitrários e
nem
 a corpos ordenados arbitrários. Em muitos casos, torna-se necessário
 considerar corpos ordenados PITAGÓRICOS (ou pitagorianos, se se desejar).
 Talvez este seja o motivo pelo qual muitos matemáticos restrinjam o termo
 norma, na álgebra linear, a funções com valores reais não-negativos.
 Exceto por essas generalizações relativas a corpos ordenados pitagóricos,
 você conhece outras aplicações interessantes de normas como funções
 tomando valores num corpo arbitrário?

 Carlos César de Araújo
 Matemática para Gregos  Troianos
 www.gregosetroianos.mat.br
 Belo Horizonte, MG



 =
 Instruções para entrar na lista, sair da lista e usar a lista em
 http://www.mat.puc-rio.br/~nicolau/olimp/obm-l.html
 =

=
Instruções para entrar na lista, sair da lista e usar a lista em
http://www.mat.puc-rio.br/~nicolau/olimp/obm-l.html
=


[obm-l] Re: [obm-l] Olimpíadas ao redor do mundo....

2003-05-30 Por tôpico Cláudio \(Prática\)



E aí rapaziada!! Tudo bem??Alguém ai tem disposição para 
pensar nesse??? Mostre que para todo inteiro a1, existe um 
número primo p tal que 1+a+a^2+...+a^(n-1) é 
composto. 
Valeu. 
Crom 

*

Oi, Crom:

Imagino que você queira dizer 1 + a + ... + a^(p-1) 
é composto.

Se esse for o caso, teremos:

a = 2 == 1 + 2 + 2^2 + ... + 2^10 = 2^11 -1 = 
23*89 == composto.

Agora, seja a um inteiro qualquer = 
3.

Seja p o menor primo que divide a - 1 (como a - 1 
= 2,a existência de um tal primo estará assegurada - foi por isso que 
eu separei o caso a = 2).
Então, p também irá dividir a^2 - 1, a^3 - 1, ..., 
a^(p-1) - 1.

Só que:
(a - 1) + (a^2 - 1) + ... + (a^(p-1) - 1) = 

(1 - 1) + (a - 1) + (a^2 - 
1) + ... + (a^(p-1) - 1) = 
(1 + a + a^2 + ... + a^(p-1)) - p, ou 
seja:

1 + a +  + a^(p-1) = p + (a - 1) + (a^2 - 1) + 
... + (a^(p-1) - 1) 

Como p divide cada parcela do lado direito (e, 
portanto, sua soma), concluímos que p também dividirá o lado 
esquerdo.

Como p dividea - 1, teremos que p = a - 1 
 1 + a = 1 + a + ... + a^(p-1). 
Logo, 1 + a + ... + a^(p-1)tempelo 
menos um outro fator primo além de p ==
1 + a+ ... + a^(p-1) é composto.

Um abraço,
Claudio.


[obm-l] Re: [obm-l] Números primos em um intervalo

2003-05-30 Por tôpico Cláudio \(Prática\)
Oi, Victor:

Você e o resto dos matemáticos do mundo. Eu diria que há uma grande chance
da pessoa que descobrir uma tal fórmula ganhar uma medalha Fields (se tiver
menos de 40 anos), um prêmio Abel e um monte de outras honrarias...

Falando sério, o que se conhece é apenas o comportamento assintótico da
função Pi(x) = número de primos no intervalo [0,x].
O chamado teorema dos números primos diz que:
lim(x - +infinito) Pi(x)*ln(x)/x = 1, ou seja, para n suficientemente
grande, o número de primos menores do que ou iguais a n é aproximadamente
igual a n/ln(n), e o erro relativo tende a 0 quando n - infinito.

Para um intervalo finito, não tem jeito: tem que calcular.
No seu caso, existem 95 primos menores do que 500, o menor deles sendo 2 e o
maior 499.

Um abraço,
Claudio.

- Original Message -
From: Victor Luiz [EMAIL PROTECTED]
To: [EMAIL PROTECTED]
Sent: Wednesday, May 28, 2003 8:42 PM
Subject: [obm-l] Números primos em um intervalo


 -BEGIN PGP SIGNED MESSAGE-
 Hash: SHA1

 Olá pessoal, eu gostaria de saber se existe alguma fórmula mágica mesmo
 que seja complicada pra calcular o número de números primos em um
intervalo.
 Esses dias eu vi um exercício que dizia mais ou menos Quantos números
 primos naturais existem no intervaldo de 1 a 500? e por isso eu quis
saber
 se existe alguma maneira mais prática de calcular do que ficar tentando
 número por número.


 Obrigado pela a atenção,
 Victor Luiz Salgado de Lima.

 PS: Não sei se isso acontece com vocês mas comigo os e-mails chegam na
lista
 com um atraso bem grande, por exemplo esse último que eu mandei sobre o
 delta foi enviado às 11:35 mas só chegou às 13:25... Não é nada demais eu
só
 quero saber se aqui no meu computador está com problema ou é assim mesmo.

 - 
 Spam sux. www.wecanstopspam.org
 -BEGIN PGP SIGNATURE-
 Version: GnuPG v1.2.1 (MingW32) - GPGOE 0.4.1

 iD8DBQE+1UlWpBwZ7xrHmVsRArWTAJwN1ZGDMX3IUHBccPfddPSR+2RRGACdERkF
 kIs+I2znWaWm3L3JS3ObHdI=
 =Vy97
 -END PGP SIGNATURE-

 =
 Instruções para entrar na lista, sair da lista e usar a lista em
 http://www.mat.puc-rio.br/~nicolau/olimp/obm-l.html
 =

=
Instruções para entrar na lista, sair da lista e usar a lista em
http://www.mat.puc-rio.br/~nicolau/olimp/obm-l.html
=


Re: [Re: [obm-l] Limite da integral]

2003-05-30 Por tôpico Cláudio \(Prática\)
Oi, Artur:

Realmente acho que você tem razão. A condição na certa é suficiente mas
nenhum livro que eu olhei falava que é necessária.

Infelizmente, não achei nenhum exemplo de sequência não-uniformemente
convergente para o qual integral do limite = limite da integral. Você
conhece algum?

Por outro lado, achei contra-exemplos onde integral do limite  limite da
integral nos casos:
1) Sequência não-unif. conv. == f_n(x) = n*x*e^(-nx^2) em [0,1]
e
2) Sequência unif.-conv. mas integral imprópria == f_n(x) = e^(-x/n)/n em
[0,+infinito)
(assim, parece que a condição é suficiente desde que o intervalo de
integração seja compacto)

Um abraço,
Claudio.

- Original Message -
From: Artur Costa Steiner [EMAIL PROTECTED]
To: [EMAIL PROTECTED]
Sent: Tuesday, May 27, 2003 5:20 PM
Subject: Re: [Re: [obm-l] Limite da integral]



  Oi, Tertuliano:
 
  A resposta é depende.
 
  Mais formalmente, seja (f_n(x)) uma sequencia de funções que converge
para
 o limite f(x).
 
  Você quer saber se:
  lim(n-inf) INTEGRAL(x1 a x2) f_n(x)dx = INTEGRAL(x1 a x2) f(x)dx.
 
  Isso só será verdade se a convergência for uniforme, ou seja:
  Se dado epsilon  0, existir N tal que, para todo n  N e todo x no
domínio
 das f_n e de f , | f(x) - f_n(x) |  epsilon.

 Eu nao estou absolutamente certo neste momento, mas me parece que esta eh
uma
 condicao suficiente, porem nao necessaria. Acho que eh possivel que a
condicao
 desejada ocorra sem que a convergencia de f_n para f seja uniforme. Vou
 consultar quando estiver em casa.

 Uma questao interessante ocorre quando fazemos uma pergunta semelhante,
agora
 nao para a integral e sim para derivadas. O fato de f_n convergir para f e
de
 que as f_n sejam diferenciaveis nao garante que f'_n convirja para f',
ainda
 que saibamos que f' exista e e que f_n = f uniformemente. Hah porem um
 teorema que diz: Seja f_n uma sequencia de funcoes diferenciaveis em um
 intervalo fechado I da reta real. Se (1) a sequencia de numeros reais
f_n(a)
 for convergente para algum a em I e (2) a sequencia f'_n convergir
 uniformemente em I para uma funcao g, entao f_n converge uniformemente em
I
 para uma funcao f tal que f' = g em I.
 A demonstracao deste teorema eh muito bonita. Ele estabelece uma condicao
 suficiente, embora nao necessaria.

 Series de potencias tem a interessante caracteristica de que
diferenciando-se
 ou integrando-se os seus termos no interior do circulo de convergencia,
 obtemos uma nove serie que converge para a  derivada ou para a integral da
 funcao limite da serie original.
 Um abraco
 Artur



 =
 Instruções para entrar na lista, sair da lista e usar a lista em
 http://www.mat.puc-rio.br/~nicolau/olimp/obm-l.html
 =

=
Instruções para entrar na lista, sair da lista e usar a lista em
http://www.mat.puc-rio.br/~nicolau/olimp/obm-l.html
=


[obm-l] Re: [obm-l] Re: [obm-l] Re: [obm-l] Olimpíadas ao redor do mundo....

2003-05-30 Por tôpico Cláudio \(Prática\)



É isso aí. A mesma solução, só queem 
linguagem decongruências.

De fato, com congruências fica até mais fácil 
mostrar o seguinte:

Para todo inteiro a  2, existe um primo p tal 
que:
1 + 2a + 3a^2 + ... + pa^(p-1) é composto (e 
múltiplo de p).

Será que pra a = 2 também vale?

Um abraço,
Claudio.



  - Original Message - 
  From: 
  Domingos Jr. 
  
  To: [EMAIL PROTECTED] 
  Sent: Thursday, May 29, 2003 3:02 
PM
  Subject: [obm-l] Re: [obm-l] Re: [obm-l] 
  Olimpíadas ao redor do mundo
  
  Cláudio, eu tive a mesma idéia que você, mas 
  havia expressado de outra forma...
  
  seja p um primo tal que a ~ 1 (mod 
p)
  
  a menos de a = 2, esse primo existe, basta pegar 
  um primo que divida a - 1 (como vc bem notou, esse primo existe sempre para a 
   2).
  
  agora note que
  1 + a + ... + a^(p-1) ~ 1 + 1 + ... + 1 = p ~ 0 
  (mod p)
  ou seja p divide o somatório, como sabemos que p 
   1 + a + ... + a^(p-1), temos que o número é composto.
  
  
  [ ]'s
  
- Original Message - 
From: 
Cláudio (Prática) 
To: [EMAIL PROTECTED] 
Sent: Thursday, May 29, 2003 12:58 
PM
Subject: [obm-l] Re: [obm-l] Olimpíadas 
ao redor do mundo

E aí rapaziada!! Tudo bem??Alguém ai tem disposição 
para pensar nesse??? Mostre que para todo inteiro a1, 
existe um número primo p tal que 1+a+a^2+...+a^(n-1) é 
composto. 
Valeu. 
Crom 

*

Oi, Crom:

Imagino que você queira dizer 1 + a + ... + 
a^(p-1) é composto.

Se esse for o caso, teremos:

a = 2 == 1 + 2 + 2^2 + ... + 2^10 = 2^11 -1 
= 23*89 == composto.

Agora, seja a um inteiro qualquer = 
3.

Seja p o menor primo que divide a - 1 (como a - 
1 = 2,a existência de um tal primo estará assegurada - foi por 
isso que eu separei o caso a = 2).
Então, p também irá dividir a^2 - 1, a^3 - 1, 
..., a^(p-1) - 1.

Só que:
(a - 1) + (a^2 - 1) + ... + (a^(p-1) - 1) = 

(1 - 1) + (a - 1) + 
(a^2 - 1) + ... + (a^(p-1) - 1) = 
(1 + a + a^2 + ... + a^(p-1)) - p, ou 
seja:

1 + a +  + a^(p-1) = p + (a - 1) + (a^2 - 
1) + ... + (a^(p-1) - 1) 

Como p divide cada parcela do lado direito (e, 
portanto, sua soma), concluímos que p também dividirá o lado 
esquerdo.

Como p dividea - 1, teremos que p = a 
- 1  1 + a = 1 + a + ... + a^(p-1). 
Logo, 1 + a + ... + a^(p-1)tempelo 
menos um outro fator primo além de p ==
1 + a+ ... + a^(p-1) é 
composto.

Um abraço,
Claudio.


Re: [obm-l] Norma

2003-05-29 Por tôpico Cláudio \(Prática\)
Oi, Carlos:

Obrigado pela clarificação.
Eu tinha certeza de que havia uma forma não-braçal de se resolver o
problema.

Um abraço,
Claudio.

- Original Message -
From: Carlos César de Araújo [EMAIL PROTECTED]
To: [EMAIL PROTECTED]
Sent: Tuesday, May 27, 2003 10:51 PM
Subject: Re: [obm-l] Norma


 Prezado Cláudio,

 Você QUASE acertou! Primeiro, recordemos que uma NORMA (num IR-espaço V) é
 uma função N: V- IR que satisfaz TRÊS condições, quais sejam (onde todas
as
 letras são quantificadas universalmente):

 (N1) N(v)=0 == v=0;
 (N2) N(av)=abs(a)N(v), com a em IR;
 (N3) N(u+v)=N(u)+N(v).

 EXERCÍCIO. As condições (N2) e (N3) acarretam N(v)=0 para todo v em V.

(N2) com a = 0 == N(0) = N(0v) = 0N(v) = 0.
(N3) com v = -u == 0 = N(0) = N(u +(-u)) = N(u) + N(-u)
(N2) com a = -1 == N(-u) = N(-1u) = |-1|N(u) = N(u)
Logo, 0 = N(u) + N(u) = 2N(u) == 0 = N(u) (desde que 2  0 em F)

 OBSERVAÇÃO 1. Satisfeitas APENAS as condições (N2) e (N3), a função N
diz-se
 uma SEMINORMA. Observo que os matemáticos utilizam prefixos como semi,
 quase, pseudo ou pré para derivar noções enfraquecidas de outras.
 Exemplos imediatos: semilinear, semigrupo, pseudométrica, pré-ordem.

 Seu Teorema. Se N é uma norma e T é uma transformação linear, então é
 fácil ver que a correspondência

 v -- N(T(v))

 define uma seminorma: a composta NoT satisfaz (N2) e (N3).

De fato:
(N2) NoT(av) = N(T(av)) = N(aT(v)) = |a|N(T(v)) = |a|NoT(v)
(N3) NoT(u+v) = N(T(u+v)) = N(T(u)+T(v)) = N(T(u)) + N(T(v)) = NoT(u) +
NoT(v)

 Vejamos se
 satisfaz (N1). Suponha N(T(v))=0. Então T(v)=0 (pois N é uma norma).
 Claramente, NoT será uma norma se pudermos provar que

 T(v)=0 == v=0

 para todo v em V. Mas esta condição sobre T equivale a supor T injetora.
Em
 resumo:

 Teorema: Se N:V-IR é uma norma e T: V-V é linear e INJETORA, então NoT é
 uma norma.


Ontem em casa eu pensei num contraexemplo pro meu teorema:
V = R^2, N(x,y) = raiz(x^2+y^2) e T(x,y) = (x,0).
Nesse caso, NoT(x,y) = |x| e teríamos NoT(0,1) = 0 mesmo sendo (0,1) 
(0,0), contrariando (N1).

Então, eu imaginei que seria preciso adicionar a condição de T ser
inversível, mas agora vejo que isso só é necessário se V tem dimensão finita
(caso em que T injetora == T sobrejetora). Com dim(V) infinita, podemos
ter T linear e injetora sem ser sobrejetora (por exemplo, V = espaço das
sequencias infinitas de nos. reais e T:V -- V definida por
T(a_1,a_2,a_3,) = (0,a_1,a_2,a_3,...) == T não é sobrejetora).

 OBSERVAÇÃO 2. Se não me engano, este fato é oferecido como exercício em
 textos básicos de álgebra linear como os de Lang.

 Abraços,

 Carlos César de Araújo
 Matemática para Gregos  Troianos
 www.gregosetroianos.mat.br
 Belo Horizonte, MG

 ---
 - Original Message -
 From: Cláudio (Prática) [EMAIL PROTECTED]
 To: [EMAIL PROTECTED]
 Sent: Tuesday, May 27, 2003 4:37 PM
 Subject: Re: [obm-l] Norma


 Oi, Tertuliano:

 Naturalmente, no braço deve sair.

 No entanto, repare que a sua norma (vamos chamá-la de F(a,b)) é igual a:
 F(a,b) = raiz( a^2 + (raiz(2)a + b)^2 )

 Se voce definir a transformação linear T: R^2 -- R^2 como sendo:
 T(x,y) = (x,raiz(2)x + y), você vai ver que:

 F(a,b) = N(T(a,b)), onde N(x,y) é a norma euclidiana usual, a qual provém
do
 produto interno usual em R^2.

 Agora, seria legal se existisse um teorema que dissesse o seguinte:
 Dado um espaço vetorial normado V sobre um corpo F e um operador linear T:
 V -- V.
 Se a norma N: V^2 -- F provém de um produto interno de V, então, a
função:
 NoT: V^2 -- F, definida como NoT(x) = N(T(x)) também é uma norma.

 Se existir um tal teorema, então acabou (e o que é melhor: sem nenhum
 braço).

 Minha pergunta é: Esse teorema existe?

 Um abraço,
 Claudio.

 - Original Message -
   From: Tertuliano Carneiro
   To: [EMAIL PROTECTED]
   Sent: Tuesday, May 27, 2003 12:24 PM
   Subject: [obm-l] Norma


   Olá para todos!!!


   Seja  /x/ = [3a^2 + 2(sqwert2)ab +b^2]^1/2, onde x = (a,b) é um vetor do
 R2 e /x/ é o módulo de x. Verificar se isso define uma norma.


   Sem mais!




 --
--
 --
   Yahoo! Mail
   O melhor e-mail gratuito da internet: 6MB de espaço, antivírus, acesso
 POP3, filtro contra spam.

 =
 Instruções para entrar na lista, sair da lista e usar a lista em
 http://www.mat.puc-rio.br/~nicolau/olimp/obm-l.html
 =

=
Instruções para entrar na lista, sair da lista e usar a lista em
http://www.mat.puc-rio.br/~nicolau/olimp/obm-l.html
=


  1   2   3   4   >